Você está na página 1de 216

£ EDiTO?

J;
VV* CIÊNCIA MODERNA
Problemas Resolvidos de Combinatória - 2a Edição

Copyright© Editora Ciência Moderna Ltda., 2018.


Todos os direitos para a língua portuguesa reservados pela EDITORA CIÊNCIA
MODERNA LTDA. •
De acordo com a Lei 9.610, de 19/2/1998, nenhuma parte deste livro poderá ser
reproduzida, transmitida e gravada, por qualquer meio eletrônico, mecânico, por
fotocópia e outros, sem a prévia autorização, por escrito, da Editora.

Editor: Paulo André P. Marques


'■ Supervisão Editorial: Aline Vieira Marques
Copidesque: Sandra Valéria Ferreira de Oliveira
Capa: Fernando Souza
Diagramação: Eduardo Luis Estrada
Assistente Editorial: Laura Souza

Várias Marcas Registradas aparecem no decorrer deste livro. Mais do que


simplesmente listar esses nomes e informar quem possui seus direitos de
exploração, ou ainda imprimir os logotipos das mesmas, o editor declara
estar utilizando tais nomes apenas para fins editoriais, em benefício exclusivo
do dono da Marca Registrada, sem intenção de infringir as regras de sua
utilização. Qualquer semelhança em nomes próprios e acontecimentos será
mera coincidência.

FICHA CATALOGRÁFICA

SANTOS, José Plínio de Oliveira; ESTRADA, Eduardo Luis


Problemas Resolvidos de Combinatória - 2a Edição
Rio de Janeiro: Editora Ciência Moderna Ltda., 2018

1. Matemática; Análise Combinatória.


I — Título

ISBN: 978-85-399-0168-5 CDD 510.


511.6

Editora Ciência Moderna Ltda.


R. Alice Figueiredo, 46 — Riachuelo
Rio de Janeiro, RJ - Brasil CEP: 20.950-150
Tel: (21) 2201-6662 / Fax: (21) 2201-6896
LCM@LCM.COM.BR
www.lcm.com.br 01/18
Para minha esposa Antonieta,

e para meus pais Altair e Maria de Lourdes


Sumário

Parte I Problemas 1
Combinatória básica 3
Princípio da Inclusão e Exclusão 26
Funções geradoras e partições 30
Princípio da Casa dos Pombos 35
Probabilidade 37
Parte II Resoluções 45
Combinatória básica 47
Princípio da Inclusão e Exclusão 130
Funções geradoras e partições 149
Princípio da Casa dos Pombos 172
Probabilidade 178
Referências Bibliográficas 203
Prefácio

Este livro de problemas de combinatória, com soluções, foi escrito com o objetivo
principal de servir de complementação a textos básicos de Matemática Discreta
adotados na graduação. De forma particular, tivemos em mente o livro Introdução
à Análise Combinatória, publicado pela Editora Ciência Moderna1, do qual o
primeiro autor deste livro é um dos co-autores e que tem sido adotado por várias
universidades além da Unicamp. Vale destacar que muitos dos problemas aqui
apresentados poderão servir aos professores do ensino médio e principalmente
na preparação para o exame de vestibular. O texto inclui questões que, em sua
maioria, podem ser resolvidas apenas com ferramentas vistas em cursos iniciais de
Matemática Discreta. Incluímos muitas questões relacionadas a aplicações sim­
ples dos Princípios Aditivo e Multiplicativo. Em muitos problemas fazemos uso
do Princípio da Inclusão e Exclusão nos quais, por exemplo, obtemos fórmulas
explícitas para a função 0 de Euler e para o número de funções sobrejetoras
f : A —> B. Funções geradoras são usadas em várias ocasiões, tendo como im­
portante e simples aplicação a demonstração da existência e unicidade da re­
presentação de um inteiro em uma base qualquer b (b inteiro > 2). Outras in­
teressantes aplicações envolvem o conceito de partições de um inteiro. Incluímos,
também, questões sobre probabilidade, que são pouco exploradas em textos de
Matemática Discreta. O Princípio da Casa dos Pombos, que nos permite obter
informações relacionadas a questões de existência, é utilizado em vários proble­
mas.
Nesta segunda edição, além de várias correções, decidimos separar os proble­
mas por grupos com o objetivo de facilitar, aos leitores, a localização de questões
de seu interesse. A um professor do ensino médio, por exemplo, a localização de
questões de seu particular interesse será facilitada pelo conjunto que chamamos
“Combinatória básica”, em que são encontrados 129 problemas daquele nível.
São 22 problemas envolvendo o uso do Princípio da Inclusão e Exclusão. So­
bre funções geradoras e partições, são apresentados 20 problemas. Questões de

1 Tendo sido, as três primeiras edições, publicadas pela Editora da Unicamp.


Problemas Resolvidos de Combinatória

xistência que envolvem o Princípio da Casa dos Pombos estão contidas nos 9
roblemas daquele grupo. Finalmente, no grupo de probabilidade, são dados 29
roblemas.
Seria interessante mencionar que 1800 exemplares do livro “Introdução à
Lnálise Combinatória”, mencionado acima, e para o qual este foi escrito com
i objetivo de servir de complementação, foram adquiridos pelo IMPA (Instituto
le Matemática Pura e Aplicada) para distribuição aos alunos classificados nas
Olimpíadas Brasileiras de Matemática das Escolas Públicas e que participam do
5rograma de Iniciação Científica.
Agradecimentos

Agradecemos, de modo particular, aos professores Andréia C. Ribeiro, Jair C.


Filho, Idani T. C. Murari e Paulo Mondek, por importantes sugestões dadas.
Aos pós-graduandos Robson da Silva e Marcelo Barreto, pela cuidadosa leitura
e pelas valiosas sugestões, nosso especial agradecimento. A professora Edith O.
Santos, agradecemos pela revisão ortográfica.

l.’
Parte I

Problemas
Parte I. Problemas - Combinatória básica 3

Combinatória básica
1. Há 5 estradas distintas ligando as cidades A e B, 3 distintas ligando B e C
e 2 distintas ligando AeC, diretamente.

(a) Quantas são as possíveis rotas ligando as cidades A e C?


(b) Quantas são as possíveis rotas que partem de A, vão até C e voltam a
X? (considere que cada estrada possa ser utilizada nos dois sentidos)
(c) Quantas das rotas de (b) passam pela cidade B, ao menos uma vez?
(d) Quantas das rotas de (b) não utilizam uma mesma estrada duas vezes?

2. De quantas maneiras podemos selecionar duas cartas, sem reposição, de um


baralho de 52 cartas, de modo que:

(a) a primeira carta seja um valete e a segunda não seja uma dama?
(b) a primeira carta seja de copas e a segunda não seja um rei?

3. Considere todos os anagramas de 4 letras (podendo haver repetições) que


são possíveis de se formarem com as 23 letras do nosso alfabeto.

(a) Em quantos deles a primeira e última letras são vogais?


(b) Em quantos deles as vogais aparecem, obrigatória e exclusivamente,
na primeira e última posições?

4. Um cesto contém 16 maçãs diferentes e 13 bananas diferentes. De quantas


maneiras distintas pode Severino escolher uma maçã ou uma banana e de
quantas maneiras ele pode escolher uma maçã e uma banana?

5. Listando os números inteiros de 1 a 100.000, quantas vezes o dígito 5


aparece?

6. Quantas coleções não vazias de letras podem ser formadas com n A’s, n
B’s, n C’s e n D's?

7. Quantos números distintos podem ser formados pelo produto de dois ou


mais números do multiconjunto2 {3,4,4,5,5,6,7,7,7}?

8. Quantos números naturais distintos podem ser obtidos pela soma de 2 ou


mais números do conjunto {1,3,5,10,20,50,90}?
2A denominação multiconjunto é utilizada ao nos referirmos a um conjunto que considera
relevante o número de cópias de cada elemento em sua composição.
4 Problemas Resolvidos de Combinatória

9. 7 moças e 5 rapazes vão jogar vôlei. De quantas maneiras eles podem ser
divididos em 2 grupos de 6 jogadores cada, de modo que os rapazes não
fiquem todos no mesmo grupo?

10. (a) Encontre o número de retângulos que podem ser formados num tabu­
leiro de xadrez 8x8, isto é, o número de retângulos da forma i x j, para
i e j inteiros tais que l<i<8el<j<8 (desconsidere simetrias do
tabuleiro de xadrez ou, em outras palavras, considere-o fixo em relação
àquele que efetua a contagem dos retângulos).
(b) Generalize o resultado do item anterior paia um tabuleiro de xadrez
m x n (na verdade, um retângulo quadriculado m x n cujas casas são
alternadas nas cores preta e branca).

11. Maria tem 7 livros diferentes e Alberto tem 9 livros diferentes. De quantas
maneiras Maria e Alberto podem trocar 3 livros entre si?

12. De quantas maneiras 8 contas distintas podem ser colocadas num cordão
elástico de modo a formar uma pulseira?

13. De quantas maneiras se pode colocar 12 peças brancas e 12 peças pretas


nas casas pretas de um tabuleiro de damas? (apesar da semelhança entre
as cores das casas do tabuleiro, considere distintas todas as suas casas)

14. De quantas maneiras as letras da palavra INDIVISIBILIDADE podem ser


permutadas de modo que duas letras I nunca fiquem juntas?

15. Dispondo de 3 cópias de um livro, duas cópias de um segundo livro e uma


cópia de um terceiro, de quantas maneiras se pode distribuí-los para um
grupo de 12 pessoas de tal modo que:

(a) nenhuma pessoa receba mais do que um livro?


(b) nenhuma pessoa receba mais do que uma cópia de cada livro?

16. Um coro possui 10 membros. De quantas maneiras se pode selecionar 3


grupos distintos de 6 membros cada, por ocasião de 3 eventos distintos?

17. Considere a expressão ‘a aranha arranhou o jarro’ para responder o que


segue:

(a) Qual o número total de seleções não vazias que podem ser feitas a
partir das letras de tal expressão?
(b) A partir dessas mesmas letras, quantas seleções de 3 letras podem ser
feitas?
Parte I. Problemas - Combinatória básica 5

(c) Quantas permutações de 3 letras podem ser realizadas?

18. A caixa A contém as letras da palavra amanhã, B contém as da palavra pa-


taca (antiga moeda de prata, que valia 320 réis) e C contém as da palavra
sarada. De quantas maneiras as letras das caixas A, B e C podem ser
trocadas entre si, de modo a permanecerem, cada uma, com seis letras
(considere õ=a)?

19. De quantas maneiras as letras da palavra CADAVERICO podem ser arran­


jadas de modo que não se altere a ordem das vogais que aparecem (considere
É=E)?
20. De quantas maneiras 10 livros distintos podem ser colocados em 5 caixas
idênticas, contendo dois livros cada uma?

21. De quantas maneiras as letras da palavra SERRILHOU podem ser permu-


tadas, mantendo-se as vogais em sua ordem natural e não permitindo que
duas letras R fiquem juntas?

22. Considerando a palavra PROPOR, responda:

(a) Quantas seleções de 4 letras podem ser feitas a partir de suas letras?
(b) Quantas permutações de 4 letras podem ser feitas a partir de suas
letras?

23. Considerando a palavra fictícia libboolicco, responda o que segue:

(a) Quantas permutações podem ser feitas com suas letras, de modo que
consoantes e vogais se alternem?
(b) Quantas permutações de 4 letras podem ser feitas sem que as 3 letras
o apareçam juntas?

24. De um conjunto de 30 números consecutivos, de quantas maneiras podemos


selecionar 3 distintos de modo que a soma dos números seja par?

25. (a) Qual o número de soluções inteiras positivas de aq + x% H------- 1- xn — p


(xí inteiro maior do que ou igual a 1, para cada z), sendo p um inteiro
não negativo (p > n)?
(b) Resolva novamente o item anterior, supondo que x, possa também ser
igual a 0, para cada i.
(c) Forneça uma interpretação combinatória para as soluções encontradas
em cada um dos itens anteriores.
6 Problemas Resolvidos de Combinatória

26. Dispondo de um número ilimitado de moedas de cada um de 3 tipos dis­


tintos, de quantas maneiras podemos selecionar 20 moedas?

27. Considere neste problema o lançamento de dados idênticos.

(a) Quantas somas diferentes podem ser obtidas através do lançamento de


dois dados cujas faces contêm os valores 0, 1, 3, 7, 15 e 31? Discuta,
também, o caráter único de ocorrência de cada soma, isto é, se há
possibilidade de uma soma ocorrer de duas maneiras distintas.
(b) Idem ao item (a), supondo agora o lançamento de 3 dados com faces
contendo os valores 1, 4, 13, 40, 121 e 364.

28. Considere um baralho padrão de 52 cartas.

(a) De quantas maneiras podem ser selecionadas 13 cartas, sendo uma de


cada tipo? (considere como tipos: ás, valete etc.)
(b) De quantas maneiras podem ser selecionadas 4 cartas, sendo uma de
cada naipe?
(c) Em quantas das maneiras do item (b) as 4 cartas não contêm nenhum
par (isto é, dois ases, dois valetes etc.)?
(d) De quantas maneiras podemos realizar o item (b) de modo que as 4
cartas formem um par de cartas pretas e um par de cartas vermelhas
(isto é, dois ases pretos, dois valetes vermelhos etc.)?

29. De quantas maneiras um baralho de 52 cartas pode ser igualmente dis­


tribuído entre 13 jogadores, de modo que:

(a) cada um receba uma carta de cada naipe?


(b) alguém receba uma carta de cada naipe e ninguém mais receba cartas
de mais do que um naipe?

30. De quantas maneiras 18 objetos distintos podem ser divididos entre 5 pes­
soas de modo que:

(a) 4 pessoas fiquem com 4 objetos cada e uma fique com 2 objetos?
(b) 3 pessoas recebam 4 objetos cada e as outras duas recebam 3 cada
uma?

31. Considerando que haja 14 tipos de objetos e 2 objetos de cada tipo, encontre
o total de seleções nas quais pelo menos um objeto é selecionado.
Parte I. Problemas - Combinatória básica 7

32. Havendo 20 tipos de objetos e 9 objetos de cada tipo, mostre que o número
total de seleções distintas não vazias que podem ser feitas é igual a
99.999.999.999.999.999.999 (um número com vinte algarismos iguais a 9).

33. Considerando um tabuleiro de damas fixo em determinada posição, respon­


da ao que segue:

(a) De quantas maneiras 2 quadrados podem ser selecionados, sendo um


preto e um branco?
(b) Idem ao item (a), supondo agora que os quadrados selecionados não
possam estar na mesma vertical ou horizontal.

34. Considere todos os números naturais menores do que um milhão a fim de


responder as seguintes questões:

(a) Quantos desses números podem ser expressos utilizando somente os


dígitos 8 e 9?
(b) Repita o item (a) supondo que só possam ser utilizados os dígitos 7, 8
e 9?
(c) Repita o item (a) supondo que só possam ser utilizados os dígitos 0, 8
e 9?

35. Em quantos dos números menores do que um milhão não ocorrem números
idênticos pareados (isto é, não aparecem os blocos 11, 22 etc.)?

36. (a) Calcule a soma de todos os números de 4 algarismos distintos formados


com os dígitos 1, 2, 3 e 4.
(b) Calcule a soma de todos os números de 4 algarismos formados a partir
de 2 dígitos iguais a 1 e 2 dígitos iguais a 4.

37. De quantas maneiras 27 livros diferentes podem ser distribuídos entre as


pessoas A, B e C de modo que A e C fiquem, juntos, com o dobro de livros
de B e ninguém fique com todos os livros?

38. Considere um tabuleiro de xadrez ausente de peças e fixo em determinada


posição para responder o que segue:

(a) De quantas maneiras podemos nele dispor uma torre branca e uma
torre preta de modo que cada uma esteja em condições de atacar a
outra? - ■ ; ,
Problemas Resolvidos de Combinatória

(b) De quantas maneiras podem ser dispostos uma rainha branca e um


bispo preto de modo que uma peça esteja apta a tomar a outra? E de
quantas maneiras de modo que cada peça não possa tomar a outra?
(c) De quantas maneiras um cavalo branco e um cavalo preto podem ser
colocados no tabuleiro de modo que um esteja em posição perigosa em
relação ao outro?

(Sugestão: Para resolver este exercício, é muito útil que se trabalhe com um
labuh iro de xadrez de maneira concreta, executando movimentos com as
peças a fim de compreender as situações em questão)

(a) Dentre os primeiros 100 números naturais, de quantas maneiras pode­


mos selecionar 3 de modo que a soma obtida seja divisível por 3?
(b) A partir do 3/? números naturais consecutivos, de quantas maneiras
podemos selecionar 3 cuja soma seja divisível por 3?

Dispõe-se de m parcelas de objetos distintos, das quais a primeira contém


>. a segunda contem 2n. a terceira 3n. e assim por diante. Encontre o
■ úmero de maneiras de selecionar n objetos de cada uma das parcelas.

' quantas maneiras pq + r objetos podem ser distribuídos entre p pessoas


. maneira mais igualitária possível'! (r < p)

número dc maneiras de se escolherem 1 objetos distintos dentre n difercn-


- c igual a do número de maneiras de se escolherem 4 objetos a partir
_'/? objetos, sendo estes de n tipos distintos, havendo 2 objetos de cada
><>. Com base nesses chulos, encontre o valor de n.

Em certo local de uma fazenda, há 9 tocas dc coelho distintas, sendo


mu- c ada uma comporta no máximo um coelho em seu interior. No
m<--mo local, existem 7 coelhos brancos indistinguíveis e 2 coelhos
i !-' to.' indistinguíveis. Encontre o mimeio total de disposições dos
nas 9 tocas, considerando que nem todas as tocas precisam
< • í.a <>c ipadas.
(b) Repita a resolução do item (a) supondo agora que haja 7 coelhos bran­
cos indistinguíveis, 1 preto e 1 mestiço.
(c) Agora, obtenha uma generalização para o caso dc haver n + 1 tocas
distintas, n coelhos brancos indistinguíveis e 1 coelho preto.

44. Quantos paralelepípcdos diferentes podem ser construídos de tal maneira


que cada uma de suas arestas seja um inteiro positivo não excedendo 10?
Parte I. Problemas - Combinatória básica 9

45. Considere um aparelho para tocar CDs (Compact Disk Plaver) com espa- .,
para 6 CDs. A bandeja que os armazena tem formato circular, sendo qu< c
espaços para os CDs são círculos de mesmo raio cujos centros se encom u.
igualmente espaçados na circunferência única que os contêm. Dispondo <i<
80 CDs distintos, responda o que segue:

(a) Considerando que os espaços para os CDs sejam numerados de L


de quantas maneiras o aparelho pode ser carregado com 6 CDs?
(b) Supondo agora que os espaços sejam indistinguíveis, de quantas maix 1
ras podemos fazer o mesmo carregamento do item (a)?

46. Quantos são os números de 6 algarismos que possuem pelo menos d


dígitos consecutivos iguais em sua composição?

47. Dispõem-se os inteiros de 1 a 20 em um tabuleiro retangular quadricula !•


4 x 5 (4 linhas por 5 colunas). Não importa a ordem em que as linha-
figuram*, embora importem os números que são dispostos em cada linha
a ordem em que eles aí se encontram. Nessas condições, calcule o núnu ■
de configurações dos 20 números nas 20 casas do tabuleiro.
* Em outras palavras, dada uma configuração do tabuleiro, se permutarnios suas liul».-
obtemos configurações cuja distinção deve ser ignorada para efeitos de contagem.

48. (a) Quantas sequências binárias formadas pelos algarismos 0 e l cxisb


tais que seu comprimento é n e o número de algarismos 0 nas mesi..
é k (n > À.j?
(b) Quantas seqüências ternárias formadas pelos algarismos 0. 1 e 2 exi
tem tais que seu comprimento é n e o número de algarismos 0 ?i ■
mesmas é k (n > £)?

49. Uma máquina industrial apresenta seis botões numerados de 1 a G em -ir


mesa de comandos. Dar um comando à máquina significa executar um
sequência de quatro ações, sendo que cada ação consiste em pressionar :tm
dos botões ou dois botões simultaneamente. Considerando que ordens <*■!'•
rentes de ações implicam em comandos diferentes para a máquina, responl

(a) Supondo que, ao pressionarmos um botão, este volte à posição origiu d


podendo ser reutilizado em qualquer ação seguinte, qual o número d-
comandos que podem ser executados pela máquina?
(b) Supondo agora que, ao pressionarmos um botão, este não possa m d-
ser reutilizado em qualquer ação seguinte, qual o número de comando-
que podem ser executados pela máquina?
10 Problemas Resolvidos de Combinatória

50. Prove as seguintes identidades por argumentos combinatórios, isto é, sem


fazer cálculos explícitos, mostre que ambos os membros das igualdades ‘con­
tam’ o mesmo número de possibilidades de um dado evento:
(a) Zc(k) = para n, k G N tais que n > k > 1;
(b) (fc) (m) = (rn) (fc-rn)> Para n, fc, m G N tais que n > k > m > 0.
51. Sendo n > 2 um número natural, prove, por argumentos combinatórios,
que:
2n + 4 2n 2n „ (2n\ í 2n \ 2n
n -I- 2 n 4- 2
+4
n+1
+6(
\n/
) + 4l )
\n — 1/
+ n—2

52. (a) Encontre o número de maneiras de se acomodarem 12 pessoas tais que


7 delas fiquem numa mesa redonda e as 5 restantes em outra mesa
redonda.
(b) Encontre o número de maneiras de se acomodarem 12 pessoas tais que
7 delas fiquem numa mesa redonda e as 5 restantes fiquem num banco.
53. (a) Considere um Triângulo de Pascal3 na seguinte forma:
coluna 0

coluna 1
linha 0 -------- * ]
/ coluna 2
// coluna 3
linha 1-------- - 1 1
linha 2-------- - J 2 1 coluna 4
linha 3--------- - J 3 3 1 coluna 5
linha 4 - j 4 6 4 1
linha 5--------- * 1J 5 10 10 5 1

Figura 1: O elemento da linha n e coluna k é igual a ©•


Para calcular (£) por meio do Triângulo, não é necessário gerá-lo com­
pletamente até sua n-ésima linha, pois basta que disponhamos dos
elementos que estão situados numa cunha acima de (£), a saber, os dois
elementos cuja soma resultou (£), aqueles que originaram tais elemen­
tos, e, assim, sucessivamente. Isto ocorre devido ao método utilizado
na construção do Triângulo (item (b) do Exercício 60). Sabendo, pois,
3O Capítulo 3 de [13] traz diversas propriedades e características do Triângulo de Pascal.
Parte I. Problemas - Combinatória básica 11

que cada número do Triângulo é igual à soma dos dois situados ime­
diatamente acima dele (à exceção dos números iguais a 1), estime o
mínimo de adições necessárias ao cálculo de (^).
(b) Generalize o item anterior, encontrando o mínimo de adições necessá­
rias ao cálculo de (£).

54. (a) Quantas são as partições4 do conjunto {1,2, ...,10} em exatamente


duas partes?
(b) Generalize a resolução de (a) para o conjunto {1,2,..., n}, com n G N.
55. Considere um retângulo quadriculado 8x8.

(a) Tomemos duas moedas distintas que se podem mover apenas diagonal­
mente pelo retângulo. Assumamos que, dada uma certa posição inicial
dessas duas moedas em casas do retângulo, todas as posições que re-
sultarem de movimentos diagonais das moedas sejam equivalentes à
posição inicial. Por exemplo, na figura abaixo, as duas configurações
são equivalentes:

Figura 2: Retângulo 8x8 com duas moedas distintas, movendo-se para uma
posição equivalente à inicial.

Posto isso, e sabendo também que as duas moedas podem ocupar a


mesma casa, quantas configurações não equivalentes existem? E se as
duas moedas fossem iguais?
(b) Generalizando o conceito de configurações equivalentes para um núme­
ro maior de moedas, refaça o item (a) considerando agora cinco moedas
distintas e, em seguida, três moedas idênticas e duas distintas.
4Dado um conjunto A, uma partição de A é um conjunto de subconjuntos não vazios, dois a
dois disjuntos, cuja união é A.
12 Problemas Resolvidos de Combinatória

56. Dê uma interpretação combinatória para a seguinte identidade, provando-a


por argumentos combinatórios em seguida:

+ ...

57. Encontre o número de maneiras de se formarem seqüências de cinco letras


a partir das letras A, B, C, D, E e F se:

(a) repetições de letras são permitadas;


(b) repetições de letras não são permitidas;
(c) as seqüências contêm a letra C e repetições não são permitidas;
(d) as seqüências contêm a letra C e repetições são permitidas.

58. Um estacionamento dispõe de 20 vagas numeradas de 1 a 20. Desejam-se


colocar 20 carros em suas vagas, sendo 7 carros vermelhos, 5 carros brancos
e 8 carros pretos (considere os 20 carros distintos entre si).

(a) De quantas maneiras os carros podem ser colocados nas 20 vagas?


E se considerarmos que só importa a cor do carro que ocupa deter­
minada vaga, isto é, se considerarmos que carros de mesma cor são
indistinguíveis?
(b) De quantas maneiras podemos lotar o estacionamento de modo que
os carros vermelhos fiquem nas primeiras 7 vagas, os brancos nas
próximas 5 vagas e os pretos nas 8 vagas remanescentes? E se desejar­
mos apenas que carros de mesma cor fiquem em vagas consecutivas?

59. Considere que se tenham 10 pedras preciosas distintas.

(a) Dispõe-se de um quadro em cuja moldura existem 10 espaços para o


cravejamento das pedras. De quantas maneiras podemos fazê-lo?
(b) Dispõe-se de um espelho de formato circular em cuja moldura também
se encontram 10 espaços para o cravejamento das pedras. De quantas
maneiras podemos fazê-lo?
(c) De quantas maneiras podemos colocar essas 10 pedras num cordão
para montar um colar?

60. Prove as seguintes identidades através de argumentos combinatórioS:

(a) O = (n-J’ sendo n > Â: > 0;


Parte I. Problemas - Combinatória básica 13

(b) (Z) = (V) + (fc-J)’ sendo n > À: > L


(Esta é a chamada Relação de Stifel-Pascal, utilizada na construção
do Triângulo de Pascal (veja Exercício 53).)

61. Prove, por argumentos combinatórios, que:

sendo n > 0 um número inteiro.

62. Demonstre que:

(a) (p£2n) é múltiplo de p, sendo p, n e N;


(b) o produto de k inteiros consecutivos é divisível por kl
(Sugestão: considere as possíveis maneiras de selecionar k objetos den­
tre n 4- k.)

63. Prove a seguinte identidade através de argumentos combinatórios:

+ n3,

para n > 3.

64. (a) Encontre o coeficiente de afe4c2d5 na expansão multinomial da ex-


pressão (a 4- b 4- c 4- d)12.
(b) Encontre o coeficiente de a3ò4c2d3 na expansão multinomial da ex­
pressão (3a — b + c — 4d)12.
(c) Qual o número de termos distintos existentes na expansão multinomial
da expressão (a 4- b 4- c 4- d)12.

65. Encontre o número de r-seqüências (seqüências constituídas por r letras)


e de r-seleções (seleções de r letras) que podem ser realizadas a partir das
letras A, B, C, D, E, F, G e H se:

(a) r = 5 e os elementos são distintos;


(b) r = 5, permitindo-se repetições;
(c) r = 10, permitindo-se repetições.
66. De quantas maneiras um baralho comum de 52 cartas pode ser distribuído
entre quatro jogadores de modo que cada um receba exatamente 13 cartas,
sendo 4 de um dado naipe e 3 de cada um dos naipes restantes?
14 Problemas Resolvidos de Combinatória

67. São dados os pontos A, B, C, D e E sobre uma reta r e A, F, G, H, I e J


sobre uma reta s, distinta de r (as retas são concorrentes em A). Quantos
triângulos podem ser formados unindo-se tais pontos?
68. (a) Determine o número de divisores inteiros e positivos de 540 e de
283.500.
(b) Determine a soma dos divisores inteiros e positivos de cada um dos
números do item (a).
69. (a) Há 12 cadeiras em fila. De quantas maneiras 6 casais podem sentar-se
nas cadeiras de modo que marido e mulher sentem-se um ao lado do
outro?
(b) Agora, resolva novamente o item (a) supondo que o número de cadeiras
em fila foi aumentado para 17.
70. A figura abaixo mostra um mapa com 5 países:

POLÔNIA
ALEMANHA

RER
L TCHECA
Fsiov;
,AusnaA

(a) Dispondo de m cores diferentes (m > 5), de quantos modos esse mapa
pode ser completamente colorido de tal maneira que países fronteiriços
não sejam pintados da mesma cor?
(b) Satisfazendo as condições do item (a), qual o menor valor de m que
permite colorir o mapa? Quantas são as maneiras de colorir o mapa
nesse caso?

71. Considere a seguinte identidade:

(n)=è( n~p\ íp\


m — kj \ky
Parte I. Problemas - Combinatória básica 15

para n > p + m e p > m.


(Esta identidade é conhecida por Convolução de Vandermonde.)

(a) Demonstre-a igualando os coeficientes de xm em ambos os membros


de (1 + x)n = (1 + z)n-p(l 4- x)p.
(b) Demonstre-a por argumentos combinatórios.
72. (a) Considere um conjunto P de 25 pontos no espaço e Pi um subconjunto
de 12 pontos coplanares de P. Sabe-se que sempre que 4 pontos de
P são coplanares, então eles são pontos de P±. Quantos são os planos
que contêm pelo menos 3 pontos de P?
(b) Considere um conjunto Q de 22 pontos no plano e Qi um subconjunto
de 10 pontos colineares de Q. Sabe-se que sempre que 3 pontos de Q
são colineares, então eles são pontos de Qi. Desse modo, quantos são
os triângulos possíveis de se formarem com os pontos de Q?

73. Encontrar o número de triângulos que podem ser formados pelos vértices
de um polígono regular de n lados:

(a) não havendo nenhum tipo de restrições sobre os lados dos triângulos;
(b) se os lados do polígono não puderem ser lados dos triângulos.

74. Dentre os números 1, 2, ..100, de quantas maneiras podemos selecionar


dois inteiros distintos se:

(a) a diferença entre eles deve ser exatamente 9?


(b) a diferença entre eles deve ser menor do que ou igual a 9?

75. Dado um baralho completo de 52 cartas, de quantas formas se pode escolher


7 cartas de modo que entre elas haja pelo menos uma carta de cada naipe?

76. De quantas maneiras podemos distribuir 10 maçãs, 7 laranjas e 7 pêras em


4 caixas diferentes de modo que cada caixa receba pelo menos uma fruta
de cada tipo?

77. Quantos são os desarranjos das letras das seguintes palavras?


(Entenda-se por desarranjo das letras de uma palavra toda permutação das
mesmas na qual nenhuma letra ocupa sua posição original. Por exemplo,
OBOL é um desarranjo da palavra BOLO, mas LBOO não é, pois a última
letra O está numa posição que também é ocupada por O na palavra original.)

(a) RADAR;
16 Problemas Resolvidos de Combinatória

(b) AGRAVAM.
78. Uma roda-gigante possui 9 bancos de dois lugares cada um. De quantos
modos 16 crianças podem ocupar 8 de seus bancos, ficando duas em cada
banco?
79. Na figura que vem em seguida, os pontos A e C devem ser unidos por uma
escada, e o segmento BC é perpendicular ao segmento AB. A distância
entre A e B é igual a 6 metros e a distância entre B e C é igual a 2
metros. Sabendo que cada degrau deve ter 25 centímetros de altura e que
a largura de cada degrau deve ser um inteiro múltiplo de 40 centímetros,
de quantas maneiras a escada pode ser construída, considerando-se que o
primeiro degrau deva começar em A e que o último degrau deva terminar
em C?
C

A B

80. De quantas maneiras é possível selecionar de um total de 28 dominós, im­


portando a ordem de cada escolha, um par de dominós que tenham um lado
em comum (por exemplo, a escolha dos dominós 1-3 e 3-5 deve ser levada
em conta, mas não se deve contar a escolha de 2-2 e 4-6)? E se a ordem
das escolhas não importar?
81. Há um conjunto de 14 livros distintos alinhados numa prateleira de certa
livraria. Quantas são as possíveis seleções de 6 desses livros que não incluam
dois livros vizinhos? Em seguida, generalize este resultado para o caso de
desejarmos selecionar k livros dentre n alinhados numa prateleira, sendo
n > 2k — 1.
82. Cavaleiros da Távola Redonda-. Doze cavaleiros devem assentar-se à Távola
Redonda do Rei Artur. Apenas os dois vizinhos imediatos de cada um dos
12 cavaleiros são por ele considerados inimigos. Cinco cavaleiros devem ser
escolhidos para salvar uma princesa encantada. De quantas maneiras se
pode selecionar um grupo compatível de cavaleiros, isto é, um grupo que
não contenha cavaleiros inimigos? Em seguida, resolva o mesmo problema
supondo que k cavaleiros devem ser selecionados dentre n (n > 2Zc).
Parte I. Problemas - Combinatória básica 17

83. A fila do caixa: m-rk pessoas encontram-se na fila do cinema para comprar
seus ingressos, sendo que m delas possuem 10 reais e k possuem 5 reais. O
ingresso custa 5 reais, e o caixa não dispõe de nenhum dinheiro inicialmente.
Isso significa que algumas filas são “ruins” no sentido de que em alguns
pontos o caixa pode ficar sem troco, e outras são “boas”, no sentido de que
o caixa sempre disporá de troco para distribuir aos cinéfilos. Quantas são,
desse modo, as filas “boas” (suponha que m < A:)?

84. (a) Duas crianças coletaram 12 rosas, 13 margaridas e 10 lírios. De quan­


tas maneiras elas podem dividir tais flores entre si? E se impusermos
a condição de que cada criança fique com pelo menos três flores de
cada tipo?
(b) Duas crianças coletaram ni flores do tipo 1, rt2 do tipo 2, e, assim,
sucessivamente, até do tipo k. De quantas maneiras elas podem
dividir tais flores entre si? E se impusermos a condição de que cada
criança fique com pelo menos si flores do tipo 1, so do tipo 2, e, assim,
sucessivamente, até pelo menos flores do tipo k (sí < para
1 < i < k)7

85. n bandeiras distintas devem ser dispostas em k mastros distintos. Cada


mastro comporta um número qualquer de bandeiras, e a ordem das ban­
deiras em cada um deles é relevante. Sabendo que todas as bandeiras devem
ser utilizadas, mas que nem todos os mastros precisam ser utilizados, en­
contre o total possível de configurações.

8G. 5 rapazes e 5 moças devem posar para uma fotografia, ocupando õ degraus
de uma escadaria. Supondo que cada degrau deva conter exatamente duas
pessoas, responda:

(a) Do quantas maneiras isso pode ser feito sem nenhuma restrição?
(b) De quantas maneiras isso pode ser feito de modo que em cada degrau
fique um rapaz e uma moça?

S7. De quantas maneiras uni grupo <le 7 pessoas pode ser agraciado com quatro
prêmios diferentes: (todos os prêmios devem ser distribuídos)

(a) se nenhuma pessoa puder receber mais do que um prêmio?


(b) se cada pessoa puder receber qualquer número de prêmios (até quatro,
naturalmente)?
(c) se o vencedor do primeiro prêmio não puder receber outro prêmio, mas
vencedores de outros prêmios puderem receber mais de um prêmio?
18 Problemas Resolvidos de Combinatória

88. Tem-se um conjunto de m + n+p objetos, dos quais m são iguais entre si, n
são também iguais entre si (porém distintos dos anteriores), e os p restantes
são todos distintos entre si e dos anteriores. Calcule o número de coleções
não vazias que podemos formar com esses objetos.
89. No quadro abaixo, de quantas maneiras é possível formar a palavra COM­
BINATÓRIA partindo-se de um C e indo sempre para a direita ou para
baixo?
C
cO
COM
C OMB
C OM B I
C OMB I N
C O M B I NA
C OMB I NA T
COMBINATÓ
COMB INATÓR
COMBINATÓRI
COMBINATÓRIA

90. Encontre o número de zeros em que termina o número 873!.


91. (a) Quantos números podemos formar com os algarismos do sistema deci­
mal de modo que eles comecem por 3, terminem por 6, tenham o
algarismo 0 eqüidistante dos extremos e cada algarismo seja utilizado
no máximo uma vez?
(b) Determine o número de elementos do conjunto de todos os números
de 6 dígitos em que o algarismo das dezenas é 5 e o algarismo das
centenas é diferente do das unidades.
92. Estabeleça uma relação de ordem entre os números nn, (n2)! e (n!)2, para
n > 3.
93. Exprima os seguintes somatórios de maneira mais simplificada:

(a)
(Sugestão: considere a identidade i ■ í! = (i + 1)! — il.)
(b) E"=l jrfiy.
(Sugestão: encontre uma igualdade análoga à da sugestão do item an­
terior.)
Parte I. Problemas - Combinatória básica 19

94. (a) Calcule a soma de todos os números de 6 algarismos distintos formados


por 1, 2, 3, 4, 5 e 6.
(b) Escrevendo-se todos os números de 5 algarismos distintos em ordem
crescente, utilizando-se os algarismos 1, 2, 3, 4 e 5, qual será o lugar
ocupado pelo número 35.142?
(c) Escrevendo-se todas as possíveis “palavras” de 6 letras distintas utili­
zando-se a, b, c, d, e e f e, em seguida, dispondo-as em ordem al­
fabética, qual será a posição ocupada pela “palavra” cafdeb?

95. Considere 16 objetos distintos.

(a) Quantas são as maneiras de decompô-los em 4 grupos indistinguíveis


de 4 objetos cada um?
(b) Quantas são as maneiras de decompô-los em 6 grupos indistinguíveis,
sendo 3 com 2 objetos cada um, 2 com 3 objetos cada um e um com
4 objetos?

96. Com respeito à palavra IMPORTUNAS, determine:

(a) o número de anagramas possíveis;


(b) o número de anagramas que começam por IMP, nesta ordem;
(c) o número de anagramas que começam por IMP, numa ordem qualquer;
(d) o número de anagramas que têm juntas, nesta ordem, as letras IMP;
(e) o número de anagramas que começam por IMP, nesta ordem, e termi­
nam por UNAS, numa ordem qualquer.

97. De quantos modos 4 rapazes e 3 moças podem ocupar 11 lugares em fila, de


forma que as moças se sentem juntas umas das outras e os rapazes juntos
uns dos outros?

98. (a) De quantas maneiras 7 casais podem sentar-se em torno de uma mesa
circular contendo 14 lugares, de modo que não sentem juntos 2 homens
e que cada homem sente ao lado de sua esposa?
(b) De quantos modos 6 casais podem sentar-se em uma roda-gigante com
12 bancos de 2 lugares cada um, de modo a ficarem sempre juntos
marido e mulher?

99. Um homem possui 14 amigos, sendo 6 mulheres e 8 homens; sua esposa pos­
sui também 14 amigos, sendo 8 mulheres e 6 homens. De quantas maneiras
20 Problemas Resolvidos de Combinatória

o casal pode convidar um grupo de 14 pessoas, sendo 7 homens e 7 mulhe­


res, de modo que 7 delas sejam amigas do marido e 7 sejam amigas de sua
esposa?
100. (a) Prove, por argumentos combinatórios, que:

-O- (n — p + 1)
n 0, sendo n,p G N tais que n > p > 1.

(b) Valendo-se do resultado demonstrado no item anterior, simplifique o


somatório abaixo:
n—1

E (?)
101. (a) Formam-se as combinações de 8 elementos dos objetos ai, a2, ■ ■ ai2>
escrevendo-se os objetos, em cada uma delas, em ordem crescente de
índices. Em quantas dessas combinações a$ aparece e não ocupa o
terceiro lugar?
(b) Formam-se as combinações de p elementos dos objetos ai, 02, •••>
an (p < n), escrevendo-se os objetos, em cada uma delas, em ordem
crescente de índices. Em quantas dessas combinações ai aparece e não
ocupa o j-ésimo lugar (j < i < n)?
(c) Formam-se as combinações de p elementos dos objetos ai, (Z2, ...,
an (p < n), escrevendo-se os objetos, em cada uma delas, em ordem
crescente de índices. Quantas são as combinações em que o elemento
a, ocupa a j-ésima posição para os casos i > j, i = j e i < j?
102. (a) Dado um polígono convexo de n lados, qual o número máximo de
pontos dc intersecção de suas diagonais?
(b) Dados n pontos de um plano, 3 a 3 não colineares, qual o número
máximo de intersecções das retas formadas por esses pontos, excluindo-
se as intersecções ocorridas nos próprios pontos dados?
103. Têm-se 13 pontos cuja maioria pertence a uma reta r e os restantes se
situam sobre uma paralela a r, denominada s. Tendo esses pontos como
vértices, constróem-se todos os triângulos e quadriláteros possíveis. A razão
entre o número de quadriláteros, não necessariamente convexos, e o número
de triângulos é yj. Qual o número de pontos de cada uma das retas?
104. Suponha que os números inteiros de 1 a 9.999.999 tenham sido escritos
em colunas de tal forma que em cada coluna figurem todos os números
Parte I. Problemas - Combinatória básica 21

formados por um certo conjunto de 7 dígitos (observe que 1.457 pode ser
visto como 0.001.457). Por exemplo, 3.447.645 pertence à mesma coluna do
número 4.745.634, pois o multiconjunto de ambos os números é o mesmo,
mas não pertence à coluna de 5.744.624, pois seus dígitos são distintos dos
deste número. Quantas foram as colunas escritas?

105. (a) Quantas diagonais possui um polígono convexo de n lados?


(b) Quantas diagonais possui um prisma cuja base é um polígono convexo
de n lados? Nota-se alguma relação entre este valor e o encontrado no
item anterior? Justifique sua resposta.

106. Mostre que:

Wl [(:}•©] ■■[(.:>)•(:)]■
para n > 1.

107. Prove as seguintes propriedades do Triângulo de Pascal:


(a) Propriedade das linhas: “Dada uma linha n do Triângulo de Pascal,
vale o seguinte:

2n »

(b) Propriedade das colunas: “Dada uma coluna k do Triângulo de Pascal,


vale o seguinte:
ík\ Zâ:4-1\ /fc4-2\ k 4- r _ ík 4- r 4-1\ „
\k) \ k / \ k ) + ••• + k \ k 4-1 )

(c) Propriedade das diagonais: “Vale a seguinte identidade:


fn\ /n4-l\ (n + 2\ n+r n + r 4- 1 n

UM i M 2)+-+ r r

108. Prove, algebricamente, que:

a 4- b
i=0 x \ /
k

para a>keb>k — 1.
22 Problemas Resolvidos de Combinatória

109. Explicite as seguintes somas de maneira mais concisa:

(a) (»)+2(")+3Q)+.••+«(");
(b) (5)+3(í)+5®+- + (2n+l)C);
(c) 1.2®+2-3©+3-4(J)+... + (n-l).nC).

110. Com relação a um conjunto de n variáveis reais, responda:


(a) Quantos são os possíveis termos de um polinômio homogêneo com­
pleto5 de grau p que podem ser formados com tais variáveis (ignoran-
do-se as constantes multiplicai ivas)? A rigor: “Calcule a dimensão do
espaço dos polinômios homogêneos de n variáveis e graup.”
(b) Quantos são os possíveis termos de um polinômio completo de grau p
que podem ser formados com tais variáveis (ignorem-se as constantes
multiplicativas)? A rigor: “Calcule a dimensão do espaço dos polinô­
mios de n variáveis e grau p. ”

111. Num prédio de seis andares, há 10 pessoas no térreo, prontas para tomar
um elevador que ali se encontra. Admitindo que o elevador possua capaci­
dade para apenas 8 pessoas, de quantas maneiras podemos lotá-lo e nele
transportar as pessoas escolhidas, descarregando-as da seguinte maneira:
duas no primeiro andar, uma no segundo, nenhuma no terceiro, uma no
quarto, três no quinto e uma no sexto?
112. Considere uma série de 21 lançamentos de um dado comum de 6 faces. De
quantas maneiras distintas pode ocorrer que a face i apareça i vezes, para
i = 1,2,... ,6.

113. Uma sala de aula possui kr alunos. Uma professora tem elaborados k temas
distintos para trabalho, e deseja dividir a classe em k grupos de r estudantes
cada um. Determine o número de divisões possíveis se:

(a) cada grupo trabalhar com um tema diferente;


(b) todos os grupos trabalharem com um mesmo tema, escolhido dentre
os k disponíveis.
114. Paulo e Marta fazem parte de um grupo de oito pessoas. De quantos modos
podemos permutá-las:
5Dizemos que um polinômio é homogêneo completo se todos os seus termos possuem o mesmo
grau, a saber, o grau do polinômio homogêneo, e se todos os termos possíveis de ser formados
figuram no polinômio.
Parte I. Problemas - Combinatória básica 23

(a) em fila, de modo que, entre Paulo e Marta, haja 3 pessoas?


(b) em uma mesa redonda, de modo que, entre Paulo e Marta, haja
também 3 pessoas?

115. De quantas maneiras um conjunto de 8 meninos e 8 meninas pode ser


dividido em dois grupos contendo, cada um, um número ímpar de meninos e
um número ímpar de meninas? E se, além da condição anterior, impusermos
a condição de que os grupos resultantes devam conter o mesmo número de
integrantes?

116. Qual o número de maneiras de distribuirmos 15 cartas diferentes em 5 caixas


numeradas de modo que a j-ésima caixa receba j cartas (j = 1,2,3,4,5)?

117. Considere o seguinte procedimento de se particionar um conjunto de n ele­


mentos em n subconjuntos unitários. Inicialmente, o conjunto original é
particionado em dois subconjuntos quaisquer. Se um desses dois subconjun­
tos obtidos for unitário, então ele já é considerado componente da partição
desejada. De qualquer modo, cada um dos subconjuntos obtidos que pos­
suir dois ou mais elementos deve ser novamente particionado em dois outros
subconjuntos, repetindo o passo anterior, até que se obtenha a partição de­
sejada, após n — 1 realizações do procedimento supracitado. Mostre, então,
que o número de maneiras de se executar esta partição é igual a:

C)C”;■)-©©- n!(n — 1)!


2n-i

118. Prove as seguintes identidades tanto algebricamente como por argumentos


combinatórios:

(a) n(£) = (fc + IJCt+i) + fc(£), sendo n > k 4-1 > 1;


(b) © © = © (fc+PJ) (fc+s-j)> sendo m = min{*» s}es + k<n.

119. Prove algebricamente que:

(a) n©=+(fc+i)> send° n>k+i-

(b) ©© = xx0 (i) (í) ("■©> sendo m = min{fc, «}•


120. (a) De quantas maneiras podemos dispor 25 livros distintos em 5 pratelei­
ras de uma livraria, se cada prateleira suporta até 25 livros?
(b) Utilizando-se as 23 letras do nosso alfabeto, de quantas maneiras pode­
mos formar 5 palavras, se cada letra deve ser usada uma única vez?
24 Problemas Resolvidos de Combinatória

121. (a) Por argumentos combinatórios, mostre que:

m 4- n — 1\
n—1 /

C=i)C m + n — p — 1\
. n-p-1 ) +
m + p — 1\ ín — p — 1\
/ p \/m+n—p—2
n-p-1 + ...

+
. P~ 1 An-P-V’
sendo m, n,p G N tais que m + n> 1, n — p> 1.
(b) Através de propriedades dos números binomiais, realize alterações no
resultado do item (a) a fim de provar que:

Zm + 1\ _ fp\ fm — p' 'm — p—1


\n+lj \p/\n — p
+ . n-p
4....

+
P
vn-’\
'm — n + p
J\n — pJ

sendo, como em (a), m,n,p 6 N tais que m > n e n > p.

122. Encontre o número de trios de números naturais distintos, não maiores do


que 125, que formam uma progressão geométrica.

123. Cada um de n países envia 6 representantes a um congresso internacional.

(a) De quantas maneiras podemos pôr em fila os 6n participantes do con­


gresso, de modo que os 6 membros de cada país ocupem posições con­
secutivas?
(b) De quantas maneiras podemos pôr em fila os 6n representantes envia­
dos, de modo que dividamos cada um dos conjuntos de 6 compatriotas
em dois blocos de mesmo tamanho, os quais podem ocupar qualquer
posição na fila?

124. (a) Dados n objetos idênticos e n distintos entre si e distintos dos n


idênticos, de quantas maneiras podem ser selecionados n objetos?
(b) Dispondo de n objetos de cada um de 2 tipos distintos, quantas são
as possíveis escolhas de n dentre esses 2n objetos?
(c) Considerando todas as escolhas possíveis dos mesmos 2n objetos do
item anterior, quantas são as ordenações que com eles podem ser re­
alizadas?
Parte I. Problemas - Combinatória básica 25

125. Cada um dos números do conjunto {1,2,..., n2} é colocado em um dos n2


quadrados de um retângulo quadriculado n x n, de tal forma que os números
em cada coluna e em cada linha componham uma progressão aritmética.
De quantas maneiras isso pode ser feito?

126. Um cesto contém 2n + r maçãs diferentes e 2n — r pêras diferentes. Mostre


que, para um dado n, o número de maneiras de se escolherem n maçãs e n
pêras é máximo para r = 0.

127. Mostre que, ao se lançarem n dados idênticos simultaneamente, o número


de possíveis resultados das faces é igual a:

128. Prove que, para cada k G N, existe e é única a representação de um inteiro


não-negativo N na forma:

para 0 < x± ... < Xk-i < zjt, com a?i, ..., Xk-i, Xk inteiros não negativos.

129. Seja X um conjunto contendo n elementos e k G N um número fixo.


Quantas são as seqüências (Si, <$2, • • • ■> Sk) de subconjuntos de X tais que
Si ç S2 Ç ... Ç Sk?
26 Problemas Resolvidos de Combinatória

Princípio da Inclusão e Exclusão


130. Considerando todos os números naturais menores do que 1.000, responda:
(a) Em quantos deles o dígito 9 aparece? Em quantos o dígito 9 ocorre
pelo menos duas vezes?
(b) Idem ao item (a) para o dígito 0.
(c) Em quantos deles os dígitos 0 e 9 ocorrem concomitantemente? Refaça
este item considerando os dígitos 8 e 9 em lugar dos anteriores.
131. Considerando 6 lançamentos consecutivos de um dado comum (6 faces dis­
tintas) e o resultado obtido em cada um deles, responda:
(a) Qual o número de resultados possíveis?
(b) Calcule o número de maneiras que os lançamentos resultem em 6 faces
iguais, exatamente 2 distintas, exatamente 3 distintas, e assim por
diante, até o número de maneiras que resultem em 6 faces distintas.
Verifique, em seguida, se a soma das parcelas obtidas condiz com o
número encontrado no item (a).
132. 4 moças trabalham no serviço de atendimento ao consumidor de uma grande
empresa. Num certo intervalo de tempo, chegam-lhes 8 ligações. De quantas
maneiras as moças podem recebê-las de modo que nenhuma delas fique sem
ocupação?
133. Quantos são os anagramas da palavra INSANAS que não contêm duas letras
iguais juntas?
134. Quantas permutações de 6 letras podem ser realizadas a partir das letras
da expressão ‘O ATLETA LEO’ sem deixar que duas letras iguais fiquem
juntas (considere E=E)?
135. Considere a expressão ‘a carambola azeda’:
(a) De quantas maneiras podemos dividir suas letras entre 3 pessoas de
modo que ninguém fique sem nenhuma letra?
(b) De quantas maneiras podemos fazer a divisão de suas letras entre as
mesmas 3 pessoas de tal forma que cada uma fique com 5 letras?
136. A partir das 23 letras do nosso alfabeto, o número de permutações de qua­
tro letras que não permitem duas letras iguais juntas é 23 • 223 (há uma
maneira muito simples de se chegar a este resultado). Através do Princípio
Parte I. Problemas - Princípio da Inclusão e Exclusão 27

da Inclusão e Exclusão, conte o número de permutações de quatro letras


que possuem pelo menos duas letras iguais juntas e, com isso, forneça outra
resolução para o mesmo problema citado anteriormente.

137. Considere a seguinte identidade, para n G N:


n
9n

(a) Demonstre-a utilizando a fórmula do Binômio de Newton6.


(b) Demonstre-a através do Princípio da Inclusão e Exclusão.

138. Considere os conjuntos A e B, com |j4| = n e |B| = k.

(a) Qual o número total de funções irrestritas f : A —♦ B?


(b) Se n = k > 0, qual o número de funções bijetoras f : A B? Discuta
o caso n — k = 0.
(c) Se 0 < n < k, qual o número de funções injetoras f : A —* B?
(d) Se n > k > 0, qual o número de funções sobrejetoras f : A —> B?

139. Dispõe-se de 28 peças distintas de um jogo de dominó. De quantas maneiras


podemos dividir tais peças entre 4 pessoas de modo que:

(a) todas recebam o mesmo número de peças?


(b) ninguém fique sem nenhuma peça?

140. (a) De quantos modos 5 casais podem sentar-se ao redor de uma mesa
circular com número exato de cadeiras de tal forma que marido e
mulher não fiquem juntos?
(b) Generalize o problema para n casais.

141. “Chamamos de função </> de Euler à função que atribui a cada inteiro posi­
tivo m o número de inteiros positivos menores do que ou iguais ame rela­
tivamente primos com m. ” 7
cSendo a e b valores reais e n um número natural, a fórmula do Binômio de Newton é dada
por:
(a + fe)" = ]T
i=0

7Vide [13], Capítulo 4.


28 Problemas Resolvidos de Combinatória

Aplicando o Princípio da Inclusão e Exclusão, mostre que:


= m \1 — ) ( 1 — ] • • ■ ( 1-1)
\ PiJ \ P2/ \ Pr J
sendo pi, P2, ..., pr os divisores primos distintos do inteiro m (p^p^2 ' ‘ 'P?r
é a decomposição de m em fatores primos).
142. De quantas maneiras se podem comprar 15 latas de refrigerante se existem
4 tipos disponíveis e:
(a) há um número tão grande quanto se queira de latas de cada tipo?
(b) há somente 6 latas de cada tipo?
143. Numa classe de 40 crianças, 26 estudam português, 20 estudam italiano e 14
estudam francês. Se 10 crianças não estudam nenhuma dessas três línguas
e nenhuma estuda as três línguas, quantas crianças estudam português e
italiano, quantas estudam italiano e francês e quantas estudam português
e francês?
144. De quantas maneiras podemos ordenar as 9 letras de {a, a, b, b, b, c, c, d, d}
de modo que letras iguais nunca fiquem juntas?
145. Uma caravana composta por 10 camelos dispostos em fila viaja pelo deserto.
A viagem dura muitos dias e os “viajantes” consideram cansativo ter que
ver sempre o mesmo camelo à sua frente. De quantas maneiras os camelos
podem permutar entre si de modo que cada camelo seja precedido por um
diferente daquele que o precedia na fila anterior?
146. Quantos são os inteiros de 1 a 100.000 que são divisíveis por 2 ou por 3?
147. Quantos dos inteiros compreendidos entre 1 e 1.000.000 têm soma dos al­
garismos igual a 15?
148. Considere o conjunto de todas as permutações das letras de AAABBBCCC
para responder o que segue:

(a) Quantas dessas permutações não possuem duas letras A juntas?


(b) Quantas dessas permutações não possuem duas letras A juntas nem
duas letras B juntas?
(c) Quantas dessas permutações não possuem letras iguais juntas?
149. De quantos modos se podem sentar em fila 3 alemães, 3 espanhóis e 3
franceses, de modo que não fiquem dois compatriotas juntos?
Parte I. Problemas - Princípio da Inclusão e Exclusão 29

150. Numa caixa, há 3 tipos de objetos, cada um em quantidade de 2n, de modo


que, ao todo, são 6n objetos. De quantas maneiras podemos dividi-los entre
duas pessoas de modo que cada uma fique com 3n objetos?

151. (a) Dispondo de n objetos de cada um de três tipos distintos, encontre


o número de maneiras de se distribuírem esses 3n objetos dentre 3
pessoas, de modo que cada uma receba uma mesma quantidade de
objetos.
(b) De quantas maneiras podemos colocar 6 bolas brancas idênticas, 6
bolas pretas idênticas e 6 bolas vermelhas idênticas em três caixas
diferentes, se cada caixa deve conter 6 bolas?
30 Problemas Resolvidos de Combinatória

Funções geradoras e partições


152. (a) Qual é o coeficiente de xr em (1 4- x 4- x2 4- • ■ • )n?
(b) Qual é o coeficiente de xrm em (1 4- xm 4- x2m 4- • • • )n?
153. (a) Quantas são as soluções inteiras não negativas de xi +X2+X3+X4 =30
se cada variável deva estar compreendida entre 4 e 9?
(b) Qual a função geradora que controla o número de soluções inteiras não
negativas (a, b, c, d) da equação 2a 4- 3b 4- 5c 4- 3d = r?
154. (a) Prove, por argumentos combinatórios, que:

(l + zm)n = 1 +

sendo m,n E N.
(b) Considere as seguintes funções de x: f(x) = (1 4- x 4- x2 +---- |- xn)3 e
p(x) = (14-a:4-x24----- l-z71-1)3. Prove, por argumentos combinatórios,
que o coeficiente de x2n+1 em f(x) coincide com o coeficiente de x2n~2
em g(x). Encontre este coeficiente.
155. (a) De quantas maneiras podemos selecionar 275 balas de seis tipos dis­
tintos, se cada tipo se apresentar em pacotes de 25 balas cada um, e
devendo as seleções constarem de um a quatro pacotes de cada tipo?
(b) De quantas maneiras podemos dividir 13 livros idênticos de Matemáti­
ca, 10 idênticos de Português e 17 idênticos de História entre 2 alunos,
se cada um deve ficar com 20 livros, dos quais pelo menos 2 de cada
matéria?
156. Utilizando a identidade (1 — x2)n = (1 — z)n(l 4- a;)n, prove a fórmula
seguinte:

1=0 \ / \

sendo m par e não maior do que n.


157. (a) Utilizando funções geradoras, prove que todo inteiro positivo pode ser
expresso de maneira única como soma de potências distintas de dois.
(b) Mais geralmente, prove que todo inteiro positivo n possui representa­
ção única em cada base b G N, b > 2, isto é, existem e são únicos os
inteiros ai e {0,1,..., b — 1} tais que n = aob° 4- aibL 4- a2Ò2 4- • • •.
Parte I. Problemas - Funções geradoras e partições 31

158. Prove que o número de partições de um inteiro8 em partes distintas é igual


ao número de partições desse inteiro em partes ímpares.

159. Através de considerações em relação ao Diagrama de Ferrers de uma parti­


ção9, demonstre os seguintes resultados:

(a) O número de partições de n tendo k como a maior parte é igual ao


número de partições de n em exatamente k partes.
(b) O número de partições autoconjugadas10 de n é igual ao número de
partições de n em partes ímpares distintas.
(c) Para 1 < j < n, o número de partições de n nas quais j aparece como
parte é igual ao número de partições irrestritas de n — j.

160. Utilizando funções geradoras, demonstre que:

(a) o número de partições de n em exatamente duas partes é igual a |_^J;


(b) o número de partições de n em partes distintas, nenhuma das quais
múltipla de 3, é igual ao número de partições de n em partes da forma
6i + 1 ou 6z + 5.

161. Suponha que denote o número de partições de n em exatamente k


partes.

(a) Prove, por argumentos combinatórios, que:

q/M = Qfc-l(n - 1) + qk(™ ~ ty.

(b) Utilizando o resultado do item anterior, prove que:

162. (a) Mostre que o número de partições do inteiro n em três partes é igual
ao número de partições de 2n em três partes menores do que n.
8Denominamos partição de um inteiro n um conjunto de inteiros positivos cuja soma é igual
a n.
9Também chamado “Gráfico de uma partição”, consiste em se dispor pontos no plano,
colocando-se em cada linha, em ordem não crescente, um número de pontos igual a cada uma
das partes que compõem o número particionado em questão. Para maiores esclarecimentos sobre
o assunto, consulte o Capítulo 5 de [13].
10Dizemos que uma partição é autoconjugada se ela é igual à sua conjugada.
32 Problemas Resolvidos de Combinatória

(b) Mostre que o número de partições de n é igual ao número de partições


de 2n em n partes.
(c) Mostre que o número de partições de 2n+k em n+k partes independe
de k.
163. (a) Considerando-se as 23 letras do nosso alfabeto, encontre a função gera­
dora exponencial11 paira an, o número de palavras de n letras contendo,
no máximo, uma vogal de cada tipo.
(b) Em certo sistema criptográfico, as mensagens constam de 14 símbolos
distintos, distribuídos em 4 palavras consecutivas. Quantas são as
possíveis mensagens sujeitas à condição de que sua primeira palavra
contenha pelo menos dois símbolos (desconsidere a existência de pala­
vras “vazias”)?
(c) Com relação ao item anterior, qual seria a resposta caso não importasse
a ordem dos símbolos em cada palavra, mas apenas quais os símbolos
presentes em cada uma?
164. Resumo dos problemas sobre número de distribuições de objetos em caixas:
(a) De quantas maneiras podemos distribuir n objetos idênticos em k
caixas distintas? E se nenhuma caixa puder ficar vazia (n > Zc)?
(b) De quantas maneiras podemos distribuir n objetos distintos em k
caixas distintas se não importa a ordem dos objetos em cada caixa? E
se a ordem dos objetos em cada caixa for relevante? E se a ordem dos
objetos em cada caixa não importar, mas não puder haver nenhuma
caixa vazia (n > fc)?
(c) De quantas maneiras podemos distribuir n objetos distintos em k
caixas idênticas de modo que nenhuma caixa fique vazia (n > fc)?
E se puder haver caixas vazias?
(d) De quantas maneiras podemos distribuir n objetos idênticos em k
caixas idênticas? E se nehuma caixa puder ficar vazia (n > fc)?
165. No Exercício 157 (item (a)), provamos a identidade:
1 = (14- z)(l + z2)(l 4- z4) ■ • • (1 4- z2*) • • • •
1 4- z 4- ir2 4- z3 + • • ■ =
1—x
Utilizando-a, prove que, dentre todas as partições de um natural distinto
de um em potências de 2, há tantas contendo um número par de partes
quanto contendo um número ímpar de partes.
11 Para esclarecimentos, consulte o Capítulo 5 de [13].
Parte I. Problemas - Funções geradoras e partições 33

166. Mostre que o número de partições de r 4- k em k partes é igual ao número


de partições de r + (fcjx) em k partes distintas.

167. (a) Encontre a função geradora para an, o número de triângulos incongru­
entes de lados inteiros e perímetro n.
(b) Encontre a função geradora para òn, o número de partições de n em
três partes, nenhuma das quais maior do que a soma das outras duas.

168. No Exercício 164 (item (c)), vemos que o total de maneiras de se dis­
tribuírem n objetos distintos em k caixas idênticas sem que nenhuma fique
vazia é denotado por S(n, k), e denominado número de Stirling de segunda
espécie. Este número também pode ser interpretado como o número de
partições de um conjunto de cardinalidade n em exatamente k partes. As­
sim, é imediato que S(n, k) conta o total de partições de um conjunto
de n elementos, que é denotado por Bn, e chamado número de Bell.

(a) Prove, por argumentos combinatórios, que os números de Stirling de


segunda espécie satisfazem a relação:

S(n + !,&) = S(n, k — 1) + kS(n, k),

para n > k.
(b) Encontre uma forma sucinta da função geradora exponencial para
S(n, k), supondo k fixo, isto é, encontre uma simplificação para:
oo n

n=0

(c) Com base no item anterior, encontre uma expressão simplificada para
a função geradora exponencial de Bn.

169. (a) Utilizando funções geradoras, encontre o número de r-seqüências qua­


ternárias (seqüências de r dígitos formadas por elementos do conjunto
{0,1,2,3}) que contêm um número par de 0’s e um número ímpar de
l’s.
(b) Forneça uma solução por argumentos combinatórios para o item (a),
sem fazer uso de funções geradoras.

170. Quantas são as r-seqüências ternárias tais que:

(a) nenhum dígito ocorre exatamente duas vezes?


34 Problemas Resolvidos de Combinatória

(b) os dígitos 0 e 1 aparecem, cada um, um número par de vezes, e 2 não


aparece exatamente três vezes?
171. (a) Mostre que a função geradora que controla o número de maneiras de se
lançarem n dados idênticos e se obter soma par das faces é a seguinte:
1
1 _______ 1 1
2 (1 — x)3 (1 — x)3
+ (1 4- x)3

(b) Encontre a função geradora para o número de maneiras de se lançarem


n dados idênticos e se obter soma ímpar das faces.
(c) Sabe-se que 1,3,6,10,15,21,... formam a seqüência dos números tri­
angulares, cuja denominação decorre da representação de tais números
como pontos no plano. A relação de recorrência12 para esta seqüência
pode ser definida por tn = tn-i+n, com ti = 1. Sendo an o coeficiente
de xn na função geradora do item (a) e bn o de xn na função geradora
do item (b), prove que:

bn, se n é ímpar
an —
bn 4- tn+i, se n é par.

,2Se necessário, para revisar este tema, consulte o Capítulo 6 de [13].


Parte I. Problemas - Princípio da Casa dos Pombos 35

Princípio da Casa dos Pombos

172. Um ponto (x, y, z) do espaço é dito inteiro se todas as suas coordenadas


forem inteiras.

(a) Considere um conjunto de 9 pontos inteiros do espaço para mostrar


que pelo menos um dos segmentos de reta que os conectam possui
ponto médio inteiro.
(b) Dê um exemplo de um conjunto de 8 pontos inteiros do espaço que
não satisfaçam a propriedade dos 9 pontos do item (a).
(c) Os itens anteriores mostram-nos que nove é o menor número de pontos
inteiros em R3 que nos garantem a existência de pelo menos dois cujo
ponto médio do segmento de reta que os une seja inteiro. Generali­
zando, encontre o menor número de pontos inteiros em Rn que nos
garantem tal propriedade.

173. Dada a informação de que um ser humano possui, no máximo, 300.000 fios
de cabelo, e de que a cidade de São Paulo, por um censo recente, tem uma
população de 10.788.682 habitantes, encontre o maior número natural n
que torne a seguinte afirmação verdadeira: há n pessoas em São Paulo com
o mesmo número de fios de cabelo.

174. Considere uma propriedade P relacionada a objetos matemáticos.

(a) Para i = 1,2,3,4,5, ou o objeto ai possui a propriedade P, ou o


objeto bi possui esta propriedade. Assim, prove que pelo menos 3
dentre ai,..., a$ possuem a propriedade P, ou pelo menos 3 dentre
., &5 possuem esta propriedade.
(b) Para i = 1,2,..., 10, pelo menos um dentre a,, ò, e c, possui a pro­
priedade P. Assim, qual é o maior natural n que torna a seguinte
afirmação verdadeira: pelo menos n dentre ai,... ,aio possuem a pro­
priedade P, ou pelo menos n dentre &i,..., &io possuem a propriedade
P, ou pelo menos n dentre Ci,... ,Cio possuem a propriedade P?
(c) Para i = 1,2,3,4, 5, pelo menos dois dentre ai, bi e Ci possuem a pro­
priedade P. Assim, qual é o maior natural n que torna a seguinte
afirmação verdadeira: pelo menos n dentre a\,... ,a$ possuem a pro­
priedade P, ou pelo menos n dentre bi,..., 65 possuem a propriedade
P, ou pelo menos n dentre q, ... ,cg possuem a propriedade P?

175. Uma universidade contém 20 unidades de ensino. A fim de realizar uma


pesquisa acerca de hábitos alimentares, o restaurante universitário dessa
36 Problemas Resolvidos de Combinatória

universidade deseja selecionar 35 alunos pertencentes a uma mesma unida­


de de ensino. Dessa maneira, qual o número mínimo de alunos que devem
ser selecionados, ao acaso, no restaurante para que se chegue ao número
estipulado?
176. (a) Considere 6 pontos, 3 a 3 não colineares, no espaço. Cada um dos 15
segmentos formados pelos possíveis pares desses pontos é colorido com
uma dentre duas cores distintas, azul e vermelha. Prove que, qualquer
que seja a maneira de colorirmos os segmentos, sempre será formado
um triângulo cujos vértices são 3 dos 6 pontos e os lados possuem a
mesma cor.
(b) Considere 17 pontos, 3 a 3 não colineares, no espaço. Cada um dos
136 segmentos formados pelos possíveis pares desses pontos é colorido
com uma dentre 3 cores distintas: azul, vermelha e verde. Prove que,
qualquer que seja a maneira de colorirmos os segmentos, sempre será
formado um triângulo cujos vértices são 3 dos 17 pontos e os lados
possuem a mesma cor.
177. Considere um paralelepípedo de dimensões 3, 6 e 9 para mostrar que, dados
28 pontos quaisquer em seu interior ou em sua superfície, existem pelo
menos dois pontos que distam entre si, no máximo, i/14.
178. Mostre que toda seqüência de n2 + 1 inteiros distintos possui uma sub-
seqüência crescente de n 4- 1 termos ou uma subseqüência decrescente de
n 4-1 termos.
179. (a) Mostre que todo subconjunto de {1,2,..., 2n} contendo n 4-1 elemen­
tos possui pelo menos um par de elementos primos entre si e pelo
menos um par de elementos tais que um divide o outro.
(b) Mostre que 7 possui infinitos múltiplos da forma 434343... 43.
180. Utilizando o Princípio da Casa dos Pombos, mostre que, dados a G R e n
inteiro maior do que 1, existe um racional 1 < q < n, tal que:

a----P
Q
<±.
nq
Parte I. Problemas - Probabilidade 37

Probabilidade

181. (a) Um palhaço está na ponta do trampolim de uma piscina, carregando


uma caixa que contém n bolas vermelhas e n bolas azuis. Aleatori­
amente, ele vai retirando bolas da caixa, descartando-as em seguida.
Cada vez que seleciona uma bola azul, ele dá um passo para trás, e
cada vez que retira uma bola vermelha, ele dá um passo para frente.
Encontre a probabilidade de que o palhaço permaneça seco após a
retirada das 2n bolas da caixa.
(b) De quantas maneiras é possível organizar 2n pessoas de diferentes al­
turas em duas filas de n pessoas cada uma de modo que em cada
fila as pessoas estejam dispostas em ordem de altura e uma pessoa na
primeira fila seja sempre maior do que sua correspondente na segunda?

182. Problema de Galileu13: Suponha que 3 dados distinguíveis (um branco, um


preto e um azul) sejam lançados, sendo a o valor da face obtida para o
dado branco, b o valor obtido para o dado preto eco valor para o dado
azul. Além disso, seja A o subconjunto do total de lançamentos possíveis
para os quais se tem a + b + c = 9 e B o subconjunto para os quais se tem
a + b + c = 10. Calcule, então, |A| e |B|, a fim de concluir que p(A) < p(B).

183. (a) A partir dos elementos do conjunto A = {1,2,3,..., n}, obtemos uma
r-seqüência. Supondo que r < n, encontre a probabilidade de que cada
termo de A apareça no máximo uma vez em tal seqüência.
(b) Supondo que a probabilidade de que uma dada pessoa faça aniversário
num dia qualquer do ano seja (um ano possui 365 dias), qual o
menor valor de r que torna a seguinte afirmação verdadeira: a proba­
bilidade de que, num grupo de r pessoas, duas façam aniversário no
mesmo dia é maior do que ou igual a

184. Para a Copa do Mundo de Futebol, 24 países, dentre os quais Brasil, Ar­
gentina, Alemanha e Itália, são divididos em 6 grupos distintos, contendo
4 equipes cada um. Encontre a probabilidade de que ocorram os seguintes
2 eventos simultaneamente: Brasil e Argentina caiam num mesmo grupo
e Alemanha e Itália caiam também juntos num outro grupo. Encontre
também a probabilidade de que as quatro equipes diferenciadas caiam to­
das num mesmo grupo e compare este resultado com o anterior.
13
Galileu Galilei (1564-1642), cientista italiano.
38 Problemas Resolvidos de Combinatória

185. (a) Certa loteria possui 100.000 bilhetes diferentes (a saber, os que marcam
números compreendidos entre 00000 e 99999), dos quais somente um é
premiado em extrações semanais. Um jogador inveterado arrisca todas
as quatro semanas de um mês, comprando um bilhete em cada uma
delas. Seu filho, um jovem estudante, aconselha-o a comprar quatro
bilhetes somente uma semana por mês, pois isto ampliaria suas chances
de ganhar. Tem razão o filho?
(b) Considere que certa loteria possui N bilhetes diferentes, dos quais
somente um é premiado, em extrações periódicas. O jogador A compra
um bilhete em cada um de n períodos, e o jogador B compra n bilhetes
num único período. Assim, como generalização do item anterior, prove
que o jogador B é o que tem a maior chance de ganhar um prêmio
(2 < n < N)?
186. Escolhe-se, aleatoriamente, um elemento do conjunto {n G N|1 < n < 500}.
Qual a probabilidade de que este número não seja divisível por 2, nem por
3, nem por 5?
187. Uma caixa contém 2n bolas distintas, das quais a são vermelhas e b são
azuis, com a+b = 2n. Uma bola é retirada da caixa, ao acaso, e recolocada.
Em seguida, uma outra é retirada ao acaso. Calcule a probabilidade p de
que as duas bolas sejam da mesma cor. Mostre que tal probabilidade é
mínima para a = b = n. Sob tais condições, qual é o valor de p?
188. 6 dados de cores distintas são lançados simultaneamente. Calcule a pro­
babilidade de que:
(a) todas as faces contenham valores diferentes;
(b) todas as faces contenham o mesmo valor;
(c) 3 faces contenham um mesmo valor, sendo os valores das outras 3
distintos deste e distintos entre si;
(d) 3 faces contenham um mesmo valor, duas outras contenham um outro
valor, e a restante contenha um valor distinto dos 2 anteriores.
189. 6 dados idênticos são lançados simultaneamente. Calcule a probabilidade
de que:

(a) todas as faces contenham valores diferentes;


(b) todas as faces contenham o mesmo valor;
(c) 3 faces contenham um mesmo valor, sendo os valores deis outras 3
distintos deste e distintos entre si;
Parte I. Problemas - Probabilidade 39

(d) 3 faces contenham um mesmo valor, duas outras contenham um outro


valor, e a restante contenha um valor distinto dos 2 anteriores.

190. Consideremos uma urna contendo n bolas distintas, das quais ny > 1 são
brancas e ri2 > 1 são pretas, com n = rii + n.2. Toma-se, ao acaso, uma
amostra de r bolas, com r < e r < ri2- Qual a probabilidade de que
exatamente k das bolas selecionadas sejam brancas, se 0 < k < r.

191. Os algarismos 1, 2, 3, 4, 5, 6 e 7 são escritos em 7 cartões distintos, que


são colocados numa caixa. Em seguida, selecionam-se cartões dessa caixa,
e os números correspondentes às escolhas são escritos, sucessivamente, da
esquerda para a direita, até que se obtenha um número de 7 algarismos.
Qual a probabilidade de que o número obtido seja ímpar se:

(a) a escolha dos 7 cartões é realizada sem reposição?


(b) a escolha dos 7 cartões é realizada com reposição?
192. (a) Distribuem-se, aleatoriamente, n bolas idênticas em n caixas distintas.
Calcule a probabilidade de que exatamente uma caixa fique vazia.
(b) Distribuem-se, aleatoriamente, n bolas distintas em n caixas distintas.
Calcule a probabilidade de que exatamente uma caixa fique vazia.

193. Uma urna contém 6 bolas brancas, 6 bolas pretas e 6 bolas vermelhas.
Sacam-se 9 bolas desta urna, desconsiderando-se a ordem das bolas reti­
radas.

(a) Supondo-se que bolas de mesma cor sejam indistinguíveis, qual a pro­
babilidade de se extraírem 3 bolas de cada cor se não houver reposição?
(b) Supondo-se que bolas de mesma cor sejam indistinguíveis, qual a pro­
babilidade de se extraírem 3 bolas de cada cor se houver reposição?
(c) Supondo-se que as 18 bolas sejam distintas entre si, assemelhando-se
somente em relação às cores, encontre as probabilidades, sem e com
reposição, de se extraírem 3 bolas de cada cor.

194. No jogo da SENA, são sorteadas 6 dezenas distintas dentre 01, 02, ...,
48. Um apostador escolhe 6 dessas 48 dezenas, sendo premiado se forem
sorteadas 4 (quadra), 5 (quina) ou 6 (sena) das dezenas por ele escolhidas.

(a) Qual a probabilidade de um apostador fazer uma quadra?


(b) Qual a probabilidade de um apostador fazer uma quina?
(c) Qual a probabilidade de um apostador fazer a sena?
40 Problemas Resolvidos de Combinatória

195. No jogo da QUINA, são sorteadas 5 dezenas dentre 01, 02, ..., 80. Um
apostador escolhe 8 dezenas, sendo premiado se 3 (terno), 4 (quadra) ou
5 (quina) das dezenas sorteadas encontrarem-se entre as 8 originalmente
escolhidas. Determine a probabilidade de que um apostador faça:

(a) um terno;
(b) uma quadra;
(c) a quina.

196. Num estacionamento há 14 vagas em fila, das quais 10 estão ocupadas por
carros distintos e 4 estão desocupadas.

(a) Qual é a probabilidade de não haver vagas vazias consecutivas?


(b) Qual é a probabilidade de que haja exatamente 3 vagas vazias conse­
cutivas?

197. (a) Uma caixa contém 2n livros, sendo n idênticos de Português e n


idênticos de Matemática. Em um grupo de 2n pessoas, a (a < n)
preferem Português e b (6 < n) preferem Matemática, sendo que
as pessoas restantes não têm preferência por nenhuma das matérias.
Distribuindo-se os livros ao acaso, um por pessoa, qual a probabilidade
de que as preferências sejam respeitadas?
(b) Em um armário, há n pares distintos de sapatos. Retiram-se ao acaso
p pés de sapatos deste armário, incluindo pés esquerdo e direito de
quaisquer dos n pares (p < 2n). Qual a probabilidade de haver exata­
mente k pares de sapatos entre os pés retirados (pés esquerdo e direito
pertencentes a pares distintos não devem ser considerados como par
de sapatos)?

198. Considere um polígono regular de 2n + l lados inscrito numa circunferência.


Escolhendo-se, aleatoriamente, 3 dos seus 2n + 1 vértices, um triângulo
é formado. Qual a probabilidade de que o centro da circunferência seja
interior ao triângulo?

199. Um torneio de tênis reúne 2n jogadores igualmente hábeis (isso significa


que, numa dada partida entre dois jogadores, a probabilidade de cada um
deles vencer é 5). Inicialmente, eles são divididos em y = 2n-1 duplas
ao acaso. Os jogadores de cada dupla, então, jogam entre si, de modo que
o vencedor de cada jogo passa à próxima etapa, enquanto o perdedor é
eliminado do torneio. Todos os vencedores de cada etapa são, em seguida,
Parte I. Problemas - Probabilidade 41

reagrupados aleatoriamente em duplas, e o processo anterior se repete con­


tinuamente, até que restem apenas dois jogadores na última etapa, da qual
sairão campeão e vice-campeão do torneio.
(a) Supondo que, após a semifinal do torneio, haja disputa de terceiro
lugar, qual a probabilidade p(A:) de que um jogador determinado A
jogue exatamente k partidas no torneio, sendo 1 < k < n?
(b) Supondo que não haja disputa de terceiro lugar, qual a probabili­
dade de dois jogadores determinados A e B se enfrentarem durante o
torneio?
200. Suponha um torneio de tênis como o do Exercício 199, porém com jogadores
de diferentes habilidades. Isto significa que dois jogadores são sempre com­
paráveis, no sentido de que um dado jogador é sempre melhor ou pior do que
outro. Suponha, ainda, que não haja surpresas nos resultados das partidas
disputadas, ou seja, um jogador sempre vence a partida se for melhor do
que seu adversário e, reciprocamente, perde se for pior do que ele. Calcule,
então, a probabilidade de que o segundo melhor jogador do torneio seja
vice-campeão.

201. Suponha que um ano possua 365 dias para encontrar o menor valor de n
que satisfaz a seguinte afirmação: a probabilidade de que pelo menos uma
dentre n pessoas aniversarie hoje é maior do que
202. Um relé é um dispositivo magnético que controla a passagem da corrente
elétrica entre dois terminais: quando ele se fecha, passa corrente elétrica
entre os terminais; caso contrário, não há passagem de corrente. Na figura
abaixo, cada relé é representado por um número de 1 a 6:

1
A B

Supondo que a probabilidade de fechamento de cada relé seja igual a p


(0 < p < 1), calcule a probabilidade de que haja corrente elétrica entre os
42 Problemas Resolvidos de Combinatória

terminais A e B. Com o auxílio de um software matemático, estime p a fim


de que a probabilidade de haver corrente entre A e B seja maior do que
203. Dispõe-se de um molho de n chaves distintas para abrir determinada porta.
Somente uma das chaves do molho abre a porta.
(a) Supondo que, a cada tentativa malsucedida, descarte-se a chave usada
e que a escolha de cada chave seja aleatória, qual a probabilidade de
se abrir a porta na fc-ésima tentativa?
(b) Supondo que as chaves não sejam descartadas e que a escolha de cada
uma seja sempre aleatória, qual a probabilidade de se abrir a porta na
Zc-ésima tentativa?
204. (a) No estacionamento de um supermercado, há n 4- 1 vagas em fila.
Destas, n estão ocupadas, incluindo-se as das extremidades, e apenas
uma está desocupada. Um consumidor chega e estaciona seu carro
nesta vaga. Após voltar com as compras, ele constata que apenas
m 4-1 (m < n) carros estão estacionados, incluindo-se o seu. Qual a
probabilidade de que as duas vagas adjacentes à de seu carro estejam
vazias?
(b) Se n crianças, dentre as quais Maria e João, são postas em fila, qual a
probabilidade de haver exatamente m (m < n) crianças entre as duas
cujos nomes foram citados?
205. Sacam-se, com reposição, n bolas de uma urna que contém nove bolas
numeradas de 1 a 9.
(a) Supondo que importe a ordem de retirada das bolas, calcule a pro­
babilidade de que o produto dos valores marcados nas n bolas retiradas
seja divisível por 6.
(b) Supondo que a ordem de retirada das bolas seja irrelevante, isto é,
que só importem quais as n bolas sacadas da urna, calcule a proba­
bilidade de que o produto dos valores marcados nas n bolas retiradas
seja divisível por 6.
206. Um certo jogo lotérico consiste na escolha de 15 números dentre 1, 2, ...,
25. São sorteados exatamente 15 destes 25 números, e um apostador é
premiado se acertar de 11 a 15 dos números sorteados, sendo o prêmio,
logicamente, proporcional ao número de acertos.
(a) Qual a probabilidade de que o apostador ganhe o prêmio máximo, isto
é, acerte os 15 números sorteados em sua aposta?
Parte I. Problemas - Probabilidade 43

(b) Qual a probabilidade do apostador não ganhar nenhum prêmio?

207. Um certo matemático14 sempre leva consigo duas caixas de fósforos, uma
em cada bolso. Toda vez que ele deseja fumar, toma um palito de uma das
duas caixas, a qual é escolhida aleatoriamente. Inicialmente, cada caixa
contém exatamente n palitos, os quais vão sendo, sucessivamente, consumi­
dos. Como o matemático é distraído e fuma muito, há um momento em que
ele tira uma das caixas de um dos bolsos e percebe-a vazia. Neste momento,
qual a probabilidade de que a outra caixa contenha exatos k palitos?

208. Suponha que p(z) se refira à probabilidade de, no lançamento de um dado,


ocorrer a z-ésima face. Para um dado especial, tem-se que p(l) = p(3) = |,
p(2) = p(5) = tí e p(4) = p(6) = |. Encontre a probabilidade de que, num
único lançamento desse dado, resulte:

(a) um número par;


(b) um número menor do que ou igual a 3;
(c) um número distinto de 4.

209. Numa eleição dividida em 2 turnos, 5 candidatos (C*i, C2, C3, C4 e C5)
disputam a Prefeitura de uma cidade. Pesquisas realizadas por fontes con­
fiáveis revelam que a probabilidade de Ci ganhar o primeiro turno da eleição
é igual à probabilidade de C2 ganhá-lo. A probabilidade de C3 ganhar é um
terço desta probabilidade, enquanto a probabilidade de C4 ganhar é igual
à de C5 ganhar, correspondendo ao dobro da probabilidade de C3 ganhar.

(a) Calcule as probabilidades de cada um dos candidatos ganhar o primeiro


turno das eleições.
(b) Supondo que apenas Ci e C4 tenham passado ao segundo turno da
eleição, mantendo-se, proporcionalmente, suas chances de ganhar, cal­
cule as probabilidades de cada um deles ganhar a eleição.

14Segundo Feller, W. ([5]), o matemático polonês H. Steinhaus, tendo convivido com o também
polonês S. Banach, elaborou uma versão parecida deste bem-humorado problema, aludindo aos
hábitos de fumante de Banach.
Parte II

Resoluções
Parte II. Resoluções - Combinatória básica 47

Combinatória básica
1. (a) Há duas opções: ir de A até C passando por B ou ir de A até C
diretamente. Passando por B, há 5 possíveis rotas de A a B e, para
cada uma destas, 3 possíveis rotas ligando B eC. Assim, pelo Princípio
Multiplicativo, existem 5 • 3 = 15 possíveis rotas. Diretamente, por
sua vez, há 2 possíveis caminhos ligando A e C. Então, pelo Princípio
Aditivo, temos 15 4- 2 = 17 rotas possíveis ligando A e C.
(b) Como existem 17 caminhos para ir de A até C, devem existir, também,
17 caminhos para ir de C até A, pois cada rota pode ser percorrida
em ambos os sentidos. Logo, pelo Princípio Multiplicativo, existem
17 ■ 17 = 289 rotas diferentes.
(c) Do total de 289 rotas de (b), apenas 4 não passam pela cidade B
nenhuma vez, pois há 2 opções para ir de A a C, sem passar por B, e
2 para voltai' de C até A, sem passar por B. Assim, há 289 — 4 = 285
rotas que passam por B, ao menos uma vez.
(d) Faremos uma análise caso a caso:
i. Rotas que vão de A a C e voltam a A, sem passar por B;
Neste caso, há duas possibilidades para ir de A a C e somente
uma para voltar de C até A, já que não se pode passar duas vezes
pela mesma estrada. Logo, há duas possíveis rotas.
ii. Rotas que vão de A a C, passando por B, e voltam a A, sem
passar por B;
Para ir de A até (7, são 15 possíveis rotas (item (a)) e, para voltar
a A, são 2 possibilidades. Há, então, 15 • 2 = 30 possíveis rotas
nessas condições.
iii. Rotas que vão de A a C, sem passar por B, e voltam a A passando
por B;
Caso simétrico ao anterior, tendo também 30 possibilidades como
resposta.
iv. Rotas que vão de A a C e voltam a A, passando por B na ida e
na volta.
Como já foi visto na resolução do item (a), existem 15 caminhos
ligando A e C. Para voltar a A passando por B, há duas possibi­
lidades até B e 4 possibilidades de B até A (não podemos passar
pelas mesmas estradas utilizadas no caminho de ida até A). Assim,
há 2-4 = 8 possíveis rotas para voltar até A. Logo, são 15-8 = 120
as possíveis rotas.
48 Problemas Resolvidos de Combinatória

Pela análise dos itens i. a iv., dentre as rotas enumeradas em (b), temos
que 2+30+30+120 = 182 não passam duas vezes pela mesma estrada.

2. (a) Como há 4 cartas valetes no baralho (uma de cada naipe), este é o


número de escolhas para a primeira carta. Já para a segunda, não se
podem escolher nem as 4 damas, nem o valete escolhido como primeira
carta. Restam, portanto, 52 —4— 1 = 47 escolhas para a segunda carta.
Assim, temos 4 • 47 = 188 possíveis escolhas das duas cartas.
(b) Neste item, é necessário fazer uma divisão em dois casos:
i. Se a primeira carta for o rei de copas, então restam 52 — 4 = 48
opções para a segunda carta e, conseqüentemente, para o número
de escolhas, nesse caso;
ii. Se a primeira carta não for o rei de copas, temos 12 possíveis
cartas para a primeira escolha. Com relação à segunda carta, não
se podem utilizar os 4 reis do baralho, tampouco a primeira carta
escolhida. Deste modo, restam-lhe 52 — 4 — 1 = 47 possibilidades.
Logo, temos 12 • 47 = 564 possibilidades.
Combinando i. e ii., há 48 + 564 = 612 maneiras de se escolherem as
cartas.

3. (a) Como a primeira e a última posições devem ser preenchidas com vo­
gais, há 5 opções para cada uma delas. Para as posições centrais
(segunda e terceira), existem 23 opções para cada uma, já que se pode
preenchê-las com qualquer letra do alfabeto. Portanto, o número total
de anagramas é 5 • 23 • 23 • 5 = 52 • 232.
(b) A resolução deste item emprega raciocínio análogo ao do item anterior,
exceto que as posições centrais só permitem o uso de consoantes. Logo,
são 18 opções para a segunda posição e 18 para a terceira. Assim, o
número de anagramas passa a ser 52 ■ 182.

4. Para escolher uma maçã ou uma banana, Severino tem 16+13 = 29 opções.
Já para escolher uma maçã e uma banana, ele tem 16 opções de maçã e 13
de banana. Então, são 16 ■ 13 = 208 maneiras distintas de se realizar tal
escolha.

5. Representemos os possíveis números da seguinte maneira, na qual cada


posição se refere a um dígito:

ls. 2^- 3^ 4^ 5—6—


Parte II. Resoluções - Combinatória básica 49

Como só se vai até 100.000, a 1- posição só pode ser ocupada por 0 ou


1. Como o número 100.000 não contém o dígito 5, tal posição deve ser
ocupada por 0, de sorte que o problema se reduz a contar o número de
dígitos 5 que aparecem nos números compreendidos entre 1 e 99.999. Para
tanto, façamos uma divisão em casos:
(a) Números em que o dígito 5 aparece uma única vez;
Há 5 posições para serem preenchidas com 5 (da 2- a 6a-). Uma vez
preenchida uma posição, as restantes podem ser preenchidas por qual­
quer dígito diferente de 5. Então, são 9 opções para cada uma dessas 4
posições, donde há 5 • 94 números nos quais o dígito 5 aparece apenas
uma vez. Observe que o fato de se admitirem 9 opções para as ‘casas’
restantes permite a formação de números que variam de 1 a 5 algaris­
mos, já que 0 se inclui nessas 9 opções. Por exemplo, no resultado
5 - 94, está incluído o número 520 (= 00520). Tal observação vale,
ainda, para os próximos casos.
(b) Números em que o dígito 5 aparece exatamente duas vezes;
São duas ‘casas’ a serem preenchidas com o dígito 5. Como há 5
opções, temos (2) = 10 possibilidades de se escolherem as posições que
serão ocupadas pelo dígito 5. Para as 3 posições restantes, sobram,
novamente, 9 opções. Há, então, 10 • 93 números que satisfazem (b).
(c) Números em que 0 dígito 5 aparece exatamente 3 vezes;
Com raciocínio análogo ao do caso anterior, há (3) = 10 maneiras de
se escolherem as três ‘casas’ que serão ocupadas pelo dígito 5. Para as
duas ‘casas’ restantes, há 9 opções. Então, neste caso, existem 10 • 92
números.
(d) Números em que 0 dígito 5 aparece exatamente 4 vezes;
Analogamente, há Q) • 9 = 5 • 9 números.
(e) Números em que o dígito 5 aparece exatamente 5 vezes.
Como só há 5 posições, todas devem ser preenchidas com o dígito 5, o
que resulta em somente uma possibilidade.
Agora, é preciso descobrir quantas vezes o dígito 5 aparece nos números con­
tados. Considerando-se a divisão em casos efetuada, basta multiplicar cada
número encontrado pelo número de algarismos 5 que aparecem em cada
caso. Em seguida, basta somar tais valores. Assim, a resposta é igual a:

(5 • 94) ■ 1 + (10 • 93) • 2 + (10 • 92) • 3 + (5 • 9) • 4 + (1) ■ 5 = 50.000.


Uma resolução alternativa compreende contar em quantos dos números en­
tre 1 e 99.999 o dígito 5 aparece em cada casa. Assim, o dígito 5 aparece
50 Problemas Resolvidos de Combinatória

104 vezes em cada uma das 5 casas. De fato, fixando-se 5 em qualquer casa,
há 10 possíveis preenchimentos para as outras 4 casas. Logo, a resposta ao
problema será 5 • 104 = 50.000, como já sabíamos.
6. Para se compor uma coleção, pode-se tomar de 0 a n A’s, de 0 a n B's, de
0 a n C’s e de 0 a n D's. Logo, há n 4-1 opções para cada letra. Então,
pode-se formar (n 4- l)4 coleções distintas com as 4 letras dadas. Porém,
como se pedem apenas as coleções não vazias, deve-se retirar o caso em que
isso ocorre, isto é, o caso em que não se escolhe nenhuma letra. Assim, a
resposta é (n 4-1)4 — 1.
7. Pode-se visualizar tais produtos na forma 3a • 4b • 5C • 6d • 7e, com 0 < a, d < 1,
0 < 5, c<2e0<e<3. Assim, por exemplo, 3-5-6 seria o caso em que
a — c = d — 1 e ò = e = 0. Assim, temos 2 possibilidades para a e d (0
ou 1), 3 para b e c (0, 1 ou 2) e 4 para e (0, 1, 2 ou 3), donde o número
de possibilidades se torna 22 • 32 • 4 = 144. No entanto, este número inclui
o caso em que a = ò = c = d = e = 0, que não deve ser considerado, e os
seguintes casos também inconvenientes, por haver necessidade de se ter um
produto com pelo menos dois fatores:

(a) a=leb = c = d = e = 0 (1 caso)


(b) &=lea=c=d=e=0 (1 caso)
(c) c=lea = 5 = d = e = 0 (1 caso)
(d) d=lea=&=c=e=0 (1 caso)
(e) e=lea=5=c=d=0 (1 caso)
Então, retirando-se os casos inconvenientes, obtemos como resposta a quan­
tia de 144 — 1 — 5 = 138 produtos possíveis.
8. E fundamental notar, antes da resolução do exercício, que cada um dos
resultados obtidos pela soma de dois ou mais números do conjunto dado
ocorre de maneira única. Assim, por exemplo, a única maneira de se obter
34 com os números fornecidos é fazer 1 4- 3 4- 10 4- 20 = 34. Para se chegar
a tal conclusão, basta observar que, na expansão do produto:

(1 4- rr)(l 4- z3)(l 4- x5)(l 4- x 10)(l+r'20)(l 4-x:50


1 )(14-X:!90),

não aparecem potências de x com coeficiente maior do que 1. Para me­


lhor compreender esta conclusão, concernente à área de Funções Gerado­
ras, afirmamos que, na expansão do produto acima, o coeficiente de x'n
se refere ao número de maneiras de se escrever o inteiro positivo m como
Parte II. Resoluções - Combinai ária básica 51

soma dos números propostos no enunciado15. Portanto, é válido o método


resolutivo que propomos a seguir. Escrevamos cada soma em questão como
la+3ò+5c+10d-l-20e+50/-l-9Ck7, para a qual a, ò, c, d, e, f e g assumem os
valores 0 ou 1. Exemplificando, o caso c — d=lea = b = e = f = g = 0
resultaria na soma 5 + 10 = 15. Então, há 27 possíveis somas. Nesse
número, porém, está incluído o caso em que todas as letras valem 0 e os
casos em que seis delas valem 0 e somente uma vale 1 (7 possibilidades).
Assim, existem 27 — 1 — 7 = 120 somas possíveis com os números fornecidos.

9. Para um dos grupos, pode-se escolher os seis jogadores de (g2) = 924


maneiras diferentes. Com isso, o outro grupo já fica determinado. No
entanto, essas 924 maneiras ordenam os grupos de mesma cardinalidade.
Como só queremos efetuar a divisão dos 12 jogadores em dois grupos com
seis cada, temos = 462 divisões possíveis.
Para que os cinco rapazes não pertençam ao mesmo grupo, deve-se retirar
de 462 o número de vezes em que isso ocorre. Se os cinco rapazes estiverem
no mesmo grupo, o sexto componente do grupo será uma moça (7 possi­
bilidades). Então, temos 7 possíveis divisões dos jogadores nas quais todos
os rapazes estão num mesmo grupo. Portanto, a resposta do problema é
462 — 7 = 455 maneiras.

10. (a) Observe o seguinte tabuleiro de xadrez, que exemplifica a notação que
será empregada nesta resolução:

Figura 3: Neste tabuleiro, temos um retângulo 3x5 em destaque.

Agora, procederemos à contagem dos retângulos, dando antes uma pe­


quena demonstração acerca do método que será empregado em nossos
cálculos. Como exemplo, suponhamos apenas a contagem de retângu­
los do tipo 3x5 (como o da figura acima). E claro que temos 6 opções
possíveis para a escolha do lado que vale 3 e apenas 4 seleções para
15Consulte a referência [13] para mais esclarecimentos a esse respeito.
52 Problemas Resolvidos de Combinatória

o lado que vale 5 (verifique-o simplesmente observando o tabuleiro


representado). Assim, o número de retângulos 3 x 5 é 6 ■ 4 = 24.
Prosseguindo com este raciocínio, obtemos:
Tipo Quantidade Tipo Quantidade
1 x 1 8 • 8 = 64 2x1 7 - 8 = 56
1 x 2 8 • 7 = 56 2x2 7 • 7 = 49
1 x 3 8 • 6 = 48 2x3 7 • 6 = 42

1 x 8 8-1 = 8 2x8 7-1 = 7


Desse modo, o número total de retângulos será facilmente calculado
como a soma das quantidades dispostas em cada setor da tabela acima.
Observe o que segue:
• número de retângulos do tipo 1 x j (1 < j < 8)16:
8 + 2- 8 + -- - + 8- 8= [8+(88^8 = 288;
• número de retângulos do tipo 2 x j (1 < j < 8):
7 + 2- 7 + -- - + 8- 7= = 252;
• número de retângulos do tipo 3 X J (1 < J < 8):
6 + 2- 6 + -- - + 8- 6= = 216.

Observe que o número total de retângulos diminui 36 unidades a cada


tipo. Assim, o número total de retângulos que podem ser formados é
288 + 256 + 216 + 180 + • • • + 288 - 36(8 - 1) = <288+36)8 = 1.296.
- ,36,

(b) Para este item, abordaremos uma solução mais simples do que a do
item anterior. Sabemos que um retângulo é definido a partir do mo­
mento em que se determinam seus quatro lados. As linhas do tabuleiro
m x n que contêm os quatro vértices podem ser escolhidas de (m^1)
maneiras, enquanto as colunas de m x n que os contêm podem ser es­
colhidas de ("2!) maneiras. Uma vez escolhidas, observe que se define
um único retângulo. Assim, (:m+1)(n+1) = corresponde
à resposta procurada. Aplicando esta fórmula para m = n = 8, obte­
mos, também, o valor 1.296, coincidindo com a resposta de (a).
1GDada uma Progressão Aritmética de n termos, sendo ai seu primeiro termo e an seu
n-ésinio termo, a soma de seus n termos iniciais Sn é obtida por meio da fórmula:

(qi + an)n
Sn =
2
Parte II. Resoluções - Combinatória básica 53

11. Maria pode ficar com quaisquer 3 dos 9 livros de Alberto, ou seja, ela tem
(3) = 84 opções. Analogamente, Alberto tem (3) = 35 opções. Assim, há
84 • 35 = 2.940 possíveis trocas de 3 livros entre Maria e Alberto.

12. Para uma dada pulseira, há (8 — 1)! = 7! = 5.040 possibilidades (usando o


conceito de permutações circulares17). No entanto, este número conta duas
vezes cada pulseira, pois cada uma tem sua simétrica obtida através de
um giro em torno de um eixo diametral. Observe a ilustração abaixo, que
exemplifica duas pulseiras idênticas que estariam sendo contadas duas vezes
na quantia de 5.040 (as letras de A a H representam as contas distintas):

Assim, são = 2.520 pulseiras distintas.

13. Num tabuleiro de damas, há 64 casas, sendo 32 pretas e 32 brancas. Pode­


mos selecionar as doze casas pretas a serem ocupadas pelas peças pretas
de (i|) maneiras distintas. Então, restam 20 casas pretas, das quais 12
deverão ser ocupadas pelas peças brancas de (j°) maneiras. Logo, temos
= toi3ioi ei = 28.443.124.054.800 maneiras, um número considera-
velmente grande!

14. Tomando-se apenas as letras distintas de I, temos o conjunto de letras


NDVSBLDADE. São 10 letras, com repetição de 3 letras D. Logo, temos
3T = 604.800 permutações possíveis. Como não se pode ter letras I vizi­
nhas, basta que se as coloquem entre as letras preexistentes ou numa das
duas extremidades. Por exemplo, considerando a disposição original sem as
letras I, podemos colocar um I entre S e B, um entre as letras D e A, um
depois do E, e assim por diante, obtendo a seguinte possível permutação:
INIDVSIBILDIADEI. Assim, como são 11 os ‘espaços’ a serem ocupados,
tem-se (g1) = 462 maneiras de se colocarem as letras I nos mesmos. Con­
siderando as permutações que já foram possíveis de se realizarem, temos
604.800 • 462 = 279.417.600 possibilidades sob as condições do enunciado.
17Com relação a este tema, consulte o Capítulo 3 de [13].
54 Problemas Resolvidos de Combinatória

15. (a) Dentre as 12 pessoas, devem ser escolhidas as 6 que receberão, cada
uma, um livro. Isso pode ser feito de (g2) = 924 maneiras. Agora,
pode-se arranjar os livros entre essas 6 pessoas, considerando que há
repetições, de 3TT = 60 maneiras. Então, há 924 • 60 = 55.440 possi­
bilidades distintas de se fazer a distribuição.
(b) Além dos casos contados em (a), denotando-se os livros pelas letras A
(3 cópias), B (2 cópias) e C (1 cópia), temos os casos correspondentes
aos itens abaixo. Os hífens separam os livros que serão recebidos por
cada pessoa e observe que, além de selecionarmos dentre as 12 pessoas
aquelas que receberão um ou mais livros, temos ainda que permutar
as pessoas que vão receber os livros determinados em cada caso:
i. AB-A-A-B-C]
(?) ■ § = 47.520.
Exemplificando: nesse caso, uma pessoa recebe uma cópia do livro
A e outra do livro B, duas pessoas recebem uma cópia do livro
A cada uma, outra pessoa recebe uma cópia do livro B e outra
recebe uma cópia do livro C. Então, (g2) servirá para escolher as
cinco pessoas contempladas e || decidirá qual pessoa ficará com
cada bloco de livros (observe que há duas cópias de A repetidas).
ii. AC-A-A-B-B-,
(12) . 5! = 23.760.
iii. BC-A-A-A-B;
(?) ' 31 = 15.840.
iv. AB-AB-A-C-,
(\2) • || = 5.940.
v. AB-AC-A-B-,
(\2) • 4! = 11.880.
vi. AB-BC-A-A;
(12) • || = 5.940.
vii. AB-AB-AC;
(3) ’ S = 660-
viii. ABC-A-A-B-,
(4) • U = 5.940.
ix. ABC-AB-A.
ft2) -3! = 1.320.
Agora, somando-se os resultados encontrados nos itens i. a ix. ao do
item (a), obtemos 174.240 como resposta. Sem utilizarmos aquele
Parte II. Resoluções - Combinatória básica 55

resultado, há uma resolução mais sucinta para o problema: basta se­


lecionarmos as 3 pessoas que receberão, cada uma, um livro A, de
(g2) = 220 maneiras, as 2 que receberão, cada uma, um livro B, de
(^) = 66 maneiras, e a que receberá o livro C, de 12 maneiras. Assim,
teremos como resposta, novamente, 220 • 66 • 12 = 174.240.
16. Para o primeiro evento, têm-se (g°) = 210 opções diferentes de escolha.
Para o segundo, haverá 209 opções e, para o terceiro, 208 (não pode haver
repetição do grupo escolhido). Assim, temos 210 ■ 209 • 208 = 9.129.120
maneiras distintas.
17. (a) Na expressão, há 7 letras a, 5 letras r, 3 letras o, 2 letras n, 2 letras h,
1 letra u e 1 letra j. Então, pode-se escolher de 0 a 7 letras a, de 0 a 5
letras r e assim por diante. Logo, temos 8-6-4-3-3-2-2 = 6.912 seleções
diferentes. Descontando o caso em que não escolhemos nenhuma letra,
temos 6.911 seleções distintas a fazer.
(b) Aqui, torna-se conveniente dividir o problema em casos:
i. seleções de três letras distintas;
Basta selecionar 3 letras dentre as 7 existentes, o que resulta em
Q) = 35 seleções distintas.
ii. alguma letra apresenta exatamente duas cópias na seleção (a, r,
o, n ou h);
Como se mostra acima, são cinco as letras que apresentam pelo
menos duas cópias. Uma vez selecionadas, resta escolher a outra
letra, dentre as seis restantes. Logo, são 5 - 6 = 30 possibilidades.
iii. selecionam-se três cópias de uma única letra.
Como se vê, isso só é possível para as letras a, r e o (3 opções).
Concluindo, o número de seleções das letras é igual a 35 + 30 4- 3 = 68.
(c) Aqui se evidencia a vantagem da resolução empregada no item (b). Isso
porque, para três letras distintas, multiplicamos o número de escolhas
por 3!, para exatamente duas, multiplicamos por 3 (a letra distinta
pode ocupar uma das 3 posições) e, para 3 letras iguais, nada há que
se fazer. Em seguida, somamos estes números, obtendo um total de
35 ■ 3! + 30 • 3 + 3 = 303 permutações.
18. Dispomos de 18 letras, das quais 9 são letras a e o restante é composto de
letras, duas a duas, distintas. Para proceder à resolução, vamos dividir o
problema em casos. No que segue, considere a seguinte notação: número
de a’s que ficam em determinada caixa-número de a’s que ficam em outra
caixa-número de a’s que ficam em outra caixa, distinta das anteriores:
56 Problemas Resolvidos de Combinatória

(a) 0-4-5;
© ■ © ' (!) • 3! = 1.512.
Observe que, como o número de letras a em cada caixa é bem deter­
minado, basta que contemos o número de escolhas possíveis entre as 9
letras restantes, que são distintas duas a duas. Ao final, multiplicamos
por 3!, considerando que A, B e C são caixas distintas.
(b) 0-3-6;
© ■ © ■ © ■ 3! = 504.
(c) 1-2-5;
©©•©•31 = 756.
(d) 1-3-5;
©■©•d)'3! = 3.024.
(e) 1-4-4;
© ■ © ' © ' 3 = 2.268.
(f) 2-2-5;
©•©.(!). 3 = 1.890.
(g) 2-3-4;
©•©©■31 = 7.560.
(h) 3-3-3.
© • © • © = 1-512.
Somando os valores encontrados nos itens anteriores, obtemos 19.026 possí­
veis alocações das 18 letras nas caixas. Como uma dessas alocações é a
originalmente estabelecida, e estamos interessados apenas no número de
trocas possíveis da letras, temos 19.025 como resposta do problema.
19. A palavra tem 10 letras e, portanto, podemos imaginar 10 posições a serem
preenchidas por 5 consoantes (das quais duas são letras C repetidas), já que,
uma vez preenchidas, as posições das vogais ficam determinadas. Logo,
são (g0) = 252 possibilidades de escolha dos espaços para as consoantes.
Em seguida, devemos permutar tais letras, com repetição, o que nos dá
252 • |j = 15.120 possibilidades.
20. Devemos primeiramente selecionar 2 livros dentre 10. Em seguida, 2 livros
dentre os 8 restantes e, assim, sucessivamente. Ainda, como as parcelas
de dois livros são indistintas (as caixas são idênticas), devemos dividir o
resultado final pela permutação do número de parcelas existentes, obtendo
um total de disposições possíveis igual a:
(© ■ © • © ■ © • © = 945
Parte II. Resoluções - Combinatória básica 57

21. Temos 9 posições a serem preenchidas. Como dispomos de 5 consoantes,


devemos escolher os espaços que elas ocuparão, o que resulta num total de
(;?) = 126 possibilidades. Agora, devemos permutar as consoantes, obtendo
126 • = 7.560 possibilidades. Desse número, porém, devemos retirar o
número de vezes em que o dígrafo RR aparece. Tal contagem pode ser
feita fixando-se RR como um único bloco dentro da palavra, contando com
8 espaços a serem preenchidos por 7 letras mais um dígrafo. Novamente,
basta que escolhamos as posições que serão ocupadas pelas consoantes S,
L, H e pelo bloco RR, permutando em seguida. Assim, (®) • 4! = 1.680 é
o número de vezes em que duas letras R aparecem juntas e, portanto, a
resposta do nosso problema é 7.560 — 1.680 = 5.880.

22. (a) Não podemos escolher quatro letras distintas, visto que só se dispõe
de três letras distintas (P, O e R). Logo, temos os seguintes casos:
i. duas letras iguais e duas outras distintas entre si;
Temos apenas que escolher as duas letras iguais, pois, como só há
três letras distintas, as outras duas já estarão determinadas. São,
portanto, 3 opções.
ii. duas letras iguais e outras duas iguais entre si.
Podemos imaginar que vamos deixar de escolher exatamente um
tipo de letra, obtendo, desse modo, 3 opções também.
Assim, são 6 as seleções possíveis de se fazerem.
(b) Cada um dos casos do item i. de (a) nos fornece 12 permutações e cada
um dos casos do item ii. de (a) nos fornece 6. Logo, temos um total
de 3 • 12 + 3 • 6 = 54 permutações possíveis.

23. (a) Trata-se de uma palavra de 11 letras, das quais 5 são vogais e 6 são
consoantes. Para que seja possível obter a configuração proposta no
enunciado da questão, devemos invariavelmente preencher a primeira
posição com uma consoante. Sendo assim, os espaços a serem ocupa­
dos por vogais ou consoantes já ficam determinados, bastando que per-
mutemos vogais e consoantes em Suas posições, aplicando, em seguida,
o Princípio Fundamental da Contagem: ? = 900 (lembre-se
de que há repetições). Portanto, 900 são as maneiras possíveis de se
realizarem as permutações propostas.
(b) Vamos dividir o problema em casos para calcular, antes, o número de
seleções de quatro letras que podem ser feitas:
i. quatro letras distintas entre si;
Dispomos de duas letras l, duas letras i, duas letras ò, duas letras
58 Problemas Resolvidos de Combinatória

c e três letras o. Assim, para selecionar uma letra de cada tipo,


temos um total de escolhas igual a (&) = 5.
ii. duas letras iguais e outras duas distintas entre si e distintas das
duas iguais;
Neste caso, temos 5 opções de escolha da letra que tem ocorrência
dupla e, conseqüentemente, (2) = 6 escolhas das outras duas le­
tras. Logo, são 5 ■ 6 = 30 as escolhas possíveis.
iii. dois pares distintos de letras iguais;
Basta que escolhamos os dois tipos de letras diferentes, resultando
em (2) = 10 escolhas possíveis.
iv. três letras ‘o’ e outra letra distinta.
E fácil ver que são apenas 4 as opções de escolha, pois basta que
escolhamos a letra distinta de o.
Agora, uma vez dispondo do número de seleções que são possíveis de
se fazerem em cada caso, podemos multiplicar tais números separada­
mente pela quantia de permutações que contemplam as condições do
problema e, em seguida, somar as parcelas obtidas. O único caso que
apresentará alguma dificuldade para a contagem é o caso iv., pois é 0
único que apresenta três letras 0 em suas seleções. Assim, uma per­
mutação típica, nesse caso, seria oobo. Observe, então, que, para cada
escolha, temos apenas duas permutações que não dispõem três letras
o juntas (além do já citado, oboo, no caso exemplar). Logo, a resposta
do nosso problema é:
5 • 4! 4-30 • ^ + 10 • 4!
+ 4 • 2 = 548.
2! -2!
24. Para que a soma dos 3 números resulte par, temos duas opções de escolha:
ou escolhemos 3 números pares, ou escolhemos um número par e 2 números
ímpares. Observando que, dentre 30 números consecutivos, sempre temos
15 números pares e 15 números ímpares, podemos escolher 3 números pares
de (g5) maneiras distintas. Por outro lado, podemos escolher 2 números
ímpares de maneiras distintas e um número par de 15 maneiras dis­
tintas. Assim, como tratamos de eventos independentes, devemos somar os
resultados obtidos nos 2 casos, isto é, devemos fazer (g5) 4- (^) • 15 = 2.030,
obtendo a resposta almejada.
25. (a) Consideremos um conjunto de p barras verticais e de n — 1 sinais de 4-.
Desse modo, cada solução da equação fornecida pode ser associada
a uma interposição dos n — 1 sinais de 4- entre os p — 1 espaços de­
terminados pelas p barras. De fato, nesse caso, o número de barras
Parte II. Resoluções - Combinatória básica 59

compreendidas do início até o primeiro sinal de 4- corresponde ao valor


de a?i; o número de barras compreendidas entre o primeiro e segundo
sinais de 4- corresponde a x?, e, assim, sucessivamente. Ora, o número
de interposições dos n — 1 sinais de 4- entre os p — 1 espaços existentes
é que, portanto, corresponde à resposta do problema.
(b) Supondo agora que Xi > 0, isto é, que 0 também seja considerado como
possível valor de cada x», não há mais a necessidade de se ter p > n.
Nesse caso, temos Xi 4- 1 > 1, donde, atribuindo j/, = x, + 1, isto é,
Xi = yi — 1, temos a nova equação:

(1/1 — 1) 4- (3/2 — 1) H------- F (yn - 1) = P => y\ + 2/2 +----- \-yn=p + n,

para yi > 1. Agora, o número de soluções desta nova equação, que


é idêntico ao da equação original, pode ser calculado utilizando-se o
item (a). Logo, a resposta do problema é = (n+p-1)-
(c) Supondo-se que se dispõe de n tipos de objetos e de pelo menos p
cópias de cada um, a resposta do item (a) conta o número de maneiras
de selecionarmos p objetos de tal maneira que seja tomada pelo menos
uma cópia de cada tipo. O item (b), por sua vez, conta o mesmo
número de seleções, mas não impõe a condição de que seja tomada
pelo menos uma cópia de cada tipo de objeto.
Por haver mais de uma cópia de cada objeto, as seleções ora discutidas
recebem o nome de Combinações com repetições.

26. O que importa na seleção que devemos efetuar é o número de moedas de


cada um dos três tipos que for escolhido. Podemos, então, pensar na
equação Xi 4- X2 + X3 = 20 para representar o problema. Nela, xi, X2
e X3 representam o número respectivo de moedas de cada um dos três
tipos distintos que forem escolhidas. Atribuindo o valor 20 à soma, esta­
mos impondo que o número total de moedas escolhidas seja sempre igual
a este valor. Assim, a resposta ao nosso problema é igual ao número de
soluções inteiras não negativas da equação acima, o que sabemos ser igual
a (3+20-1) = (20) = 231, pelo Exercício 25.

27. (a) Procederemos primeiro com a justificativa de que cada soma só pode
ocorrer de uma única maneira. Note que os números das faces formam
uma seqüência crescente de números, na qual cada elemento é uma
unidade maior do que o dobro do elemento anterior. Com isso, é
possível concluir que cada soma é única. Quanto ao número de somas
possíveis no lançamento dos dois dados, uma vez sabendo que cada
60 Problemas Resolvidos de Combinatória

soma só tem uma possibilidade de ocorrência, podemos selecionar duas


faces distintas e, então, somá-las ou somar duas faces idênticas. No
primeiro caso, são (®) = 15 somas possíveis e, no segundo, são exa­
tamente 6 as somas possíveis. Logo, a resposta é igual a 15 + 6 = 21.
Note que, se as somas não fossem únicas, teríamos de adotar outra
estratégia de resolução.
(b) Analogamente, observe que as faces agora compõem uma seqüência
crescente na qual cada elemento é uma unidade maior do que o triplo
do anterior. Novamente, isso nos garante que cada soma é única e
que, portanto, ao efetuarmos a contagem padrão na resolução, não
estaremos contando casos repetidos.
Como são agora 3 lançamentos, temos 3 opções a fazer. Para maior
clareza, julgamos por bem enumerá-las:
i. o lançamento resulta em 3 faces distintas;
Neste caso, há (®) = 20 somas possíveis de ocorrer.
ii. o lançamento resulta em duas faces idênticas e uma diferente;
Agora, são 6 • 5 = 30 as somas possíveis. Explicando: 6 escolhe o
valor das faces idênticas e 5 escolhe a face diferente.
iii. o lançamento resulta em 3 faces idênticas.
Como são 6 as faces, temos também 6 posibilidades neste caso.
Portanto, o total de somas obtidas no lançamento dos três dados é
20 + 30 + 6 = 56.
28. (a) Cada tipo de carta apresenta 4 naipes à disposição para escolha. São
treze os tipos e, portanto, 413 = 67.108.864 as escolhas possíveis.
(b) Para cada naipe, dispomos de 13 tipos de cartas distintas. Logo, são
134 = 28.561 as seleções possíveis, neste caso.
(c) Agora, para a primeira carta, temos ainda 13 opções. Para a segunda
carta, porém, temos apenas 12, já que não podemos selecionar uma
carta do mesmo tipo da primeira. Assim, sucessivamente, chegamos a
um total de 13 • 12 • 11 • 10 = 17.160 maneiras.
(d) Para a carta de ouros, temos 13 opções de escolha. Uma vez escolhida
tal carta, a carta de copas (também vermelha) já está determinada,
pois deve ser formado um par vermelho. O mesmo raciocínio se em­
prega na formação do par preto, donde vem que são 13 • 13 = 169 as
possíveis seleções nesse caso.
29. (a) Podemos distribuir um naipe por vez (ouros, espadas, copas e paus),
observando que cada naipe contém 13 cartas. Assim, tem-se 13 cartas
Parte II. Resoluções - Combinatória básica 61

distintas a serem distribuídas entre 13 jogadores. O número de manei­


ras de se fazê-lo é simplesmente 13!. Logo, para cada naipe, têm-se
13! maneiras de se distribuírem suas respectivas cartas, donde (13!)4
é o número de maneiras de se distribuírem as cartas, valor da ordem
de 1039.
(b) São 13 opções para escolhermos quem receberá uma carta de cada
naipe. Uma vez escolhido tal jogador, temos 13 opções para sua carta
de paus, 13 para a de copas, 13 para a de ouros e 13 para a de espadas.
Logo, são 13 • 13 ■ 13 • 13 • 13 = 135 opções para o jogador diferenciado.
Uma vez distribuídas as cartas a este jogador, sobram 48 cartas, sendo
12 de cada naipe, para serem distribuídas aos restantes 12 jogadores.
Tomando um naipe de cada vez, como cada jogador tem de receber
4 cartas de um mesmo naipe, podemos dividir as 12 cartas em três
grupos de 4 cartas cada um da seguinte maneira:
/12\ /8\ /4\ 12! ■ 8! • 4! 12!
\4/ ’ \4j ' vJ “ 8! ■ 4! - 4! - 4! - 4! - 0! “ (4!)3'

Observe que não há necessidade de se dividir o resultado por 3!, visto


que os jogadores são distintos, isto é, não queremos apenas separar as
cartas, mas dividi-las entre pessoas. Então, para cada naipe, temos
esse número de possibilidades de separar suas cartas em 3 grupos de 4
cada. Agora, precisamos também escolher quais serão os 3 jogadores
que receberão cada um dos grupos de cartas, observando que, para
cada grupo de 3 jogadores que for selecionado para receber um certo
naipe, temos opções de distribuição deste naipe. Logo, podemos
fazê-lo de:

© W © (4!p
12! 12!
©■«.■©
12!
' (4!)3 “

© © ©©
(12!)4
“ (4!)12 ‘

(12!)4 12! 9! 6!
“ (4!)12 •1
9! ■ 3! ’ 6! • 3! ‘ 3! ■ 3!
(12!) 5
= (4!)12 ■ (3!)4

maneiras. Agora, anexando o resultado do jogador diferenciado, obte­


mos:
(12!)5 (13 • 12!)5 (13!)5
• 135 =
(4!)12 • (3!)4 (4!)12 • (3!)4 (4!)12 • (3!)4’
62 Problemas Resolvidos de Combinatória

que é a resposta procurada, da ordem de 1029.


30. (a) Antes de mais nada, temos que escolher quais serão as quatro pessoas
que receberão 4 objetos cada. Isso pode ser feito de (^) = 5 maneiras
distintas. Uma vez escolhidas tais pessoas, podemos distribuir-lhes os
objetos de:
/18\ /14\ /10\ /6\ 18!
U/U/U; vj— (4!)4• 2!
maneiras. Note que não dividimos o valor inicial por 4!, pois, apesar
da cardinalidade dos blocos de objetos ser igual, as pessoas que os rece­
berão devem ser consideradas distintas. Anexando o último resultado
ao número de seleções daqueles que receberão 4 objetos, temos como
resposta 5 • = 48.243.195.000 maneiras distintas.
(b) Temos (|) opções de escolha das pessoas que vão receber 4 objetos
cada. Agindo assim, o número de objetos de cada uma das outras duas
já estará determinado. Logo, como devemos distribuir três pacotes de
4 objetos e dois pacotes de 3 objetos, a resposta ao nosso problema
torna-se igual a:

0©©C”)-0 0-
5! 18! • 14! • 10! • 6! • 3!
“ 3! ■ 2! ' 14! • 4! • 10! • 4! • 6! • 4! • 3! • 3! - 0! • 3!
10 • 18!
= 128.648.520.000.
- (4!)3 • (3!)2

31. Com relação a cada um dos 14 tipos de objetos, pode-se proceder de 3


maneiras distintas: não escolher esse tipo de objeto, tomar apenas um
objeto desse tipo e tomar dois objetos desse tipo. Logo, são 314 seleções
possíveis. Descontando, porém, o caso em que nenhum objeto é selecionado,
temos um total de 314 - 1 = 4.782.968 seleções.
32. Por contarmos com nove objetos de cada tipo, temos 10 opções de escolha
para cada um dos objetos. De fato, podemos não selecioná-lo ou selecionar
qualquer quantia de objetos do tal tipo, compreendida entre 1 e 9. Assim,
uma vez que os tipos de objetos são em número de 20, obtemos um total de
102° — 1 = 99.999.999.999.999.999.999 seleções distintas não vazias a re­
alizar, e está mostrado o que pede o enunciado.
33. (a) Um tabuleiro de damas tem 32 casas pretas e 32 casas brancas. Pode­
mos selecionar primeiro um quadrado preto de 32 maneiras diferentes
Parte II. Resoluções - Combinatória básica 63

e, em seguida, um quadrado branco de 32 maneiras distintas. Logo,


são 322 = 1.024 as maneiras de se fazer a tal seleção.
(b) Agora, para a primeira seleção, supondo que seja a de um quadrado
preto, temos também 32 opções. Note, então, que cada vertical ou
horizontal do tabuleiro conta com 4 quadrados de cada cor. Logo, são 8
os quadrados brancos que não podemos escolher e, assim, reduzimo-nos
a 24 opções de seleção para o quadrado branco, donde são 32-24 = 768
as escolhas possíveis nesse caso.

34. (a) Os números menores do que um milhão são aqueles que contêm um
número de dígitos menor do que ou igual a seis. Com seis dígitos,
temos duas opções para o preenchimento de cada casa decimal (8 ou
9). Logo, são 26 números posíveis de serem formados. Com cinco
dígitos, conseqüentemente, são 25 opções. Prosseguindo o raciocínio,
temos 26 + 25 + 24 + 23 + 22 + 2 = = 126 números possíveis18.
(b) Agora, utilizando os dígitos 7, 8 e 9, podemos simplesmente repetir o
raciocínio do item (a) lembrando que, para cada casa a ser preenchida,
temos agora uma opção a mais (o algarismo 7). Desta forma, temos
36 + 35 + 34 + 33 + 32 + 3 = 1.092 números possíveis de serem formados
neste caso.
(c) Aqui, o fato de podermos preencher casas com o dígito 0 apresenta um
complicador ao nosso problema. De fato, para um número de 6 dígitos,
por exemplo, não podemos preencher sua primeira casa com 0, pois isto
nos daria um número de 5 dígitos. Logo, nesse caso, teremos apenas
duas opções para a primeira casa (8 e 9), sendo que para as 5 casas
restantes há 3 opções, pois nelas o dígito 0 pode ser empregado. Assim,
são 2 • 35 os números de 6 dígitos possíveis. Repetindo o raciocínio
para números de 5 dígitos, temos 2 ■ 34 números possíveis. Logo, são
2 -354-2 -34+ 2-33+ 2-32+ 2- 34-2 = 728 os números menores do que
um milhão e que contêm apenas os dígitos 0, 8 e 9 em sua composição.

35. Vamos considerar os números de 6 dígitos ou menos. Quanto aos que têm
6 dígitos, temos 9 opções para preencher sua primeira casa (não se pode
utilizar 0), 9 para a segunda (não se pode usar o dígito utilizado na casa
18Lembre-se de que, dada uma Progressão Geométrica de primeiro termo ai e razão q, a soma
Sn de seus n primeiros termos é igual a:
ai(gn - 1)
Sn =
4-1
64 Problemas Resolvidos de Combinatória

anterior), e assim por diante, chegando a 96 possibilidades. Para números de


5 dígitos, chegamos a 95 possibilidades. Repetindo a estratégia empregada,
temos 96 4- 95 4- 94 + 93 + 92 + 9 = 597.870 números nas condições do
problema.
36. (a) Os números que podemos formar possuem todos 4 dígitos. Observe
que, na primeira posição, o algarismo 1 aparece 3! vezes (basta que
permutemos os demais algarismos nas outras posições), o mesmo acon­
tecendo com os algarismos 2, 3 e 4. O mesmo raciocínio se aplica às
demais posições. Note, porém, que, caso um certo algarismo apareça
na primeira posição (a dos milhares), teremos uma contribuição de
1.000 vezes o valor do algarismo para a soma. Repita-se o mesmo ar­
gumento para as demais posições. Assim, a soma dos algarismos de
cada casa é 3! • 1 + 3! • 2 + 3! • 3 + 3! • 4 = 66. Portanto, considerando
as contribuições de cada casa para a soma final, temos um total de
66 ■ 1.000 + 66 • 100 4- 66 • 10 4- 66 • 1 = 66.660 como resposta.
(b) Tomando cuidado para o fato de aqui haver repetições dos algarismos
disponíveis, partamos direto aos cálculos, que seguirão a mesma sis­
temática do item anterior. A soma dos algarismos que aparecem em
cada casa é igual a • 1 4- • 4 = 15. Considerando, pois, as con­
tribuições de cada casa para a soma final, obtemos, como resposta,
15 • 1.000 4-15 • 100 4-15 • 10 4-15 • 1 = 16.665.
37. Para que a distribuição atenda à condição do problema, B deve ficar com 9
livros, ficando A e C, juntos, com 18 livros. Primeiro, podemos escolher os
9 livros que vão ficar com B. Isso pode ser feito de (2g7) maneiras distintas.
Em seguida, devemos distribuir os 18 livros restantes entre A e C. Para
cada um dos livros, note-se, temos duas opções: entregá-lo a A ou entregá-
10 a C, donde são 218 as possibilidades. Excetuando-se o caso em que A
fica sem nenhum livro e o caso em que C fica sem nenhum livro, temos que
o número total de distribuições é (297)(218 — 2) = 1.228.613.679.150.
38. (a) Podemos dispor a torre preta em qualquer uma das 64 casas do tabu­
leiro. Como as torres atacam em verticais e horizontais, para que as
torres branca e preta estejam em condições uma de atacar a outra,
podemos dispor a torre branca numa das 7 casas da linha em que está
a torre preta ou numa das 7 casas da vertical em que está a peça preta.
Logo, temos 14 opções para situarmos a torre branca. Logo, a resposta
ao nosso problema é 64 • 14 = 896 configurações possíveis.
(b) Para melhor compreendermos o problema, observemos a ilustração que
vem em seguida:
Parte II. Resoluções - Combinatória básica 65

Vamos dividir o problema em quatro casos, deixando as verificações


mais detalhadas das afirmações aqui feitas, ao leitor:
i. casas marcadas com a letra A;
Fixemos a rainha numa dessas casas. Para que tanto a rainha
como o bispo estejam em posição de ataque, só podemos situar
o bispo numa das diagonais em que está a rainha. Ocorre que,
se a rainha está numa das casas marcadas por A, são exatamente
7 as casas em que podemos colocar o bispo de modo que ambos
estejam em posição perigosa. Como são 28 as casas marcadas por
A, temos um total de 28 • 7 = 196 configurações possíveis nesse
caso.
ii. casas marcadas com a letra B;
Fixando a rainha em qualquer uma das casas marcadas por B,
temos agora 9 opções para o bispo. Logo, como temos 20 casas
marcadas por B, há 20 • 9 = 180 disposições nesse caso.
iii. casas marcadas com a letra C;
Basta repetir o raciocínio já empregado, chegando a 12 • 11 = 132
configurações.
iv. casas marcadas com a letra D.
Agora, temos apenas 4 casas marcadas por D e 13 opções para
situar o bispo, donde chegamos a 4 • 13 = 52 opções.
Portanto, somando todos os casos possíveis, temos 560 maneiras de
situarmos um bispo e uma rainha num tabuleiro de modo que os dois
estejam em posição de ataque.
Agora, prosseguiremos com a resolução da segunda pergunta deste
item. Para tanto, podemos nos utilizar da mesma divisão do tabuleiro
feita na figura anterior:
i. casas marcadas com a letra A;
Novamente, fixemos a rainha numa dessas casas. Para que nem
o bispo nem a rainha estejam em posição perigosa, não podemos
66 Problemas Resolvidos de Combinatória

colocar o bispo em nenhuma das 7 casas de diagonais às quais


pertence a rainha, nem nas 7 casas da vertical à qual ela pertence,
nem nas 7 casas da horizontal à qual ela pertence. Obviamente,
também não podemos situar o bispo na casa em que está a rainha.
Logo, temos 64-3-7—1 = 42 opções, donde se têm 28-42 = 1.176
disposições possíveis.
ii. casas marcadas com a letra B;
Agora, temos 20 casas marcadas por B e 40 opções para dispor o
bispo, donde se têm 20 • 40 = 800 configurações possíveis.
iii. casas marcadas com a letra C;
Basta que se repita o raciocínio, chegando a 12-38 = 456 con­
figurações.
iv. casas marcadas com a letra D.
Novamente, temos agora 4 casas para fixar a rainha e 36 opções
para situar o bispo, donde chegamos a 4 • 36 = 144 opções.
Logo, somando todas as possibilidades, temos 2.576 maneiras de situar
um bispo e uma rainha no tabuleiro de modo que os dois estejam
isentos de perigo.
(c) Agora, precisaremos ‘enxergar’ o tabuleiro de outra forma, devido ao
tipo de movimentação do cavalo. Observe a figura que vem em seguida:

Fpl |T| I TI
B c c E E|
F A A |Q
C A A F|
IF A A C
C A A F|
|E C C B
E F F PJ

Aqui também vamos dividir o problema em casos correspondentes às


letras distintas que temos representadas no tabuleiro:
i. casas marcadas com a letra A;
Fixando um dos cavalos numa dessas 16 casas, temos 8 casas nas
quais podemos colocar o outro cavalo de modo que um possa estar
em condições de capturar o outro. Logo, temos 16 • 8 = 128
disposições.
ii. casas marcadas com a letra B;
Colocando um cavalo numa dessas 4 casas, temos 4 opções para o
Parte II. Resoluções - Combinatória básica 67

outro cavalo. Logo, temos 4 • 4 = 16 opções.


iii. casas marcadas com a letra C;
A partir daqui, faremos apenas as contas, uma vez que o raciocínio
é o mesmo que o dos itens anteriores. São 16 ■ 6 = 96 disposições
possíveis.
iv. casas marcadas com a letra D;
Agora, temos 4-2 = 8 opções.
v. casas marcadas com a letra E;
Neste caso, são 8 • 3 = 24 as opções.
vi. casas marcadas com a letra F.
Finalmente, aqui temos 16 • 4 = 64 maneiras distintas.
Unindo os resultados encontrados, chegamos a um total de 336 manei­
ras de se disporem dois cavalos num tabuleiro de xadrez de sorte que
um esteja apto a tomar o outro.

39. (a) Sabemos que, ao se dividir um número inteiro por 3, o resto da divisão
é igual a 0, 1 ou 2. Assim, cada um dos números envolvidos no proble­
ma assume uma das 3 formas: 3Â;, 3k 4- 1 ou 3fc 4- 2. Ademais, é
conveniente observar que, de 1 a 100, temos 33 números da primeira
forma, 34 números da segunda forma e 33 números da terceira forma.
Logo, temos as seguintes possibilidades de obter soma múltipla de 3:
i. tomar 3 números da forma 3k;
Nesse caso, temos (333) = 5.456 escolhas possíveis.
ii. tomar 3 números da forma 3k + 1;
Agora, são (334) = 5.984 as opções.
iii. tomar 3 números da forma 3k 4- 2;
Analogamente, as opções ocorrem em número de (333) = 5.456.
iv. tomar um número de cada uma das 3 formas.
Agora, são 33 • 34 • 33 = 37.026 as opções de escolha.
Portanto, somando os resultados obtidos, temos um total de 53.922
seleções possíveis.
(b) De certa maneira, este item é mais simples do que o anterior, uma vez
que, dentre 3n números naturais consecutivos, sempre há n elementos
da forma 3k, n da forma 3k 4- 1 e n da forma 3k 4- 2. Analogamente,
os casos em que temos de dividir o problema são os mesmos do item
(a), exceto que aqui estamos trabalhando com valores incógnitos:
i. tomar 3 números da forma 3k;
Nesse caso, temos (3) escolhas possíveis.
68 Problemas Resolvidos de Combinatória

ii. tomar 3 números da forma 3fc 4-1;


Também são (£) as opções.
iii. tomar 3 números da forma 3k 4- 2;
Analogamente, as opções ocorrem em número de (3).
iv. tomar um número de cada uma das 3 formas.
Agora, são n • n • n = n3 as opções de escolha.
n(3n2—3n+2)
Assim, o total possível de escolhas é 3 • (3) 4- n3 = 2

40. De cada uma das m parcelas, temos que selecionar n objetos. Pelo Princípio
Fundamental da Contagem temos, pois:

(X) n! (2n)! (3n)! (mn)\


n! -0! n! • n! n! • (2n)! n! • ((m — l)n)l
(mn)!
~ (n!)™

maneiras de se efetuar a seleção.

41. Observe que, em geral, a divisão dos n = pq 4- r objetos entre as p pessoas


não é possível de se fazer de modo igualitário, isto é, de modo que cada
um receba o mesmo número de objetos. Como r < p, a única exceção
é o caso r = 0, no qual cada pessoa poderia receber q objetos. Note a
analogia do problema com o algoritmo da divisão de Euclides. Valendo-nos
do mesmo, torna-se possível propor uma solução ao problema: distribuir
exatos q objetos para p — r pessoas e exatos q 4-1 objetos para r pessoas.
Assim, obtemos uma divisão relativamente igualitária, e o mais igualitária
possível para o problema. Devemos, pois, escolher quais serão as p — r
pessoas que receberão q objetos cada e, em seguida, devemos selecionar os
objetos que cada uma das p pessoas irá receber. Vamos primeiro calcular as
possíveis distribuições de q objetos para as p — r pessoas que os receberão:

pq + r ^(p- l)g 4-r' í(r 4- 1)q4-t\


\ q )
(P9 + r)! ((p- l)ç4-r)! ((r + l)g + r)l
<7! • ((p - 1)<7 4- r)! ql ■ ((p - 2)g 4- r)! g! • (rg 4- r)!
ÍP9 + r)!
(g!)P~r • (rg4-r)!’
Parte II. Resoluções - Combinatória básica 69

que é o número procurado. Agora, prossigamos com o mesmo cálculo para


as r pessoas que receberão q 4- 1 objetos cada:

írq + r\ / (r — l)q 4- (r — 1) ? +1
U+1A Ç+1 <7+1
(rq + r)!_______________ ((r - l)q + (r - 1))!
(q 4- 1)! • ((r - l)q + (r - 1))! (q 4- 1)! • ((r - 2)q 4- (r - 2))!
(q + 1)! (rq + r)!
‘ (q + 1)! • 0! “ ((q + l)!)r ’

que é o resultado procurado. Portanto, basta agora que multipliquemos os


resultados encontrados, lembrando também de ‘escolher’ as p — r pessoas
que vão receber q objetos cada (com isso, as que vão receber q 4-1 também
ficam determinadas):

(P9 + r)! (rq + r)!


(A) (q!)p-r • (rq 4-r)! ((q + l)!)r
p\ (pq + r)l 1
(p — r)! ■ r! (q!)p-r (<z + ir. (<?!)r
p! • (pq + r)!
(p - r)! • r! ■ (q!)P ■ (q + l)r

Assim, este é o número de maneiras de se distribuírem os pq + r objetos


entre as p pessoas da maneira mais igualitária possível. Observe que, para
r = 0, a divisão (igualitária) é feita de ^7 maneiras.

42. E claro que o problema só faz sentido se supusermos, de antemão, que n


deve ser maior do que ou igual a 4. O número de maneiras de se escolherem
4 objetos dentre n é, claramente, (™). A parte trabalhosa do problema é
selecionar 4 objetos dentre 2n, apresentando-se estes em n pares distintos:

(a) selecionam-se 4 objetos de 4 tipos distintos;


Nesse caso, é fácil ver que temos (2) seleções possíveis (já que são n
os tipos distintos de objetos).
(b) selecionam-se 2 pares distintos de objetos;
Basta escolhermos quais serão os tipos de objetos de cada par. Como
são n os tipos disponíveis, resultam em (2) 33 seleções nesse caso.
(c) seleciona-se um par de objetos idênticos e 2 outros objetos distintos,
de modo que se tenham 3 tipos de objetos escolhidos.
Temos n possíveis escolhas para o par de objetos idênticos. A seguir,
70 Problemas Resolvidos de Combinatória

temos (n2escolhas para os 2 objetos distintos. Como os ‘blocos’ con­


siderados possuem tamanhos diferentes, são n(n21) 33 escolhas nesse
caso.

Adequando-se os resultados encontrados à condição do enunciado, basta


agora que façamos alguns cálculos:

CHICM)*•(";') n—1
2
n! n! (n- 1)!
=> 5
41-(n —4)1 = 2! • (n - 2)! + n 2! ■ (n —3)1
5n(n — l)(n — 2)(n — 3) n(n — 1) ( n(n — l)(n — 2)
24 “ 2 +
+ 2
5(n-2)(n-3) , .
-------- J2-------- -l + (n-2)
=> 5(n2 - 5n + 6) = 12(n - 1)
=> 5n2 — 37n + 42 = 0,

donde vem que n = | ou n = 6. Como n deve ser inteiro, temos que n = 6


é a resposta do problema.

43. (a) Para maior praticidade, vamos dividir o problema em três casos:
i. nenhum coelho preto se entoca;
Temos que contar os casos em que só há coelhos brancos nas tocas.
Podemos variar o número de coelhos brancos que estão entocados
de 0 a 7 e, como as tocas são consideradas distintas, basta que
escolhamos as tocas que serão ocupadas pelos coelhos brancos.
Assim:

(M)— 29
© - ©=512-9-1=502
é o número de maneiras de se entocarem os coelhos neste caso. (
Só como exemplo, (9) conta o número de configurações em que
exatamente 3 coelhos brancos estão entocados, isto é, escolhe as 3
tocas a serem por eles ocupadas.
ii. exatamente um coelho preto se entoca;
Agora, são sempre 9 as possibilidades de alocar o coelho preto.
Parte II. Resoluções - Combinatória básica 71

Uma vez entocado, basta que variemos o número de coelhos bran­


cos entocados de 0 a 7, e somemos todos esses casos, obtendo,
como em i.:

•©’•©
maneiras diferentes.
9 28 = 9-255 = 2.295

iii. os dois coelhos pretos se entocam.


Neste caso, basta que escolhamos as duas tocas que os alocarão,
fazendo novamente o número de coelhos brancos entocados variar
de 0 a 7:

©©*©©—©©-© •27 = 36-128 = 4.608.

Portanto, somando todos os resultados encontrados, obtemos 7.405


configurações possíveis dos coelhos nas tocas.
(b) Novamente, a divisão em casos se torna conveniente:
i. só os coelhos brancos são entocados;
De modo análogo ao do item i. de (a), temos 502 possibilidades.
ii. entre coelhos não brancos, há somente um coelho mestiço ou so­
mente um coelho preto entocado;
Como o coelho não branco pode ser de 2 tipos distintos, deve­
mos multiplicar o resultado encontrado no item ii. de (a) por 2,
obtendo 2 • 2.295 = 4.590 possibilidades.
iii. os coelhos mestiço e preto são entocados.
Temos 9 opções para a toca em que vai ficar o coelho preto e, por
conseguinte, 8 para o mestiço. Então, basta repetir o raciocínio
do item iii. de (a) para obter:

9-8
km:)—©]-
possibilidades.
9 ■ 8 • 27 = 72 • 128 = 9.216

Desta maneira, temos 502 4- 4.590 + 9.216 = 14.308 configurações


possíveis.
(c) Dividindo também em casos, temos:
i. casos em que só há coelhos brancos entocados;
Procedendo com raciocínio análogo ao empregado nos itens ante­
riores, temos um total de:
n+1
+ + ... + = 2n+1 - 1
1
72 Problemas Resolvidos de Combinatória

casos.
ii. há um coelho preto entocado.
Observando atentamente a resolução dos itens anteriores, fica fácil
intuir que temos, neste caso:

<•*'>© + (n + 1) Q (n + l)2n

possibilidades.
Assim, considerando a subdivisão em casos realizada, temos um total
de 2n+1 — l + (n+l)2n = (n+l+2)2n — 1 = (n+3)2n —1 possibilidades.
44. Facilitaremos muito nosso problema se o dividirmos em casos:
(a) paralelepípedos com três arestas distintas;
Fixemos nosso olhar sobre uma das seis faces do paralelepípedo, esco­
lhendo-a como base. Uma vez feito isso, temos 10 opções de tamanho
para um dos lados dessa face (qualquer natural entre 1 e 10), 9 opções
para o outro lado (já que as arestas do sólido devem ter tamanhos
distintos) e 8 opções para a altura do sólido. Portanto, 10 • 9 • 8 = 720
é o número de sólidos possíveis. Observe, no entanto, que estamos
contando 6 vezes cada um dos sólidos com 3 arestas distintas. A ilus­
tração que vem a seguir exemplifica 6 sólidos idênticos que, dentre os
720, estariam sendo considerados distintos:

3
2
1

2
V

3
4

Portanto, são ™ — 120 possibilidades nesse caso.


(b) paralelepípedos com duas arestas idênticas e uma distinta;
Agora, vamos fixar nosso olhar sobre uma das faces que apresenta
as duas arestas idênticas (observe que esta face será um quadrado).
Então, temos 10 opções para tal valor idêntico e, fixando esta face
como base, teremos 9 opções para a altura do paralelepípedo. Logo,
são 10 • 9 = 90 as opções nesse caso. Observe agora que cada caso está
sendo contado uma única vez, pois fixamos o quadrado como base.
Parte II. Resoluções - Combinatória básica 73

(c) paralelepípedos com 3 arestas idênticas.


Obviamente, temos 10 cubos distintos cujas arestas são números natu­
rais não maiores do que 10.

Portanto, temos 120 + 90 + 10 = 220 paralelepípedos distintos no total.

45. (a) Primeiramente, temos que escolher os 6 CDs que ocuparão a bandeja
do aparelho. Como dispomos de 80 CDs diferentes, são (86°) as pos­
sibilidades para isso. Uma vez tendo escolhido os CDs, precisamos
escolher em qual espaço cada um deles vai ficar. Como os espaços são
todos numerados, são 6! as maneiras de se permutarem os 6 CDs nos
mesmos. Portanto, (86°) • 6! = - 6! = fgf = 216.360.144.000 é a
resposta ao nosso problema. De fato, poderiamos tê-lo resolvido sim­
plesmente escolhendo qual CD ficaria na posição 1 (80 opções), qual
ficaria na posição 2 (79 opções) e, assim, sucessivamente, obtendo um
total de 80 • 79 • 78 • 77 • 76 • 75 maneiras, que corresponde ao mesmo
valor calculado anteriormente.
(b) A resolução que empregamos no item anterior agora se fará assaz útil
à que empregaremos neste. Assim como naquele item, escolhemos os
6 CDs que ocuparão a bandeja e, em seguida, precisamos dispô-los na
mesma. Devido ao fato de os espaços destinados aos CDs serem indis­
tinguíveis, teremos que utilizar o conceito de permutações circulares,
que já fora empregado num exercício precedente. Isto porque rotações
da bandeja em torno do seu centro conduzem a configurações idênticas
dos CDs. Assim, podemos permutar de (6 — 1)! = 5! maneiras os CDs
escolhidos. Logo, o número de maneiras de se escolherem e colocarem
6 CDs na bandeja é igual a (86°) • 5! = 36.060.024.000.

46. Primeiro, vamos calcular o total de números em que não figuram 2 dígitos
iguais consecutivos. Para a primeira posição, temos 9 opções de preenchi­
mento (não podemos fazê-lo com 0), para a segunda, são também 9 as
opções (só não podemos utilizar o dígito anterior). Assim, sucessivamente,
obtemos 96 números. Ademais, não é difícil ver que o total de números com
6 algarismos é igual a 9 ■ 105. Portanto, 9 ■ 105 — 96 = 368.559 é a resposta
procurada.

47. Sendo importante quais números ocuparão cada linha, precisamos distribuir
os 20 números em quatro grupos de 5 números cada, desprezando a ordem
em que as linhas aparecem. Isso equivale ao problema de distribuir 20 obje­
tos entre 4 caixas idênticas de tal modo que cada uma fique com exatamente
74 Problemas Resolvidos de Combinatória

5 objetos, o que pode ser feito de:


(?) ■ (?) ■ (?) ■ (I) 20!
4! (5!)4 ■ 4!
maneiras diferentes. Agora, feita a divisão dos quatro grupos de números
que ocuparão as linhas, devemos permutar os elementos de cada um deles,
pois a ordem dos elementos de cada linha deve ser levada em conta. Em
cada linha, podemos permutar os números selecionados de 5! maneiras e,
portanto, temos a seguinte resposta ao problema:
. (5I)4 =
(5!)4 ■ 4! 1 1 4! '
De forma mais sintética, poderiamos ter resolvido o problema da seguinte
maneira: como dispomos de 20 números para preencher 20 casas de um
tabuleiro, temos 20! maneiras de fazê-lo. Entretanto, cada seleção efetuada
está sendo contada 4! vezes, pois a ordem das linhas não importa. Assim,
obtemos o total de configurações distintas, um número nada modesto,
da ordem de 1017.
48. (a) Como as seqüências devem ter n dígitos, dos quais exatamente k de­
vem ser iguais a 0, basta que escolhamos quais serão as k posições a
serem ocupadas por 0’s. Isso pode ser feito de (£) maneiras. Eis a
resposta ao problema, já que as outras posições devem ser ocupadas
invariavelmente por l’s.
(b) Da mesma forma que no item anterior, precisamos escolher as k posi­
ções a serem ocupadas por 0’s, o que já vimos serem Q) escolhas.
Agora, para as demais n— k posições, devemos escolher entre l’s e 2’s
para preenchê-las. Logo, existem (£) • 2n-fc seqüências ternárias nas
condições do enunciado.
49. (a) Como, ao pressionarmos um botão, este volta à posição original, per­
mitindo reutilização, temos, para cada ação a ser executada, um total
de 6 + (2) = 21 opções, sendo 6 referente ao total de maneiras de se
pressionar um só botão e (®) ao total de maneiras de se pressionarem
dois botões concomitantemente. Logo, como são quatro as ações para
a execução de um comando, temos 214 = 194.481 comandos possíveis.
(b) Agora, uma vez pressionado um botão, não podemos mais utilizá-lo
em nenhuma ação seguinte. Dessa forma, é conveniente que dividamos
o problema em casos. Denominemos ações ‘simples’ aquelas que resul­
tam do pressionamento de um só botão e ações ‘duplas’ aquelas que
resultam do pressionamento conjunto de dois botões:
Parte II. Resoluções - Combinatória básica 75

i. comandos em que nenhuma ação dupla é realizada;


Este caso é relativamente simples. Podemos resolvê-lo escolhendo
os 4 botões (distintos) que serão utilizados para executar as quatro
ações simples. Em seguida, devemos permutar as ações escolhidas,
pois sua ordem importa. Logo, são (°) • 4! = 360 os comandos
possíveis.
ii. comandos que apresentam apenas uma ação dupla em sua execu­
ção;
Temos (|) opções de escolha para os 2 botões que serão utilizados
na ação dupla. Restam-nos 4 botões, dos quais somente 1 não
será utilizado, já que temos ainda 3 ações simples a executar.
São, portanto, 4 opções. Assim, o número de maneiras de apenas
selecionar os botões é igual a (®) • 4 = 60. Novamente, como a
ordem das ações importa, temos 60-4! = 1.440 comandos nesse
caso.
iii. comandos que apresentam duas ações duplas e duas ações simples
em sua execução.
Considerando a seqüência de tipos de ações dupla-dupla-simples-
simples, temos (®) • (2) • 2 -1 = 180 possibilidades de comandos. No
entanto, essa seqüência pode ser alternada, de sorte a resultarem
180 • 2H! = 1.080 comandos possíveis.
Somando os resultados encontrados nos itens i. a iii., obtemos um total
de 2.880 comandos possíveis.

50. (a) É importante observar que pode haver várias maneiras de se argumen­
tar em exercícios como este, de forma que a demonstração que aqui
faremos é apenas uma delas. Suponhamos que se tenha um conjunto
com n elementos distintos dos quais desejamos selecionar k. Suponha­
mos, ainda, que, em tal seleção, importe somente qual é o primeiro
elemento selecionado. Assim, tendo escolhido os k elementos de (£)
maneiras, temos k possibilidades para o primeiro elemento escolhido,
justamente o que exprime o primeiro membro da identidade. Por outro
lado, podemos fazer esta mesma seleção da seguinte maneira: escolhe­
mos o primeiro elemento dentre os n existentes e, em seguida, es­
colhemos os outros k — 1 elementos dentre os n — 1 restantes, que é
justamente o que ‘conta’ o segundo membro da igualdade acima.
(b) Observe que, sem = 1, recaímos na fórmula do item (a). Então, de
certa maneira, este item é uma generalização daquele, de tal modo
que poderiamos repetir o raciocínio lá empregado. Entretanto, a fim
76 Problemas Resolvidos de Combinatória

de ilustrar a versatilidade de exercícios como este, vamos dar outra


interpretação para tal identidade.
Considere que uma empresa está em processo de recrutamento de es­
tagiários para compor seu quadro de funcionários. Para tanto, dado
que n candidatos se inscreveram no processo, ela adotará um esquema
seletivo consistindo em duas fases: na primeira, serão selecionados
k candidatos mediante uma prova que visa a avaliar o seu raciocínio
lógico; na segunda, cada um destes k candidatos aprovados na primeira
fase será entrevistado, sendo apenas m dentre estes admitidos na em­
presa. Tal procedimento, é claro, resulta num total de (£) (^) seleções
possíveis.
De outro modo, podemos fazer a mesma contagem dividindo de an­
temão os k candidatos que passarão à segunda fase em dois grupos:
o daqueles que serão admitidos na empresa e o daqueles que não o
serão. Podemos selecionar os que serão admitidos na empresa de (”)
maneiras e os que serão reprovados na segunda fase de (£Z™) maneiras.
Assim, (”) (£Z™) também conta o número de seleções possíveis e, as­
sim, a igualdade é estabelecida.
51. Temos que contar o número de seleções de n + 2 objetos distintos dentre
2n + 4 disponíveis. Para tanto, considere em particular 4 objetos tomados
à parte dentre os 2n + 4 existentes. Analisaremos, assim, os seguintes casos
envolvendo os 4 objetos diferenciados:
(a) nenhum dos 4 objetos é escolhido;
Neste caso, devemos escolher os n + 2 objetos entre os 2n restantes.
Logo, são (n2^2) 33 seleções.
(b) exatamente 1 dos 4 objetos é escolhido;
Agora, temos 4 opções de escolha para o objeto que será tomado entre
os 4 especiais. Feito isso, devemos escolher os n + 1 faltantes dentre
os outros 2n objetos. Logo, temos 4(n2£1) opções.
(c) exatamente 2 dos 4 objetos são escolhidos;
Este caso tem resolução análoga à do precedente, dispensando ex­
plicações detalhadas. São (2)(2^) = 6(2^) 35 °PÇÕes.
(d) exatamente 3 dos 4 objetos são escolhidos;
Temos 4(n2”1) possibilidades de escolha, aplicando o mesmo raciocínio
anterior.
(e) todos os 4 objetos são escolhidos.
Uma vez escolhidos os quatro objetos, falta-nos escolher os outros
n — 2 dentre os 2n objetos. Assim, temos (n2”2) °PÇÕes neste caso.
Parte II. Resoluções - Combinatória básica 77

Observando que as possibilidades acima também perfazem todas as opções


de escolha de n 4- 2 objetos dentre 2n 4- 4, podemos somar os resultados de
cada caso, obtendo:

2n 4- 4 2n 2n .(2n\ Â í 2n \ ( 2n \
4-4 +6 +4 4- ,
n 4- 2 n 4- 2 n 4-1 \ nJ \n—Xj \n — 2)

que é justamente o resultado que se queria provar.

52. (a) Temos (12) opções de escolha para as 7 pessoas que ocuparão uma das
mesas, ficando as pessoas da outra mesa já determinadas. Em seguida,
devemos utilizar permutações circulares para acomodar as pessoas nas
mesas. Logo, são (12) • (7 — 1)!• (5 — 1)! = 13.685.760 as possibilidades.
(b) Agora, temos também (12) opções de escolha para as 7 pessoas que
ocuparão a mesa redonda, ficando as outras 5 já determinadas. Uti­
lizando, então, permutações circulares na mesa e permutações simples
no banco, temos (72) ■ (7 — 1)! • 5! = 68.428.800 maneiras de acomodar
as pessoas.

53. (a) O mínimo de adições necessárias ao cálculo de (£) é igual ao número


de adições necessárias à explicitação de todos os números da cunha
citada no enunciado. Sabemos, ainda, que (£) é o elemento situado na
n-ésima linha e na fc-ésima coluna do Triângulo de Pascal. Com base
em tais considerações, fica simples a resolução do exercício. Observe
o seguinte esboço da cunha em questão:

Figura 4: Excerto do Triângulo que contém a cunha.


78 Problemas Resolvidos de Combinatória

É fácil ver na figura que a cunha em questão é, na verdade, um


‘retângulo’ de dimensões 41 por 81 que contém 41-81 = 3.321 números
em sua composição. Todos esses números foram obtidos mediante uma
adição, exceto aqueles iguais a 1, os quais compõem dois ‘lados’ do
‘retângulo’. Dadas suas dimensões, temos 41 4- 81 — 1 = 121 números
iguais a 1 na cunha. Portanto, é 3.321 — 121 = 3.200 o menor número
de adições necessárias ao cálculo de (420°).
(b) Pelo mesmo raciocínio do item anterior, acima de (£), temos uma
cunha de dimensões (&4-l)x(n — fc+1) que contém um número de l’s
igual a(fc+l) + (n — fc + 1) — 1 = n+1 em sua composição. Portanto,
o mínimo de adições necessárias ao cálculo de (£) é:

(k + l)(n — k + 1) — (n + 1) = k(n — k).

De modo mais simples, bastaria que tomássemos uma cunha interna à


anterior de dimensões k x (n — k) (obtida desconsiderando-se os l’s).
Para todos os seus k(n — fc) elementos, uma operação de soma foi
realizada.

54. (a) Desejamos apenas dividir os elementos do conjunto dado em duas


partes não vazias. Assim, podemos simplesmente selecionar elemen­
tos do conjunto para compor uma das partes, e aqueles que restarem
determinarão a outra parte. Como as partes compostas devem ser
não vazias, basta que escolhamos de 1 a 9 elementos e somemos os
valores encontrados. Agindo assim, porém, estaríamos contando duas
vezes cada partição: por exemplo, os números (14°) e (g°) consideram,
cada um, a partição {1,2,3,4}U {5,6,7,8,9,10}. Logo, o resultado do
exercício é:
(10) + (10)+-.+ (10) + (10) 210 — 2
= 511.
2 2
(b) Agora, depois de feito o item anterior, fica fácil observar que a genera­
lização do resultado é:
I
(í) + O + - + (n-J + („-,) 2n -2
= 271-1 - 1.
2 2

55. (a) A engenhosidade da resolução que proporemos reside num simples as­
pecto de visualização do retângulo quadriculado da figura exibida no
enunciado da questão. Imaginemos que ele seja um tabuleiro de xadrez
Parte II. Resoluções - Combinatória básica 79

e, como tal, possua suas casas diferenciadas em pretas e brancas. As­


sim, se uma moeda estiver numa casa de determinada cor, realizar
movimentos diagonais com a mesma equivale a deslocá-la para qual­
quer outra casa desta cor. Portanto, como as moedas são distintas
(digamos, moeda A e moeda B), temos 4 configurações não equiva­
lentes, a saber:
• as duas moedas em casas pretas;
• as duas moedas em casas brancas;
• a moeda A numa casa preta e a moeda B numa casa branca;
• a moeda A numa casa branca e a moeda B numa casa preta.
Se as duas moedas fossem iguais, teríamos apenas 3 configurações não
equivalentes, pois os 2 itens finais citados anteriormente seriam equi­
valentes.
(b) Pelo mesmo procedimento do item (a), isto é, visualizando um tabulei­
ro de xadrez no retângulo quadriculado, vamos resolver o problema
para cinco moedas distintas dividindo em casos:
i. todas as moedas ficam em casas pretas;
E claro que só há uma configuração não equivalente.
ii. 4 moedas ficam em casas pretas e uma fica numa casa branca;
Temos 5 opções para a moeda que ficará na casa branca e, por­
tanto, 5 configurações não equivalentes.
iii. 3 moedas ficam em casas pretas e duas ficam em casas brancas;
Temos (3) = 10 opções para escolhermos as 3 moedas que ficarão
em casas pretas e, portanto, 10 configurações não equivalentes.
iv. duas moedas ficam em casas pretas e 3 ficam em casas brancas;
Temos (|) = 10 opções para escolhermos as duas moedas que
ficarão em casas pretas e, portanto, 10 configurações não equiva­
lentes.
V. uma moeda fica numa casa preta e 4 ficam em casas brancas;
Temos 5 opções para escolhermos a moeda que ficará na casa preta
e, portanto, 5 configurações não equivalentes.
vi. todas as moedas ficam em casas brancas.
Também neste caso só temos uma configuração não equivalente.
Portanto, resulta um total de 14-54-10 + 104-5+ 1 = 32 configurações
não equivalentes das 5 moedas distintas. Tal resultado, porém, leva-
nos a inferir outra maneira de resolver o problema (32 = 25). Como as
moedas são distintas e arranjos que resultam de movimentos diagonais
80 Problemas Resolvidos de Combinatória

das mesmas são considerados equivalentes, basta que determinemos


qual a cor da casa em que ficará cada moeda. Dado que as cores são
duas, temos, de fato, 25 = 32 configurações não equivalentes.
Através do método mais simples discutido ao final do parágrafo ante­
rior, vamos resolver o problema considerando agora 3 moedas idênticas
e duas distintas. Quanto às três moedas idênticas, temos 4 opções:
colocá-las todas em casas pretas, todas em casas brancas, duas em
casas pretas e uma em casa branca ou duas em casas brancas e uma em
casa preta. As outras duas moedas, como são distintas, fornecem duas
opções cada uma: colocá-la numa casa preta ou numa casa branca.
Logo, são 4 ■ 2 • 2 = 16 as configurações não equivalentes das 5 moedas.
56. Uma possível interpretação combinatória para a identidade é que, de um
conjunto com n elementos, o total de subconjuntos com número par de ele­
mentos é igual ao total de subconjuntos com número ímpar de elementos.
A fim de apresentarmos uma prova combinatória da identidade, seja X um
conjunto com n elementos. Tomemos, então, x E X. Sejam, ainda, p o
total de subconjuntos de X com número par de elementos (observe que,
pela interpretação combinatória que apresentamos acima, p corresponde ao
primeiro membro da identidade) e i o total de subconjuntos de X com
número ímpar de elementos (da mesma forma, i corresponde ao segundo
membro da identidade). Queremos provar que p = i e, para tanto, exibire­
mos outras maneiras de encontrar p e i:
(a) Cálculo de p;
Considerando o elemento x, contaremos o número de subconjuntos
de X com número par de elementos dividindo o problema em dois
casos: subconjuntos que contêm x e subconjuntos que não contêm x.
No primeiro caso, como já temos selecionado o elemento x, basta que
selecionemos números ímpares de elementos dentre os n — 1 restantes;
no segundo, como não selecionamos x, precisamos escolher números
pares de elementos dentre os n — 1 restantes. Assim, temos:
(n — 1 n—1 n—1 n— 1
P 1
+ 3
+ ••• + 0
+ 2
+ ...

contêm x não contêm x


/'n — 1 (n—i n—1 n—1
~ \ 0
+ 1
+ 2
+ ••• + n—1
= 2n-1.

(b) Cálculo de i.
Parte II. Resoluções - Combinatória básica 81

De modo análogo ao procedido para o cálculo de p, temos:

n—1 n—1 n—1 n— 1


i =
0
+ 2
+ ••• + 1
+ 3
_|-----

contêm x não contêm x


ín — 1 ín — 1 n— 1 71 — 1
+ 1
+ 2
+ ••• + n—1
\ 0
2n-i

Logo, segue que p = 2n 1 = í, ficando provada a identidade em questão.

57. (a) Como as repetições são permitidas, temos 6 opções de letras para ocu­
par cada uma das cinco posições da seqüência. Logo, são 65 = 7.776
seqüências possíveis.
(b) Agora, como não se permitem repetições, temos 6 opções de preenchi­
mento para a primeira posição, 5 para a segunda (não podemos utilizar
a letra que preencheu a primeira posição), e assim por diante, obtendo
6 ■ 5 • 4 • 3 • 2 = 720 possíveis seqüências.
(c) Podemos escolher uma das 5 posições para ser ocupada pela letra C e,
em seguida, preencher as demais a partir das letras restantes. Temos,
então, 5 • 5 • 4 • 3 • 2 = 600 possíveis seqüências.
(d) Este item não pode ser resolvido como o item (c), pois se assim fizermos
estaremos contando casos repetidos (isso ocorre porque as repetições
de letras são permitidas). Podemos contar o número de seqüências
que não contêm a letra C e, em seguida, subtrair esse valor do total
encontrado em (a). O número de seqüências que não contêm C é igual
a 55 e, portanto, 7.776 — 55 = 4.651 é o total de seqüências procurado.

58. (a) Como todos os carros são distintos, temos 20! maneiras de colocar os
carros no estacionamento. Se considerarmos indistinguíveis os carros
de mesma cor, podemos utilizar permutações com repetições para obter
um total de = 99.768.240 disposições diferentes.
(b) Uma vez determinados os grupos de vagas para cada cor de carros,
basta permutarmos os carros de cada cor em seu grupo, obtendo um
total de 7! -5! -8! = 24.385.536.000 configurações diferentes. Para o caso
de só haver necessidade de que carros de mesma cor fiquem juntos, só
precisamos multiplicar o valor encontrado anteriormente por 3!. Logo,
3! • 7! • 5! • 8! = 146.313.216.000 é o número procurado.
82 Problemas Resolvidos de Combinatória

59. (a) Basta que permutemos as 10 pedras nos espaços a elas destinados para
descobrir o número de maneiras. Como são 10 as pedras distintas,
temos 10! = 3.628.800 maneiras de cravejar as pedras na moldura do
quadro.
(b) Agora, o fato de rotacionarmos o espelho em torno de seu centro con­
duz a configurações equivalentes, o que sugere o uso de permutações
circulares (observe que isso não vale para o item anterior, pois um
quadro, mesmo que circular, tem uma posição preestabelecida). As­
sim, são (10 — 1)! = 9! = 362.880 cravejamentos possíveis.
(c) Como já explicamos no Exercício 12, além de aplicarmos permutações
circulares para montar o colar, devemos dividir o resultado final por 2,
dado que colares idênticos podem ser obtidos ao rotacionarmos cada
um deles em torno de um eixo diametral. Logo, são = 181.440
colares possíveis.
60. (a) Uma interpretação para a identidade é a de que, dada uma coleção de
n objetos distintos, o número de maneiras de selecionarmos k objetos é
igual ao número de maneiras de selecionarmos n — k objetos. Mas isto
é evidente, uma vez que, para cada escolha de k objetos, deixamos de
selecionar n — k. Em outras palavras, podemos tanto contar os grupos
de objetos escolhidos como contar os grupos de objetos não escolhidos.
(b) O primeiro membro da igualdade conta o número de seleções de k
objetos dentre n distintos. Podemos fazer essa mesma conta, porém,
fixando um dado elemento dentre os n existentes, e dividindo as sele­
ções em dois grupos: o daquelas em que o dado elemento não é sele­
cionado (consideradas em pois basta que escolhamos k elemen­
tos dentre os n— 1 restantes) e o daquelas em que o dado elemento é se­
lecionado (como já temos um elemento selecionado, basta que escolha­
mos k — 1 dentre os n — 1 restantes). Assim, concluímos que vale
a identidade do enunciado, pois ambos os seus membros ‘contam’ o
mesmo tipo de seleções.
61. Suponhamos que existam dois conjuntos, denotados por A e B, tais que
|A| = |B| = n e, além disso, que os 2n elementos de A U B sejam distintos
entre si. Assim, considerando a seleção de n elementos quaisquer, é claro
que dispomos de (2^) opções de escolha. Por outro lado, podemos efetuar a
mesma escolha, tomando a elementos de A e b elementos de B de tal modo
que a 4- b = n, ou seja, b = n — a. Nesse caso, temos (”) (n2a) escolhas
possíveis para cada valor de a. Note, porém, que, para contarmos todas
as seleções consideradas por (2”), é preciso que variemos a de 0 a n (com
Parte II. Resoluções - Combinatória básica 83

isso, é claro que n — a vai variar de n a 0), o que é equivalente a retirarmos


diferentes quantias de elementos de A e B. Assim, podemos escrever:

Ê(X:.HXMX:.)
chegando ao resultado a que aspirávamos. Uma vez provada essa identidade,
note finalmente que, como, para cada a, temos (”) = (n”a) (Exercício 60),
segue que:
2

CX(XC)
62. (a) Note que:

f Pn \ = (pn)!
\pn — n) nl(pn — n)!
pn(pn — 1)!
n(n — l)!(pn — n)l
(pn - 1)1
= P (n — l)!(pn — n)!

donde segue o resultado.


(b) Se 0 for um dos k inteiros consecutivos em questão, é claro que o pro­
duto entre eles resulta 0 que, trivialmente, é divisível por kl. Restam,
pois, dois casos a considerar: todos os k inteiros consecutivos são nega­
tivos ou todos são positivos. O caso em que todos são negativos pode
ser analisado tomando-se o oposto de cada um dos k inteiros, de modo
que podemos nos limitar à análise do caso em que todos os k in­
teiros são positivos. Suponhamos que n 4- 1 (n > 0) seja o primeiro
inteiro do produto. Então, temos que analisar a divisibilidade de
(n 4- l)(n 4- 2) • • • (n 4- fc) por kl. Observe, porém, que:

(n 4- l)(n + 2) • ■ • (n 4- k) 1 • 2 • • • n(n 4-1) • • ■ (n + k)


kl 1 • 2 • • • n • kl
(n 4- fc)!
n! • &!
ín 4-
\ k )'
84 Problemas Resolvidos de Combinatória

número que sabemos ser inteiro, donde nossa demonstração está con­
cluída.

63. Podemos considerar a existência de uma coleção de 3n objetos distintos


da qual desejamos selecionar 3 objetos quaisquer. É claro que o número
de seleções distintas que podemos fazer é igual a (33n). A mesma seleção,
entretanto, pode ser realizada de outra maneira: podemos dividir os 3n
objetos em três conjuntos de n objetos cada, e selecionar 3 objetos das
seguintes maneiras:

(a) selecionar 3 objetos de um só grupo de n objetos;


Para cada um dos três grupos, temos (£) possíveis escolhas, donde são
3(3) as possibilidades.
(b) selecionar 2 objetos de um grupo e 1 objeto de outro grupo;
Temos 3 opções de escolha para o grupo de objetos do qual vamos
retirar 2 objetos e, por conseguinte, 2 escolhas para aquele do qual
retiraremos 1 só objeto. Logo, são 6 as opções de escolha dos dois
grupos. Quanto ao grupo do qual vamos retirar 2 objetos, temos Q)
escolhas, e quanto àquele do qual vamos retirar 1 objeto, temos n
escolhas. Logo, são ônQ) escolhas possíveis nesse caso.
(c) selecionar 1 objeto de cada um dos três grupos.
São n opções de escolha para cada grupo, donde há n3 possibilidades
nesse caso.

Somando-se os resultados dos três casos, obtemos a expressão referente ao


segundo termo da identidade a ser provada e, como esta soma ‘conta’ o
mesmo que (3.^), temos provado o que queríamos.

64. (a) Notemos que (a + b 4- c + d)12 refere-se a um produto de 12 fatores


iguais a (a + ò + c + d). O termo ab4c2d5 apresenta 1 fator a, 4 fatores
ò, 2 fatores c e 5 fatores d. Podemos selecionar 1 fator a de qualquer
um dos 12 fatores (a + b + c + d). Assim, são (12) escolhas possíveis do
fator a. Uma vez realizada tal escolha, restam 11 fatores (a + b + c+d)
para selecionarmos 4 fatores b. Assim, temos (141) possíveis escolhas.
Prosseguindo com o raciocínio, concluímos que o coeficiente de ab4(?d5
em (a + b + c + d)12 é igual a:

(X)G)©-»»
Parte II. Resoluções - Combinatória básica 85

(b) Com o mesmo raciocínio do item anterior, atentando-nos aos números


que multiplicam a, b, c e d em (3a — b 4- c — 4d)12, concluímos que o
coeficiente procurado é igual a:

33 -479.001.600.

(c) Os termos da expansão multinomial de (a 4- b + c + d)12 são da forma


aab^c7dó, com a, 0, 7 e Ô inteiros não negativos sujeitos à condição
«4-/34-74-5=12. O número de termos distintos na dita expansão
corresponde, assim, ao número de soluções inteiras não negativas da
equação anterior. Portanto, existem (12^2-1) = (12) = 455 termos
distintos, pelo Exercício 25.
65. (a) Começando pelas 5-seleções que podem ser feitas, como os elementos
devem ser distintos, temos (|) = 56 possibilidades. Agora, feitas as
seleções, podemos encontrar o número de 5-seqüências multiplicando
cada 5-seleção por 51, obtendo um total de 56 • 5! = 6.720 seqüências
de 5 letras.
(b) Permitindo-se repetições, temos 8 opções de preenchimento para cada
posição da 5-seqüência, donde há 85 = 32.768 seqüências nesse caso.
Se xx denotar o número de vezes em que a letra X é selecionada, o
número de 5-seleções que podem ser realizadas é igual ao número de
soluções inteiras não negativas da equação xa 4- %b 4- • • • 4- xh = 5,
que sabemos ser igual a (°+|-1) = (g2) = 792, pelo Exercício 25.
(c) Procedendo de modo análogo ao do item anterior, obtemos um total de
810 = 1.073.741.824 seqüências de 10 letras e um total de 10-seleçÕes
igual a (‘“Ao”1) = (13 = 19-448-

66. Consideremos que a distribuição das cartas aos 4 jogadores se dê através da


seqüência ouros-espadas-copas-paus. Suponhamos que o primeiro jogador
receba 4 cartas de ouros e 3 de cada um dos outros três naipes. Assim, na
ordem que determinamos, são:
/13\ Z13\ /13\ /13\ = (13!)4
\4/\3/\3/\3/ 4! • 9! • (3!)3 • (101)3
as distribuições possíveis para esse jogador. Agora, podemos distribuir as
cartas a um segundo jogador, supondo que este receba 4 cartas de espadas
e 3 cartas de cada um dos naipes remanescentes. Temos, portanto:
/9\ /10\ /10\ /10\ 9! • (10!)3
WU A 3 /V/ ~ (3!)3 - (6!)2 ■ 4! ■ (7!)2
86 Problemas Resolvidos de Combinatória

distribuições para o segundo jogador. Agora, podemos fazer a distribuição


para o terceiro jogador, supondo que este receba 4 cartas de copas e 3 cartas
de cada um dos 3 naipes remanescentes. Assim:

/6\ Z6\ /7\ (7\ (6!)2 ■ (7!)2


\3j \3J W\3j (3!)« • (4!)2
é o número de distribuições para ele. Com isso, já ficam determinadas as
13 cartas do último jogador, que receberá 4 cartas de paus e 3 cartas de
cada um dos 3 naipes remanescentes. Agora, podemos tomar o produto dos
resultados encontrados, lembrando, ainda, de multiplicar o resultado final
por 4!, que se prestará a decidir o naipe de quatro cartas recebido por cada
jogador, já que, na resolução aqui empregada, determinamos que o primeiro
jogador recebesse 4 cartas de ouros, o segundo 4 de espadas, o terceiro 4 de
copas e o último 4 de paus. Assim, nossa resposta é:

(13!)4 9! ■ (10!)3 (6!)2 • (7!)2 (13!)4


4!-
4! ■ 9! • (3!)3 ■ (IO!)3 ' (3!)3 • (6!)2 • 4! • (7!)2 ’ (3!)6 • (4!)2 “ (3!)12 • (4!)3’

número da ordem de 1025.

67. Observe o seguinte esboço das duas retas:

Como as retas têm intersecção no ponto A, a resolução precisa ser dividida


em dois casos:

(a) Triângulos que não contêm o ponto A;


Podemos tomar um ponto distinto de A em r e dois distintos de A em
s ou dois em r e um em s. Temos, portanto, 4 (2) +0(2) = 40 + 30 = 70
triângulos possíveis.
(b) Triângulos que contêm o ponto A.
Como o ponto A já é um dos vértices do triângulo, basta que tomemos
Parte II. Resoluções - Combinatória básica 87

mais um ponto em r e mais um em s, ambos distintos de A. Logo, são


4 • 5 = 20 triângulos nesse caso.

Resultam, assim, 70 + 20 = 90 possíveis triângulos.

68. (a) Observe que 540 = 22 • 33 • 5. Logo, os divisores inteiros e positivos de


540 são todos os números da forma 2Q • 3^ • 57, com a valendo 0, 1 ou
2, valendo 0, 1, 2 ou 3 e 7 valendo 0 ou 1. Assim, pelo Princípio
Fundamental da Contagem, 540 possui 3 • 4 • 2 = 24 divisores.
Por sua vez, 283.500 = 22 • 34 • 53 • 7. Assim, analogamente ao que
fizemos anterior mente, segue que 283.500 possui 3 • 5 • 4 • 2 = 120
divisores inteiros e positivos.
(b) Considere a seguinte tabela:
2o 3o 5o
21 31 51
22 32
33
Para formarmos um divisor de 540, devemos tomar, como já vimos em
(a), um número de cada uma das 3 colunas da tabela. Assim, a soma
dos divisores de 540 pode ser calculada por:

2°3°5° -I- 2°3°51 + 2°315° 4------- F 22335° 4- 223351 =


= (2o 4- 21 4- 22)(3° + 31 4- 32 4- 33)(5° 4- 51)
= 7 • 40 • 6 = 1.680.

Note que, na expressão acima, realmente são somados todos os divi­


sores de 540.
Agora, como já vimos que 283.500 = 22 • 34 • 53 • 7, obtemos, por
raciocínio análogo ao empregado para 540, que a soma dos divisores
de 283.500 é igual a:

(2°+21+22)(3o+31+32+33+34)(5°+51+52+53)(7<)+71) = 1.057.056.

69. (a) Podemos considerai1 cada casal como um bloco único, uma vez que
marido e mulher não podem se sentar separadamente. Assim, dispo­
mos de 6 blocos que podem ser permutados. Além disso, as ordens
marido-mulher e mulher-marido dentro de cada bloco devem ser leva­
das em conta. Assim, o número de maneiras de se disporem os casais
nas cadeiras é igual a 6! • 26 = 720 • 64 = 46.080.
88 Problemas Resolvidos de Combinatória

(b) Novamente, podemos considerar blocos formados por cada casal. As­
sim, teremos 11 posições a serem ocupadas por 6 casais. Ademais,
importa também a ordem em que se encontram marido e mulher den­
tro de cada bloco. Portanto, são (11 • 10 • 9 • 8 • 7 • 6) ■ 26 = 21.288.960
as configurações possíveis.
Uma outra maneira de resolver o exercício seria pensar que dispomos
de 11 objetos, sendo 6 casais e 5 cadeiras idênticas. Assim, utilizando o
conceito de permutações com repetições, teremos • 26 = 21.288.960
disposições diferentes, como já havíamos calculado.
70. (a) Temos 3 casos a considerar:
i. cada país é colorido de uma cor distinta;
São m(rn — l)(m — 2)(m — 3)(m — 4) maneiras distintas.
ii. a cor usada para colorir a Alemanha é igual à usada para colorir
a Eslováquia, e as cores da Polônia e da Áustria são distintas, ou
vice-versa;
Temos m opções de cor para a Rep. Tcheca, m — 1 opções para
colorir os dois países opostos de mesma cor, e (m — 2)(m — 3)
opções para colorir os dois países opostos de cores distintas. Logo,
considerando as duas possibilidades, há 2m(m — l)(m. — 2)(m — 3)
maneiras distintas.
iii. a cor usada para colorir a Alemanha é igual à usada para a Eslová­
quia, e a usada para colorir a Polônia é igual à usada para a
Áustria.
Temos m opções para colorir a Rep. Tcheca, m — 1 opções para
colorir um dos pares de países opostos e m — 2 para colorir o
outro par de países opostos. Logo, são — !}{rn — 2) maneiras
distintas.
Somando os resultados obtidos, chegamos a um total de:
m(m — l)(m — 2)(m — 3)(m — 4) 4- 2m(m — — 2}(m — 3)4-
m(m — — 2) =
= m(m — l)(m — 2)[(m — 3)(m — 4) + 2(m — 3) 4-1]
= m(m — l)(m — 2)(m2 — 5m 4- 7)
maneiras diferentes de colorir o mapa.
(b) Como vimos no item iii. de (a), são necessárias pelo menos 3 cores
para colorir o mapa. Ainda pela resolução lá empregada, temos um
total de 3(3 — 1)(3 — 2) = 6 maneiras distintas de colorir o mapa nesse
caso.
Parte II. Resoluções - Combinatória básica 89

71. (a) Sabemos que:

z=o x 7
donde o coeficiente de xm em (1 4- x)n é (£). Por outro lado, temos
também que:

’n-p p
(l + xr-p(l+:r)p
e>] ■
Para encontrarmos o coeficiente de xm na expansão acima, devemos
estabelecer a condição i 4- j = m em relação ao seu segundo membro.
Assim, o termo que multiplica é igual a:

n—p

m ,
m
fc=0 x
m — kj \fc/

Como (1 + x')n = (14- a;)n P(1 4- x)p, segue o resultado.


(b) Dado um conjunto com n elementos distintos, consideremos a divisão
do mesmo em dois subconjuntos fixos: um com n — p elementos e
outro com p elementos. Para selecionarmos m elementos dentre os n,
é claro que há (^) possibilidades. Por outro lado, podemos selecionar
i elementos do conjunto que contém n—pej elementos do que contém
p, de tal maneira que i 4- j = m. Assim, como em (a), obtemos:

/n\ = /n-p\/p\ /n-p\Z p


+ ... 4- /n - p\
\m / \ 0 / \ 1 J \m — 1 V m AO/

como queríamos.

72. (a) Para resolvermos este problema, torna-se muito útil uma divisão em
casos:
i. planos que não contêm nenhum ponto de Pi;
Neste item se mostra a importância da observação do enunciado
de que, se 4 pontos de P são coplanares, então eles são pontos de
90 Problemas Resolvidos de Combinatória

Pi. Isso implica que, tomando-se 3 pontos quaisquer em P \ P\,


obtemos um plano que contém esses 3 pontos e não contém mais
nenhum ponto de P. Em outras palavras, nãos corremos o risco
de contar planos repetidos. Assim, são (^) = 286 os planos neste
caso.
ii. planos que contêm exatamente um ponto de P\;
A observação do item anterior também se faz importante neste
para garantir que não contemos planos repetidos. Assim, são 12
opções para a escolha de um ponto de P\ e (g3) = 78 escolhas de
um par de pontos de P \ P\. Logo, como 3 pontos determinam
um plano, temos 12 • 78 = 936 planos.
iii. planos que contêm exatamente 2 pontos de Fi ;
A mesma observação dos itens anteriores desempenha, novamente,
papel fundamental na resolução deste item. Basta que tomemos
dois pontos em Pi e um ponto em P \ P\. Logo, obtemos mais
(j2) • 13 = 858 planos.
iv. planos que contêm 3 ou mais pontos de P±.
Nesse caso, há somente um plano, pois os 12 pontos de Pi são
coplanares.
Portanto, somando todos os resultados encontrados, obtemos um total
de 2.081 planos.
(b) A resolução deste item tem analogia com a do anterior e, por isso,
também nos sugere uma divisão em casos:
i. triângulos que não contêm nenhum ponto de Q\;
Basta tomar 3 pontos em Q\Q\. Logo, são (g2) = 220 triângulos.
ii. triângulos que contêm exatamente um ponto de Qi;
Podemos tomar um ponto em Qi e dois pontos em Q \ Qi. Logo,
temos mais 10 = 660 triângulos.
iii. triângulos que contêm exatamente 2 pontos de Qi.
Nesse caso, com raciocínio análogo ao do item anterior, há mais
(2°) • 12 = 540 triângulos.
Assim, é possível formar 220 + 660 4- 540 = 1.420 triângulos distintos
com os pontos de Q.
73. (a) Sabemos que um polígono regular de n lados possui também n vértices.
Assim, como estes são 3 a 3 não colineares, basta que selecionemos
3 deles para compor um triângulo. Logo, sem qualquer restrição, o
número de triângulos que são possíveis de se formarem com os vértices
do polígono é igual a (3) •
Parte II. Resoluções - Combinatória básica 91

(b) Do total de triângulos calculado no item (a), vamos subtrair o número


de triângulos que contêm 1 ou 2 lados do polígono:
i. triângulos que contêm exatamente um lado do polígono;
Temos n opções para o lado do polígono compartilhado com o
triângulo. Como só se utiliza um lado do polígono, o terceiro
vértice do triângulo deverá ser escolhido dentre os n — 4 restantes
(isso pode ser facilmente verificado num desenho). Assim, temos
n(n — 4) triângulos em tais condições.
ii. triângulos que contêm exatamente 2 lados do polígono.
Agora, basta que tomemos 3 vértices consecutivos do polígono. E
fácil ver, assim, que temos mais n triângulos.
Desse modo, há (3) — [n(n — 4) + n] = n(n~4Xn~5} triângulos nas
condições do enunciado.

74. (a) Como nas seleções não importa a ordem dos números, podemos iniciá-
las pelo número menor. Assim, tomando qualquer número compreen­
dido entre 1 e 91, existe um único número entre 1 e 100 que possui
nove unidades além dele. Assim, o número de seleções é igual a 91.
(b) De acordo com o item anterior, há 91 pares de números compreendi­
dos entre 1 e 100 cuja diferença é exatamente 9. Prosseguindo com o
raciocínio, há 92 pares cuja diferença é exatamente 8. Assim, sucessi­
vamente, até 99 pares cuja diferença é exatamente 1. Logo, a resposta
ao problema é 91 + 92 + • • • + 99 = 855.
Procedendo de outra maneira, podemos notar que, para cada número
compreendido entre 1 e 91, existem 9 números entre 1 e 100 que pos­
suem de uma a 9 unidades além do número inicial. Daí por diante,
temos: 92 apresenta apenas 8 números maiores do que ele nas mes­
mas condições; 93 apresenta apenas 7 números maiores do que ele
nas condições do enunciado. Seguindo com este raciocínio, o número
possível de seleções é igual a91-9 + 8 + 7-|------- F 1 = 855, coincidindo
com o resultado anterior.

75. Consideremos a seguinte notação: número de cartas de um primeiro naipe-


número de cartas de um segundo naipe-número de cartas de um terceiro
naipe-número de cartas de um quarto naipe. Agora, vamos dividir o pro­
blema em casos, adotando tal notação e observando que o número de cartas
de cada naipe tem que ser no mínimo 1:

(a) 4-1-1-1;
Temos (43) opções de escolha para as cartas do primeiro naipe e 13
92 Problemas Resolvidos de Combinatória

para a dos naipes restantes. Além disso, há 4 possíveis escolhas para


o naipe do qual serão retiradas 4 cartas. Assim, o número de escolhas
é igual a 4 [(J43) • 133] = 6.283.420.
(b) 3-2-1-1;
Temos, neste caso, (13) escolhas para as cartas do primeiro naipe,
(j3) para as do segundo, e 13 para cada carta dos naipes restantes.
Ademais, há || = 12 escolhas para os naipes dos quais serão retiradas,
respectivamente, 3 cartas, 2 cartas, 1 carta e 1 carta. Assim, temos
12 [(g3) • (g3) • 132] = 45.240.624 escolhas nesse caso.
(c) 2-2-2-1.
Agora, por raciocínio análogo ao anterior, as escolhas ocorrem em
número de 4 [(13)3 • 13^ = 24.676.704.

Portanto, somando as quantidades obtidas nos três itens, resulta um total


de 76.200.748 possíveis escolhas nas condições do enunciado.

76. Vamos distribuir uma fruta por vez. Com relação às maçãs, estamos in­
teressados em saber o número de soluções inteiras positivas da equação
xi + X2 + X3 + X4 = 10. Assim, há = (3) = 84 distribuições
possíveis, pelo Exercício 25. Analogamente, há (Jz}) = (3) = 20 dis­
tribuições possíveis para as laranjas e (4Zj) = (3) = 56 para as pêras.
Logo, temos 84 • 20 • 56 = 94.080 distribuições possíveis das frutas nas qua­
tro caixas.

77. (a) Note que, para obtermos um desarranjo de RADAR, a letra D não
pode ocupar a posição central. Além disso, ocupando a letra D qual­
quer uma das outras 4 posições, a palavra está determinada. Por
exemplo, se a letra D ocupar a primeira posição, então os 2 A’s devem
ficar nas posições terceira e quinta, restando as outras duas posições
para os R’s. Assim, são 4 os desarranjos possíveis.
(b) Na palavra AGRAVAM, temos 3 A’s e outras 4 consoantes distintas.
Para obtermos um desarranjo de AGRAVAM, as posições de 3 das 4
consoantes devem ser ocupadas por A’s. Por conseguinte, a posição
de uma das consoantes deve ser ocupada por outra consoante (distinta
dela, obviamente). Assim, temos 4 opções de escolha para a consoante
cuja posição será ocupada por outra consoante, e 3 opções de escolha
para a letra que ocupará tal posição. Feito isso, já determinamos as
posições dos 3 A’s e de uma das consoantes. Quanto às outras 3,
podemos permutá-las nas posições anteriormente ocupadas por A’s de
Parte II. Resoluções - Combinatória básica 93

3! maneiras. Logo, são 4-3-3! = 72 os possíveis desarranjos da palavra


AGRAVAM.

78. Consideremos inicialmente que a roda-gigante possua apenas 8 bancos.


Logo, fazendo:
(£)(?)-©© 16!
8! 28 ■ 8! ’
dividimos as 16 crianças em 8 grupos de duas crianças cada um. Além
disso, cada grupo deve ser multiplicado por 2, a fim de considerar a ordem
das duas crianças que vão ocupar cada banco. Finalmente, multiplicar por
7! permuta circularmente os 8 grupos de duas crianças. Logo, chegamos a
2^1 • 28 • 7! = disposições. Como nossa roda-gigante apresenta 9 ban­
cos, basta que “encaixemos” o banco restante na roda-gigante de 8 bancos.
Para tanto, há 8 possibilidades (podemos “encaixá-lo ” entre quaisquer dois
bancos). Portanto, o resultado final do problema é • 8 = 16!.

79. Como BC mede 2 metros e cada degrau mede 25 centímetros, é claro que
a escada deve ter 8 degraus. Como o primeiro degrau começa em A e o
último termina em C, basta que decidamos os pontos em que começarão os
outros 7 degraus. Como estes devem ter largura múltipla de 40 centímetros,
podemos dividir o segmento AB em = 15 segmentos de 40 centímetros
cada um. Determinamos, então, 14 pontos no segmento AB (observe que
não contamos os pontos A e B, pois já sabemos que o primeiro degrau
deve começar em A e que o último não deve começar em C). Como temos
que construir mais 7 degraus, basta que selecionemos 7 desses 14 pontos,
determinando os tamanhos de cada degrau da escada e, por conseguinte, a
própria escada. Assim, a resposta é igual a (*? ) = 3.432 possíveis escadas.

80. No conjunto de 28 peças de dominó, existem 7 dominós duplos (a saber,


0-0, 1-1, ..., 6-6) e 21 não duplos. De fato, se um dominó não é duplo,
então há 7 possibilidades para sua primeira posição e 6 para a segunda.
Então, como a ordem dos valores em cada dominó não importa (isto é, o
dominó 1-3 é igual ao 3-1), temos = 21 dominós com lados distintos.
Assim, se a primeira escolha for um dos 7 dominós duplos, então, para
que a segunda escolha seja a de um dominó com lado igual a algum lado
do primeiro, temos 6 opções (por exemplo se escolhermos primeiro 2-2, a
segunda escolha poderá ser 0-2, 1-2, 2-3, 2-4, 2-5 ou 2-6). Por outro lado, se
o primeiro dominó for um dos 21 não duplos, então há 12 opções de escolha
para o segundo dominó (por exemplo, se escolhermos primeiro 4-6, então o
segundo dominó pode ser qualquer um dos outros que possuem lado 4 ou
94 Problemas Resolvidos de Combinatória

6). Portanto, são 7 • 6 4- 21 • 12 = 42 + 252 = 294 as possíveis escolhas cuja


ordem importa.
Se a ordem de cada escolha não importar, basta notar que, na contagem
realizada anteriormente, cada seleção é contada duas vezes. Logo, temos
= 147 possíveis escolhas nesse caso.
81. Associando a cada livro o dígito 1 caso ele seja selecionado, e o dígito 0
caso contrário, podemos relacionar a cada escolha de 6 livros dentre os 14
disponíveis, uma permutação de seis l’s e oito 0’s. Por exemplo, na per­
mutação 11001001010100, escolhem-se, na prateleira, os livros que ocupam
as posições 1, 2, 5, 8, 10 e 12 (observe, ainda, que tal permutação não deve
ser contada para a resolução do exercício, pois a ela associamos uma escolha
que contém os livros vizinhos 1 e 2). Então, a fim de que não sejam se­
lecionados livros adjacentes, tais permutações devem ter a particularidade
de que dois l’s não apareçam juntos. Logo, nosso problema se reduz a en­
contrar o número de permutações de seis l’s e oito 0’s tais que não haja
dois l’s juntos. Isso pode ser resolvido imaginando-se uma seqüência inicial
de oito 0’s e decidindo em que lugares os seis algarismos 1 serão colocados.
Isto é, podemos escrever a seqüência da seguinte forma:

00000000.

Escolhendo quaisquer 6 dos 9 espaços marcados por e preenchendo-os por


l’s, determinamos todas as possíveis seqüências que não contêm dois l’s
juntos. Logo, a resposta ao problema é (|) = 84.
Agora, no caso de haver n livros e desejarmos selecionar k, aplicando o
mesmo raciocínio anterior, torna-se coerente a condição n > 2k — 1 pois,
caso contrário, não é possível efetuar nenhuma escolha sem que ela contenha
dois livros vizinhos. Assim, de modo análogo ao caso numérico, temos que
contar o número de seqüências contendo n — k 0’s e k l’s tais que não
apareçam dígitos 1 adjacentes. Logo, (n- ; £+1) é o número de seleções neste
caso.

82. Fixando um dos cavaleiros da Távola Redonda, digamos, Sir Lancelot, pode­
mos reduzir nosso problema a outros em que os cavaleiros em questão podem
ser vistos como se estivessem em fila. Temos dois casos a considerar:
(a) seleções que contêm Sir Lancelot;
Os dois vizinhos imediatos de Sir Lancelot são-lhe considerados ini­
migos. Logo, para efetuarmos uma escolha compatível de cavaleiros,
devemos selecionar 4 dentre os 9 restantes com a restrição de que não
Parte II. Resoluções - Combinatória básica 95

selecionemos inimigos. Assim, considerando apenas esses 9 cavaleiros,


ficamos na mesma situação do Exercício 81, tendo os cavaleiros em
lugar dos livros. Isto é, considerando que 1 identifica um cavaleiro
selecionado e que 0 indica um não selecionado, estamos interessados
no número de seqüências compostas de quatro l’s e cinco 0’s tais que
não haja dois l’s juntos. Pelo Exercício 81, são (®) = 15 escolhas.
(b) seleções que não contêm Sir Lancelot.
Neste caso, basta que escolhamos os 5 cavaleiros dentre os 11 restantes.
Adotando, pois, o mesmo procedimento do item anterior, estamos in­
teressados no número de seqüências contendo cinco l’s e seis 0’s tais
que não haja dois l’s juntos. Assim, (J) = 21 é o número de escolhas.

Portanto, o número de escolhas dos 5 cavaleiros é igual a 15 + 21 = 36.


Na situação geral, podemos fazer a mesma divisão em casos que fizemos
acima, supondo que Sir Lancelot também faça parte das Távolas Redondas
generalizadas:

(a) seleções que contêm Sir Lancelot;


Uma vez escolhido Sir Lancelot, devemos desprezar seus dois vizinhos e
escolher k — 1 cavaleiros dentre os n—3 restantes. Queremos, portanto,
encontrar o número de seqüências que apresentam k — 1 dígitos 1 e
(n — 3) — (k — 1) = n — k — 2 dígitos 0 sem que apareçam dois l’s
juntos, número que sabemos ser igual a novamente Por um
resultado do Exercício 81.
(b) Seleções que não contêm Sir Lancelot.
Agora, temos que selecionar os k cavaleiros dentre os n — 1 restantes.
De modo análogo ao do item anterior, são (n^fc) escolhas possíveis.

Logo, o total de escolhas é igual a:


n—k—1 n—k n n—k
k— 1
+ k n—k k

83. Denotemos por c as pessoas que dispõem de 5 reais e por d as que dispõem de
10 reais. Sem = 2efc = 3, então somente as filas cdccd, cdcdc, ccdcd, ccddc
e cccdd são “boas”, sendo as 5 filas restantes consideradas “ruins” (a saber,
as filas ddccc, dcdcc, dccdc, dcccd e cddcc). Observe, então, que as filas
“boas” devem começar por c e, além disso, para cada ponto das mesmas,
o número de c’s já atendidos pelo caixa deve ser maior do que ou igual
ao número de d’s já atendidos. Em outras palavras, se considerarmos as
seqüências de m d's e k c’s até determinada posição, devemos ter um número
96 Problemas Resolvidos de Combinatória

de c’s maior do que ou igual ao número de d’s. Nisso consiste, portanto,


nosso problema: encontrar o número de seqüências que apresentam tal
característica. Destas observações decorre, ainda, que m < k é condição
necessária à existência de solução para o problema. Sabendo que o número
total de filas é vamos encontrar o número de filas “ruins” e, em
seguida, subtrair este valor do total de filas.
Considere, portanto, uma seqüência “ruim”. A pessoa para a qual não se
dispõe de troco é evidentemente representada por d, e antes dela deve haver
um número de d's igual ao número de c’s (digamos, igual a s), sendo a pessoa
que a precedeu também representada por d. Assim, a 2s + 1-ésima posição
é ocupada por d e as 2s posições precedentes por s d’s e s c’s. Vamos agora
colocar mais um c à frente da fila, em primeiro lugar (aos que protestarem
deve ser dito que isso facilitará o troco!). Como resultado, obtemos uma
permutação de m d’s e k 4- 1 c’s que começa por c e, dos seus 2s 4- 2 termos
iniciais, s 4- 1 são d’s e s + 1 são c’s. Em seguida, somente nessa seqüência
inicial, fazemos com que aqueles que possuem 10 reais troquem-nos com
aqueles que possuem 5 reais (é claro que isso vai desagradar àqueles que
possuíam mais dinheiro, mas precisamos fazê-lo para resolver o problema).
Considere, por exemplo, a seqüência abaixo:

ccdcdcddcdd ccdccd

O problema com o troco ocorre na posição marcada por d. Se colocar­


mos c defronte à seqüência e fizermos a troca mencionada, obtemos a nova
seqüência:

dddcdcdccdcc ccdccd

Como nas primeiras 2s + 2 posições o número de d's é igual ao número de


c’s, nosso procedimento não alterou tais quantidades. Portanto, obtivemos
uma nova seqüência de m d’s e k 4- 1 c’s começando por d. Em outras
palavras, nós associamos a cada seqüência “ruim” de m d's e k c’s uma
seqüência de m d’s e k 4-1 c’s começando por d.
Agora, vamos mostrar que, desta forma, obtemos todas as seqüências de m
d's e k 4- 1 c’s que começam por d. Para tanto, considere uma seqüência
desse tipo. Como m < k e a seqüência começa por d, deve haver algum
segmento da seqüência de letras que parta deste d e contenha um número
igual de d’s e c’s. Neste segmento, podemos trocar d’s por c’s e vice-versa,
descartando o c inicial em seguida. Então, aquele segmento passará a ter
um número de d’s maior do que o número de c’s, o que caracteriza uma
seqüência “ruim” de m d’s e k c’s.
Parte II. Resoluções - Combinatória básica 97

Portanto, o número de seqüências “ruins” de m d's e fc c’s é igual ao número


de seqüências de m d's e k 4- 1 c’s que começam por d. Logo, descartando
o d inicial, temos um total de seqüências “ruins”. Portanto, o
número de seqüências “boas” é igual a:
(m-l-A:)! (m4-fc)! k—m+1 +k
ml • kl (m- 1)1 ■ (fc + 1)! k 4-1 m

84. (a) A primeira criança pode ficar com um número de rosas variando de
0 a 12 (13 opções), um número de margaridas variando de 0 a 13 (14
opções) e um número de lírios variando de 0 a 10 (11 opções). Logo,
as crianças podem dividir as flores de 13 • 14 • 11 = 2.002 maneiras dis­
tintas. Observe que, distribuindo-se as flores para a primeira criança,
o número de flores da segunda fica automaticamente determinado.
Agora, impondo a condição de que cada criança fique com ao menos
três flores de cada tipo, temos 7 opções de distribuição para as rosas
(a primeira criança pode receber de 3 a 9 rosas), 8 opções para as mar­
garidas e 5 opções para os lírios. Assim, temos 7 • 8 • 5 = 280 opções
de distribuição nesse caso.
(b) Basta que generalizemos o raciocínio empregado no item anterior. Sem
restrições sobre as quantias de flores, temos (ni 4-1) (n? +1) • • • (n* 4-1)
distribuições possíveis. Com as restrições impostas sobre as quantias
de cada flor, temos (ni — 2si 4- l)(n2 — 2s2 4- 1) • • • (n* — 2sfc 4- 1)
distribuições. De fato, tomemos como exemplo a distribuição das flores
do tipo 1. Como cada criança deve receber pelo menos $i flores desse
tipo, a primeira criança deverá receber um número de flores variando
de si a m — $i. Logo, temos n\ — si — (sj — 1) = ni — 2si 4-1 opções
de distribuição para este tipo de flor. O mesmo raciocínio se aplica à
distribuição dos demais tipos de flores, validando a nossa fórmula.
85. Inicialmente, devemos decidir quantas bandeiras serão colocadas em cada
mastro, considerando-as, num primeiro momento, idênticas. Em seguida,
basta multiplicar o resultado encontrado por nl, arranjando as bandeiras nos
mastros. O número de maneiras de se determinar a quantidade de bandeiras
por mastro corresponde ao número de soluções inteiras não negativas da
equação xi 4------ FZfc = n (já que podemos ter mastros vazios), que sabemos
corresponder a (. n+fc-l . ’ Pe^° Exercício 25. Logo, a resposta ao problema é
k-1 1)
n^7
A (n+fc-1)!
) ~ (fc-l)l '
86. (a) Podemos proceder selecionando 2 pessoas dentre as 10 para ocuparem
o primeiro degrau, 2 dentre as 8 restantes para ocuparem o segundo,
98 Problemas Resolvidos de Combinatória

e, assim, sucessivamente. Assim, considerando a ordem das pessoas


situadas em cada degrau, temos um total de:

25(2°)©G)(2)G)= 3-628'800
maneiras distintas.
(b) Neste caso, podemos permutar os rapazes, um em cada degrau, de
5! maneiras, o mesmo valendo para a permutação das moças. Além
disso, devemos também considerar a ordem dos casais em cada degrau,
obtendo um total de 5! ■ 5! • 25 = 460.800 maneiras distintas.
87. (a) Basta que escolhamos quais vão ser as quatro pessoas que receberão,
cada uma, um prêmio, escolhendo qual prêmio cada uma ganhará em
seguida. Desse modo, (J) • 4! = 840 é a resposta desejada.
(b) Agora, basta que decidamos a quem cada prêmio será destinado. As­
sim, cada um dos quatro prêmios pode agraciar qualquer uma das sete
pessoas, donde são 74 = 2.401 as possíveis premiações.
(c) Temos 7 opções de escolha da pessoa que receberá o primeiro prêmio.
Uma vez escolhida, dâmo-la o prêmio, e obtemos um problema análogo
ao do item anterior: distribuir 3 prêmios dentre 6 pessoas, cada uma
podendo receber qualquer número de prêmios. Temos, desse modo,
7 • 63 = 1.512 possíveis premiações.
88. Dentre os m objetos iguais entre si, temos m+1 possíveis escolhas (podemos
tomar de 0 a m objetos). Analogamente, temos n + 1 opções de escolha para
os outros n objetos iguais entre si. Com relação a cada um dos p objetos
diferentes, temos duas opções para cada um deles: escolhê-lo, ou não o
escolher. Assim, temos 2P possíveis escolhas para tais objetos. Portanto,
lembrando de retirar o caso em que nenhum objeto é escolhido, obtemos
2p(m + l)(n + 1) — 1 coleções.
89. Primeiramente, observemos que todas as vezes que formamos a palavra
COMBINATÓRIA, finalizamos no A situado no canto inferior direito do
quadro exibido no enunciado. Assim, podemos analisar as construções de
maneira inversa, partindo desse A e voltando ao C inicial. Note, então, que,
antes desse A, pode ter vindo o I situado acima dele ou o I situado ao seu
lado esquerdo. Estando num dos I’s, notamos que também há duas possi­
bilidades para a letra R que o precedeu. Prosseguindo com este raciocínio,
como a palavra em questão possui 12 letras, tivemos 11 passos para voltar
do seu A final até o seu C inicial, donde se têm 211 = 2.048 possíveis
maneireis de se formar a palavra COMBINATÓRIA.
Parte II. Resoluções - Combinatória básica 99

90. Para resolvermos este problema, devemos notar que o número de 0’s em que
873! termina é igual ao número máximo p de fatores 10 que podem ocorrer
na decomposição 873! = N • 10**. Além disso, como IO? = 2P • 5**, e o número
de fatores 2 é maior do que o número de fatores 5 na decomposição de 873!
em números primos, temos que p é igual ao expoente de 5 na decomposição
em fatores primos do número em questão. Observe, então, que:

5 10 870
873! = 1 • • • 4 • (5*^7)-6 • • • 9 • (5*^2)-11 • • • 869 - (5 • 174)-871 • ■ • 873

= (1 • • •4-6---9-11 • • • 869 - 871 • • • 873)[(5 • 1) • (5 • 2) - - • (5 • 174)]


A
= A • 5174(1 ■ 2 • • • 174)
5 170
= A • 5174(1 • • ■ 4 • (5^7) -6 • • • 169 • (5^34)-171 •• - 174)
= A • 5174 (1 - - - 4 • 6 - -•169 - 171••• 174) [(5 - 1) • • • (5 ■ 34)]
B
= A-5174 • B • 534(1•2 • • • 34)
5 30
= AB ■ 5208(l • • • 4 • (õ^í) -6 • • • 29 ■ (5 • 6) -31 • ■ • 34)

= AB ■ 5208 (1 • • • 4 • 6 • • • 29 ■ 31 • • • 34)[(5 • 1) • • • (5 • 6)]


c
= AB-5 208 • c • 56(1 • 2 ■ • • 6)
= ABC-5i214(l-2-3-4-6)-5
D
= ABOD-5215

Assim, 873! = ABCD • 5215, sendo que em ABC D não existem fatores 5,
pela construção que fizemos. Logo, pelo esquema de resolução anterior­
mente sugerido, vem que p = 215, isto é, 873! termina em 215 0’s.

91. (a) Observe que, dadas as restrições do problema, os números que devemos
contar devem possuir 3, 5, 7 ou 9 algarismos. Só há um número
com 3 algarismos que contemple as condições impostas: 306. Com 5
algarismos, temos números da forma 3_0_6. Logo, temos 7 opções
para a segunda posição e 6 para a quarta, donde são 7 • 6 = 42 os
números nesse caso. Prosseguindo, temos 7 • 6 ■ 5 ■ 4 = 840 números
com 7 algarismos, e7-6-5-4-3-2 = 7! = 5.040 números com
100 Problemas Resolvidos de Combinatória

9 algarismos. Portanto, a resposta ao nosso problema é o total de


1 + 42 + 840 4- 5040 = 5.923 números.
(b) Para a primeira posição do número, temos 9 opções (não podemos uti­
lizar 0). Para a segunda, terceira e quarta posições, há 10 opções de
preenchimento. A quinta posição já é determinada e, para a última,
restam 9 opções (não podemos utilizar o dígito que utilizamos na
quarta posição). Logo, temos 9 • 103 ■ 1 • 9 = 81.000 números nas
condições do enunciado.

92. Observe que:

(n2)! = l2 • 2 • 3 • 22 • 5 • • • (n2 — 1) • n2
= (l2 • 22 • • • n2)[2 ■ 3 • 5 • • • (n2 — 1)]
= (1 • 2 • • • n)2[2 • 3 • 5 • • • (n2 — 1)]
= (n!)2[2 • 3 ■ 5 ■ • • (n2 — 1)].

Logo, segue do desenvolvimento acima que, para n > 3 (na verdade, para
n = 2 também), (n2)! > (n!)2.
Por outro lado, temos que:

(n!)2 = [1 • 2 • • • (n — 1) • n] [n • (n — 1) • • • 2 • 1]
= (1 • n) • [2 • (n - 1)] ■ • - [(n - 1) • 2] • (n • 1).

Agora, note que cada termo do produto acima é da forma (i + l)(n — i),
com i variando entre 0 e n — 1. Para i = 0 ou i = n - 1, tem-se claramente
que (t + l)(n — i) = n. Caso contrário, isto é, se 0 < i < n — 1, então
n — i > 1, donde i(n — í) > i. Portanto, ainda nesse caso, temos que
(i + l)(n — Í) = i(n — i) + (n — i) >i + n — i = n. Convém agora que
atentemos para uma sutileza do problema. Observe que, para n = 1 ou
para n = 2, (n!)2 = nn. No entanto, para n > 3, garantimos a existência
de i tal que 0 < i < n — 1, fato que nos garantirá a desigualdade que vem
a seguir:

(n!)2 = (1 • n) • [2 • (n — 1)] • • • [(n — 1) • 2] • (n • 1) n • • ■ n = n n,

donde segue que, para n > 3, (n!)2 > nn.


Portanto, anexando os resultados, temos, para n > 3, que:

(n2)! (n!)2 nn .
Parte II. Resoluções - Combinatória básica 101

93. (a) Para resolvermos este item, faremos uso da identidade explicitada na
sugestão. Assim, temos:

1 • 1! = 2! - 1!
2- 2! = 3! - 2!
3- 3! = 4! - 3!

n • n! = (n+ 1)! — nl.

Agora, somando membro a membro as igualdades, obtemos, de um


lado, a soma a ser calculada e, de outro, efetuando os cancelamentos
possíveis, o valor (n 4-1)1 — 1. Logo, i-i! = (n4-1)! — 1.
i-
(b) Nosso grande trunfo agora será o uso da identidade = i!1 (i+l)!'
Procedendo da mesma forma que no item anterior, temos:
1 _1 £
2! 1! 2!
2 1 1
3! 2! 3!
3 1 1
4! 3! 4!

n 1 _ 1
(n 4- 1)1 n! (n 4-1)!
Portanto, somando membro a membro as igualdades obtidas, e fazendo
os cancelamentos apropriados, vem que (i+í)! = ~ (n+i)!’

94. (a) Temos uma quantia de 6! = 720 números distintos de seis dígitos
cada. Dispondo todos os números, um abaixo do outro, podemos efe­
tuar a soma coluna a coluna. Em cada uma delas (unidades, dezenas,
centenas etc.), cada algarismo aparece tantas vezes quantas forem
as permutações dos algarismos das demais, ou seja, 5! = 120 vezes.
Portanto, em cada coluna, os algarismos que nela aparecem somam
i 120 • 1 4- 120 ■ 2 4- 120 • 3 4- 120 • 4 4- 120 - 5 4- 120 • 6 = 2.520. Con­
siderando, por fim, a ordem de grandeza de cada coluna, obtemos a
soma 2.520(1 4- 10 4- 100 4- 1.000 4- 10.000 4- 100.000) = 279.999.720,
que é o número procurado.
(b) Para resolvermos este problema, basta que contemos os números que
precedem o número dado. São eles, obviamente, os números de 5
102 Problemas Resolvidos de Combinatória

dígitos que começam por 1, 2, 31, 32, 34 e 3512. Relacionamos


abaixo, separadamente, cada um destes tipos de números, calculando,
também, o número de vezes em que cada um desses tipos ocorre:
• números que começam por 1: 4! = 24;
• números que começam por 2: 4! = 24;
• números que começam por 31: 3! = 6;
• números que começam por 32: 3! = 6;
• números que começam por 34-’ 3! = 6;
• números que começam por 3512: 1.
Portanto, são 24 + 24 4- 6 + 6 + 6 + 1 = 67 os números que precedem
35.142. Logo, este número ocupa a 68- posição.
(c) Este item possui resolução análoga à do item anterior. Basta, então,
que contemos as “palavras” que precedem, na ordem alfabética, a
“palavra” cafdeb. São elas as que possuem seis letras e começam
por a, b, cab, cad, cae, cafb e cafdb:
• “palavras” que começam por a: 5! = 120;
• “palavras” que começam por b: 5! = 120;
• “palavras” que começam por cab: 3! = 6;
• “palavras” que começam por cad: 3! = 6;
• “palavras” que começam por cae: 3! = 6;
• “palavras” que começam por cafb: 2! = 2;
• “palavras” que começam por cafdb: 1.
Assim, temos 120+120 + 6 + 6 + 6 + 2 + 1 = 261 “palavras” precedendo
cafdeb que, portanto, ocupa a 262- posição.

95. (a) Primeiro, devemos selecionar 4 objetos dentre os 16 iniciais. Feito


isto, restam-nos 12 objetos, dos quais devemos retirar mais 4 objetos.
Prosseguindo com este raciocínio, e observando que a cardinalidade dos
grupos é a mesma (todos possuem 4 objetos), temos como resposta:

16!
= 2.627.625.
4! (4!)5
A divisão da fração inicial por 4! tem por finalidade desprezai* a ordem
dos grupos, pois estes, segundo o enunciado, são indistinguíveis.
(b) Agora, devemos repetir o raciocínio empregado no item anterior, obser­
vando que temos três grupos com 2 objetos e dois grupos com 3 objetos.
Parte II. Resoluções - Combinatória básica 103

Logo, temos:

16!
= 252.252.000
3! • 2! (2!)4 • (3!)3 ■ 41

maneiras distintas de simplesmente separar os 16 objetos da forma


solicitada.

96. (a) Como a palavra em questão possui 10 letras distintas em sua com­
posição, o número de anagramas possíveis da palavra IMPORTUNAS
é 10! = 3.628.800.
(b) Uma vez que os anagramas a serem contados devem começar por IMP,
nesta ordem, restam 7 letras distintas a serem permutadas nas posições
seguintes. Logo, são 7! = 5.040 anagramas, neste caso.
(c) Agora além de permutarmos as restantes 7 letras, devemos permutar
as três do trecho IMP, obtendo um total de 3! • 7! = 30.240 anagramas.
(d) Agora, o trecho IMP não precisa estar necessariamente no início do
anagrama. Podemos, então, considerá-lo como um bloco além das 7
letras já existentes. Logo, temos 8! = 40.320 anagramas possíveis.
(e) As três primeiras letras devem ser IMP, nesta ordem, as três seguintes
devem ser ORT, em ordem aleatória, e as quatro últimas devem ser
UNAS, também em qualquer ordem. Portanto, o número de anagra­
mas, neste caso, é 3! • 4! = 144.

97. Podemos denotar os rapazes por ri, 7*3 e r4, e as moças por mi, m? e 7723.
Analogamente, cada cadeira vazia será denotada por v. Assim, interessa-
nos o número de permutações de ri^rzr^mirr^msvvvv nas quais os blocos
de cadeiras destinados aos rapazes e às moças permanecem intactos, embora
as posições de cada rapaz e de cada moça em cada um deles possa variar.
Assim, considerando as repetições existentes (lugares vazios), temos um
total de - -^j'3! = 4.320 disposições diferentes das pessoas.

98. (a) Para obedecer às condições do enunciado, também as mulheres não


deverão sentar-se juntas e, portanto, percorrendo-se a mesa num de­
terminado sentido (horário ou anti-horário), deveremos notar sempre
casais na ordem marido-mulher ou sempre na ordem mulher-marido.
Como são 7 os casais, devemos considerá-los como blocos únicos a
serem permutados circularmente em torno da mesa. Logo, observando
o que comentamos acima sobre a ordem dos casais, temos 2-6! = 1.440
maneiras distintas.
104 Problemas Resolvidos de Combinatória

(b) Vamos resolver este problema de duas maneiras distintas. Podemos


imaginar que há 12 possíveis bancos para um primeiro casal, 11 para
um segundo, e assim por diante, até restarem 7 bancos para um último
casal. Além disso, cada casal pode sentar de duas maneiras diferentes
em seu banco. Observamos agora que, uma vez que a roda-gigante
gira em torno de seu eixo, uma mesma configuração dos casais pode
ser vista por um observador externo de 12 maneiras diferentes. Logo,
cada configuração dos casais na roda-gigante está sendo contada 12
vezes. Portanto, o número de configurações distintas dos casais é igual
a 12 111?*8-7-26 =3.548.160.
Analogamente, podemos interpretar os casais como seis elementos dis­
tintos e os bancos vazios como seis elementos idênticos, de tal modo
que cada configuração dos casais na roda-gigante pode ser associada
a uma permutação de 12 elementos, sendo 6 idênticos. Nesse caso
também, é claro, devemos dividir o resultado encontrado por 12 e
multiplicá-lo por 26, devido à disposição de cada casal em seu banco.
Assim, obteremos o mesmo resultado, igual a ■ 26 = 3.548.160.
99. Suponhamos que as 14 pessoas já tenham sido convidadas. Sendo x o
número de homens convidados pelo marido, como ele deve ter convidado
7 pessoas, 7 — x é o número de mulheres por ele convidadas. Além disso,
como 7 é também o número total tanto de homens como de mulheres con­
vidados, sua esposa deve ter feito convite a 7 — x homens e a x mulheres.
Esquematicamente, temos:

homens: x
Convidados do marido
mulheres: 7 — x

homens: 7 — x
Convidados da esposa
mulheres: x.
Agora, para cada x (1 < x < 7), temos as seguintes escolhas a realizar:
(a) escolha dos x homens pelo marido: ;
(b) escolha das 7 — x mulheres pelo marido: (7 ®a.);
(c) escolha dos 7 — x homens pela esposa: (7 ;
(d) escolha das x mulheres pela esposa: (®).
Note que, se x = 0, então o marido deve ter convidado 7 mulheres e a
esposa 7 homens, o que é impossível. Da mesma forma, x = 8 conduz a
Parte II. Resoluções - Combinatória básica 105

uma contradição. Assim, fazendo x variar de 1 a 7, temos, como resposta


ao problema, o valor de:
7 2 2

é©U) =
-C)’©‘+C)‘G)‘+©’C)
2 2 2 2
4-
©x©s
= 3.427.776

possíveis convites.

100. (a) Empregaremos resolução semelhante à do item (a) do Exercício 50.


Tomando-se um conjunto de n objetos distintos, passaremos a con­
siderar a escolha de p objetos, com a particularidade de que um dos
objetos escolhidos seja tido como “especial”. Para tanto, podemos
selecionar os p objetos de (p) maneiras e, em seguida, selecionar o
objeto “especial” dentre eles de p maneiras. Isso corresponde ao lado
esquerdo da identidade. Analogamente, poderiamos escolher apenas
os p— 1 objetos “não especiais” de (p2i) maneiras, escolhendo o obje­
to “especial” dentre osn-(p-l) = n- p+ l restantes. Ora, isto é
justamente o que está sendo contado pelo lado direito da igualdade.
Portanto, como as escolhas são idênticas, isto é, só diferem em seus
procedimentos, a identidade está demonstrada.
(b) Primeiramente, note que podemos ‘enxergar’ a identidade do item an-
terior sob a aparência > / < = n — p 4- 1. Então, para cada i entre 0 e
kp-i)
n — 1, temos:

+ i)G7i)
(?)
tomando p = i 4- 1 nessa ‘nova’ identidade. Logo:
n—1 n—1
(»+i)g;,) n(n 4-1)
E
i=0
(")
= ^(n — i) = 14-24-------F n =
i=0
2

101. (a) Para que as simplesmente apareça, basta que escolhamos 7 elementos
dentre os 11 restantes. Logo, (l,1) = 330 é o número de vezes em que
106 Problemas Resolvidos de Combinatória

tal objeto aparece. Agora, vamos contar o número de vezes em que


a$ aparece e ocupa a terceira posição. Como as seqüências formadas
ocorrem em ordem crescente de índices, uma vez que as deve ocupar
a terceira posição, temos de selecionar 2 objetos dentre os 4 primeiros
(de ai a a^) para comporem as duas primeiras posições da seqüência,
e selecionar 5 objetos dentre os 7 últimos (de a& a (212) para comporem
as cinco últimas posições da seqüência. Logo, o número de seqüências
que apresentam as na terceira posição é (2) (5) = 126. Portanto, o
número de seqüências que contêm as em posição distinta da terceira é
330 - 126 = 204.
(b) Como no item (a), o total de seqüências que contêm a^ é igual a (pZj).
Além disso, o número de seqüências que contêm a^ na j-ésima posição
é (}Zi)(pZj)- Logo, o número de seqüências que contêm a^ fora da
j-ésima posição é gz}) - (‘ZÜÇZj).
(c)
Este caso encontra resposta na resolução do item anterior, resul­
tando (jZJ) (^j).
• i = j\
Nesse caso, os elementos da seqüência que precedem já estão
determinados (a saber, são os elementos ai, az, ■ ■ «í-i)- Assim,
basta que escolhamos os demais p — i elementos dentre os n — i
restantes, o que pode ser feito de maneiras diferentes.

Se i < j, então i — 1 < j — 1, ou seja, o número de objetos


candidatos a preceder ai é menor do que o número de lugares
a serem ocupados, o que nos mostra a impossibilidade de que o
elemento a, ocupe a J-ésima posição para i < j.
102. (a) Note que cada quatro vértices do polígono determinam um quadriláte­
ro, que possui duas diagonais, com exatamente uma intersecção, uma
vez que o polígono em questão é convexo. Considerando todas as
diagonais de todos os quadriláteros que podem ser formados com qua­
tro vértices do polígono, estaremos também considerando todas as
diagonais do polígono. Como são (”) os quadriláteros possíveis de
ser formados, teremos, no máximo, Q) intersecções das diagonais do
polígono (esse máximo ocorre no caso em que todas as intersecções
geradas pelas diagonais dos quadriláteros são distintas).
(b) Para melhor compreender este exercício, observe a figura que vem em
seguida:
Parte II. Resoluções - Combinatória básica 107

Sejam dados os pontos A, B, C e D, como na figura. Então, observe


que as retas formadas por esses pontos determinam 3 intersecções (Aí,
N e P). Além disso, observe que poderiam ter ocorrido menos inter­
secções. De fato, se o quadrilátero ABCD fosse um trapézio, então
seriam apenas duas as intersecções e, se ele fosse um paralelogramo,
teria ocorrido apenas uma intersecção. Portanto, de maneira análoga
ao item anterior, temos, nesse caso, no máximo 3(”) intersecções ao
todo.
103. Consideremos que x seja o número de pontos da reta r e y o número
de pontos da reta s. Para construirmos um quadrilátero, basta que se­
lecionemos dois pontos de uma das retas e dois pontos da outra. Como os
quadriláteros não precisam ser convexos, obtemos um total de 2 (2) (2) (o
problema também poderia ser visto como se estivéssemos considerando a
ordem de escolha dos dois pontos em uma das retas). Por outro lado, a fim
de formarmos um triângulo, podemos selecionar um pontos de r e dois de
s, ou selecionar dois ponto de r e um de s. Temos, portanto, x(^) + 3/(2)
triângulos possíveis de serem formados. Pelo enunciado, segue, então, que:

2(3 (I) 28 ,
+ y® 11
9t(j-1) y(y-l)
=> 2 2 - 28
x^^ + y^^- 11
(*-i)(y-i) = 28
0/ - 1) + (z - 1) ' 11
28
=> (rr - l)(j/ - 1) = — (x + y - 2). (1)

Observando, agora, que x + y = 13, substituindo este valor em (1), obtemos


108 Problemas Resolvidos de Combinatória

o seguinte sistema de equações lineares:

{x — l)(y — 1) = 28
(x - 1) + (y - 1) = 11.

Os únicos dois valores inteiros cujo produto é 28 e cuja soma é 11 são os


números 4 e 7. Como o número de pontos da reta r é maior do que o
número de pontos da reta s, segue que x > y, donde x — 1 = 7, ou seja,
x = 8, donde r possui 8 pontos e s, por conseguinte, possui 5 pontos.

104. Podemos supor a existência de 10 caixas, cada uma das quais contendo
algarismos idênticos entre si. Isto é, suponhamos que a caixa 0 contenha
apenas algarismos iguais a 0, a caixa 1 contenha apenas algarismos iguais
a 1, e, assim, sucessivamente. Suponhamos, ainda, que todas as caixas
contenham os algarismos correspondentes em quantidade ilimitada. Con­
sideremos que, da caixa 0, vamos retirar xq algarismos iguais a 0, da caixa
1, vamos retirar aq algarismos iguais a 1, e assim por diante. Como os
números em questão devem ter 7 algarismos, o número de soluções inteiras
não negativas da equação xq + rcj + • • • + xg = 7 é justamente o resultado
que procuramos, ou seja, o número de colunas escritas. Isso ocorre porque
cada coluna se caracteriza pela quantidade de dígitos de cada tipo existentes
em cada um de seus elementos, isto é, com uma solução da equação acima.
Portanto, retirando-se o caso em que são selecionados sete algarismos iguais
a 0, obtemos, como resposta ao problema, — ~ Cg ) — 1 ~ 11.439,
utilizando o Exercício 25.

105. (a) Trata-se de um problema trivial. Basta observarmos que, de cada um


dos n vértices, partem n — 3 diagonais (descontam-se de n o próprio
vértice e os dois a ele adjacentes). Repetindo-se o mesmo raciocínio
para todos os vértices, contamos duas vezes cada diagonal (dados dois
vértices A e B, contamos a diagonal que sai de A e termina em B e
a que sai de B e termina em A). Logo, o número de diagonais de um
polígono de n lados é -■
Como outra forma de resolução do problema, notemos que (£) ‘conta’
o número de maneiras de selecionarmos dois vértices quaisquer do
polígono e uni-los por um segmento de reta. Ademais, observe que,
dentre esses (£) segmentos, apenas n não devem ser considerados di­
agonais do polígono, uma vez que constituem os n lados da figura.
Portanto, o número de diagonais do polígono pode ser também calcu­
lado por (£) — n = n(n~3).
Parte II. Resoluções - Combinatória básica 109

(b) Este item possui resolução similar à do anterior. Tomando cada vértice
da base, podemos conectá-lo a cada um dos n vértices da face paralela
à base (a qual chamaremos “tampa” do prisma). Entretanto, três
dessas conexões não devem ser consideradas diagonais. De fato, cada
vértice da base é ponto de encontro de três arestas, sendo duas delas
pertencentes à base e uma delas perpendicular a esta, conectando a
base à “tampa”. Essas três arestas formam, pois, um triedro. Assim,
os três segmentos que partem do vértice em questão e terminam em
pontos de intersecção do triedro com a “tampa” (por sua vez, vértices
da “tampa”) não devem ser considerados diagonais do prisma. A figura
que vem na seqüência ilustra nossas considerações para um prisma
pentagonal:

c*
D’ B*

D B

E A

Figura 5: Na figura, as arestas AE, AA! e AB compõem o triedro que está em


destaque, hachurado. Os segmentos AE', A A' e AB', apesar de conectarem a
base do prisma à sua “tampa”, não devem ser considerados diagonais. Deste
modo, somente os segmentos AD' e AC são diagonais do prisma.

Assim, repetindo o raciocínio do caso geral para os n vértices da base,


concluímos que o prisma possui n(n — 3) diagonais.
Observe que este valor é o dobro do encontrado no item (a). Uma
das maneiras de se entender o porquê disto é realizarmos a projeção
das diagonais do prisma sobre sua base. Ao se fazer isso, cada dia­
gonal da base (um polígono de n lados) será obtida duas vezes, provin­
do de duas diagonais diferentes do prisma! Esta é, inclusive, uma outra
maneira de resolvermos aquele item.

106. Pela relação de Stifel-Pascal (vide parte (b) do Exercício 60), temos que
110 Problemas Resolvidos de Combinatória

(”) (fc+i) = (Z+i)> ^es<ie Que sejam obedecidas as condições de existência


necessárias. Além disso, temos:

(n + 1)! n+1 n! n + 1 (n\


(k + 1)! • (n — fc)! k+1 k\ • (n — fc)l k + l\k)

Logo:

K00][0+0]-[(
n
=n[0+(.
n-1 / I l\
k 4-1

= n (n j
=n>0 n—1 _ —1 /
n— \

k=o k=0 x '

-i—
(n + l)n
n!
00 ■■■(:)■

Neste desenvolvimento, utilizamos várias das propriedades do símbolo pro-


dutório19.

107. (a) Vamos demonstrar essa identidade de duas maneiras diferentes, ambas
muito elegantes: algebricamente e por argumentos combinatórios.

• Demonstração algébrica;
O Binômio de Newton nos garante que (a + ò)n.

19Para maiores esclarecimentos, consulte o Capítulo 1 de [13]


Parte II. Resoluções - Combinatória básica 111
I----- —-----------------------------------------------------------------

Logo, segue que:

(;)•(?)•-4:) = È(")
i=0 V 7

= è(?) r-i
i=0 X 7
= (l + l)n
= 2n.

• Demonstração por argumentos combinatórios.


Considere um conjunto de n elementos distintos. O total de sub­
conjuntos que podem ser obtidos a partir desses n elementos pode
ser contado de duas maneiras. Para contarmos todos os subcon­
juntos vazios, não devemos selecionar nenhum elemento de (£)
maneiras. Para contarmos os subconjuntos contendo um só ele­
mento, basta selecionarmos um dentre os n existentes de (”) ma­
neiras. Assim, sucessivamente, (q) 4- (7) 4- (2) + ’' ' + (n) refere-
se ao total de subconjuntos do conjunto dado. Analogamente,
poderiamos construir subconjuntos genéricos, partindo da seguinte
idéia: com relação a cada um dos n elementos, optamos por incluí-
lo ou não no subconjunto a ser formado. Logo, temos duas alter­
nativas para cada elemento e, portanto, 2n subconjuntos possíveis,
obtendo a identidade almejada.
(b) Através da relação de Stifel-Pascal (Exercício 60), podemos escrever:

k 4- 1 k 4-1 fc 4-2
4-
k k +1 k+1
k+2 k + 2' k 4~ 3
k
+ k +1 k 4" 1
k+3 fc +3 k + 4'
k
+ k 4- 1 k 4- 1

k+r k 4- r k 4- r 4- 1
k
+ k 4“ 1 k 4- 1

Somando, então, todas as igualdades obtidas membro a membro, fa­


zendo os devidos cancelamentos, e observando que (£+J) = (£), vem
112 Problemas Resolvidos de Combinatória

que:

k 4- r k -I- r + 1
k k 4- 1

como queríamos mostrar.


(c) Primeiro, observemos que, para cada p G N tal que p < n, temos
(p) = (n-p) (Exercício 60). Logo:

/ n\ /n+l\ Zn4-2\ n+r


W+( i M 2)+-+ r
= /n\/n+l\ n+2 n+r
4------ 1-
ynj \ n j n n
(n + r + 1\
\ n+1 J
(n 4- r 4-1\
=l r )’
sendo a penúltima passagem justificada pelo resultado do item ante­
rior, e a última, novamente, pelo Exercício 60.

108. Primeiramente, notemos que:

a
(A: - z) (k (k — i)!(a — k 4- i)l
k—i
a (q-i)!
(k — i — l)!(a — k 4- ?)!
a— 1 \
a k — i — 1) ’

de tal maneira que:

fc-i z
0-1
k — i — 1/
i=0 '

fa 4~ b —
a k— 1 )’
Parte II. Resoluções - Combinatória básica 113

utilizando, para a última igualdade, a Convolução de Vandermonde (Exer­


cício 71), considerando, naquele exercício, n = a + b — l,p = bem-k- 1.
Desenvolvendo, assim, o valor encontrado, temos:

a+b— 1 (a + b — 1)1
a a
k—1 (k — l)!(a + b — k)l
ak (a + 6)!
a + b kl(a + b — k)\
ak í a + b\
a + b\ k J

e, portanto, está provada a igualdade.

109. (a) Note que, para cada i E N tal que 1 < i < n, vale o fato de que
í(”) = ^iT(n-í)! = n(í-i)7(n-í)i ~ n(?-i) (este mesmo resultado se
encontra também no item (a) do Exercício 50). Logo:

-§•©n / 1\

1=1 ' '


n / 1 \
v—>f n — 1\
= n) . ,
1=1
v
x
- 1 z/
= n2n-1,

utilizando, na última passagem, o item (a) do Exercício 107.


114 Problemas Resolvidos de Combinatória

(b) Note que:

+ -" + (2n + l)Q) -

-g—>©
i=0
n

i=l \ / i=0 x 7
= 2 • n2n-1 + 2n
= (n+ l)2n,

utilizando, na penúltima passagem, o resultado do item anterior e


também o item (a) do Exercício 107.
n-l
(c) Como no item (a), utilizaremos que i(™) = n('.í-i ). Nossa soma se
reduz, desse modo, a:

-K3 n — 1\ n—1
+• ■•+("-1)-"(”) =

n—1
= 1•n 1 ) +2•n +3•n 4-----
2 3
n — 1\
4- (n — 1) • n
n — 1J
n— 1 n—1 n— 1 n—1
=n +2 +3
1 2 3 n—1
= n • (n - l)2<n-1>“1
= n(n- l)2n-2,

tendo sido utilizado, na penúltima passagem, o resultado do item (a),


tomando n — 1 em lugar de n.

110. (a) Suponhamos que xi, x^, ..., xn sejam as n variáveis que temos à
disposição. Um termo típico de um polinômio homogêneo de grau p
tem a forma sendo cuj 4- «2 + • • • + &n = p, já que
os termos devem ter grau idêntico. E fácil ver, então, que o número
de termos diferentes de um tal polinômio corresponde ao número de
Parte II. Resoluções - Combinatória básica 115

soluções inteiras não negativas da equação acima, pois os expoentes das


variáveis devem ter valores inteiros não negativos. Assim, o número
de termos, ou a dimensão do espaço em questão, corresponde ao valor
(er1 Ç1) = (n+p-1)> Pe^° Exercício 25.
(b) Adotando a mesma notação da resolução do item anterior, como os
polinômios precisam ser completos, o número de termos, ou a di­
mensão procurada, nesse caso, corresponde à quantidade de soluções
da equação ai 4- «2 4- • • • 4- an = i, para 0 < i < p. Logo, o valor
procurado é:
/O 4- n — 1
+
1 4- n — 1
4-
^2 4- n — 1
4-... + p + n — 1
\ n—1 n—1 n—1 n—1
(n — 1
“ \n - 1
4-
n x
+
n 4-1
+ ... + n 4- p — 1\
n—1 )
n — 1, n— 1
_ ín + p

pelo resultado do item (b) do Exercício 107, tomando k = n — 1 e


r = p.

111. Primeiramente, é necessário escolher quais serão as 8 pessoas que adentrarão


o elevador dentre as 10 disponíveis. É claro que isso pode ser feito de
(g°) maneiras diferentes. Agora, uma vez escolhidas tais pessoas, devemos
dividi-las em seis grupos, distinguindo, obviamente, a ordem dos mesmos, já
que os andares são distintos. Assim, basta escolhermos as duas pessoas que
descerão no primeiro andar de (f) maneiras, a que descerá no segundo de
maneiras (observe que já descontamos as que desceram no primeiro andar),
e, assim, sucessivamente. Portanto, temos (g0) (2) (i) (o) (i)(3) (i) = 151.200
maneiras de efetuar o transporte desejado.
112. Observe que, a fim de que a face i apareça i vezes para cada i entre 1 e 6,
devemos ter 14-24-34-44-54-6 = 21 lançamentos do dado, que é justamente
o número de lançamentos informado pelo enunciado. Considerando os valo­
res obtidos em cada lançamento como se fossem uma seqüência numérica,
para satisfazer o que se pede, basta que selecionemos as posições a serem
ocupadas por cada valor, pois cada uma delas se associa a um lançamento
diferente. Assim, como a face 1 deve aparecer uma vez, ela pode ocorrer
em (21) posições possíveis. Como a face 2 precisa aparecer duas vezes,
temos (22°) seleções possíveis, nesse caso. Prosseguindo com tal raciocínio,
(”)(?) (3 ) Cf) G,1)® = ■..21.3MW6I = 205.323.039.200 lançamentos
possíveis.
116 Problemas Resolvidos de Combinatória

113. (a) Como os temas são todos distintos, os k grupos devem ser também
considerados distintos. Assim, a partir dos kr alunos originais, pode­
mos escolher os r responsáveis pelo primeiro tema de (^r) maneiras
diferentes, os r responsáveis pelo segundo tema de (fcr~r) =
maneiras distintas. Prosseguindo com esse raciocínio, os k grupos
poderão ser formados de:
(kr^ Ç(k — l)r ) Ç(k - 2)r) Ç [k — (k — l)]r
r
(fcr)!
________ [(fe ~ lk]! [(fc-2)r]l r!
r![(fc — l)r]! r![(fc — 2)r]! r![(fc —3)r]! r!0!
(fcr)!
(r!)fc
maneiras distintas.
(b) Agora, os grupos devem ser considerados indistinguíveis, pelo fato de
todos possuírem o mesmo tema e a mesma cardinalidade. Logo, es­
colhido um tema dentre os k existentes, é suficiente que dividamos o
resultado encontrado no item anterior por fc!, visando a desprezar a
ordem dos grupos. Assim:
(fcr)! (fcr)!
k-
fc!(r!)fc (fc — l)!(r!)*
é o número de maneiras de se realizar a divisão dos grupos, nesse caso.
114. (a) Entre Paulo e Marta, deve haver 3 pessoas. Contando, além delas,
Paulo e Marta, temos um bloco composto de 5 pessoas. Como a fila
possui 8 posições, esse bloco pode ocupar 4 posições diferentes. Além
disso, Marta e Paulo podem, a cada permutação, trocar de posição, o
que nos impele a multiplicar o resultado final por 2. Finalmente, as
6 posições restantes podem ser preenchidas de 6! maneiras distintas.
Portanto, a resposta ao problema é 2 • 4 • 6! = 5.760.
(b) Agora, como as 8 pessoas devem sentar-se numa mesa redonda, haverá
três pessoas entre Paulo e Marta nos dois sentidos, a saber, horário e
anti-horário. Fixando, então, Paulo e Marta em duas cadeiras (note
que todas as maneiras de se fazer isso são equivalentes, uma vez que a
mesa é redonda), sobram-nos 6 cadeiras, as quais podem ser ocupadas
pelas 6 pessoas restantes de 6! = 720 maneiras, resultado do problema.
Analogamente, poderiamos, em primeiro lugar, permutar 6 pessoas
circularmente (excetuando-se as duas em destaque) de (6 — 1)! = 5!
Parte II. Resoluções - Combinatória básica 117

maneiras e, em seguida, assentar Paulo e Marta na mesa. Temos 6


opções para o lugar em que Paulo se assentará, ficando o lugar de
Marta estabelecido de modo a ficarem três cadeiras entre as duas pes­
soas em destaque. Logo, 6 • 5! = 6! = 720 é o resultado procurado,
coincidindo com o anteriormente encontrado.

115. Nossa tarefa pode ser facilitada sobremaneira se dividirmos o problema em


casos. Observe que, como só estamos interessados na divisão das pessoas
em grupos, basta que definamos os integrantes de um deles. Assim, para
satisfazer as condições do problema, temos as seguintes possibilidades:

(a) um grupo apresenta 1 menino e uma menina;


Temos 8 escolhas possíveis de menino e 8 escolhas possíveis de menina.
Fazendo isso, os integrantes do outro grupo ficam também determina­
dos. Temos, portanto, 8 • 8 = 64 possibilidades neste caso.
(b) um grupo apresenta 1 menino e 3 meninas;
São 8 as escolhas possíveis do menino e (|) as das meninas, donde
8(|) = 448 possibilidades neste caso.
(c) um grupo apresenta 1 menino e 5 meninas;
A partir deste item, faremos apenas as contas, repetindo o raciocínio
anterior. Logo, temos 8 (|) = 448 opções.
(d) um grupo apresenta 1 menino e 7 meninas;
8(?) = 64.
(e) um grupo apresenta 3 meninos e uma menina;
©'8 = 448.
(f) um grupo apresenta 3 meninos e 3 meninas;
@© =3.136.
(g) um grupo apresenta 3 meninos e 5 meninas;
©©=3-136.
(h) um grupo apresenta 3 meninos e 1 meninas.
©(?)=448.
Observe que os grupos de 5 meninos estão sendo contados nos casos (e), (f),
(g) e (h), e os grupos contendo 7 meninos estão sendo contados nos casos
(a), (b), (c) e (d). Isso ocorre porque, para cada escolha realizada acima,
um outro grupo é formado, composto das pessoas que restam. Assim, a
resposta ao nosso problema é igual à soma dos valores encontrados nos 8
itens acima, ou seja, 8.192.
118 Problemas Resolvidos de Combinatória

Agora, se os grupos formados contiverem o mesmo número de integrantes


(no caso, 8 cada um), basta que somemos apenas os valores encontrados
nos casos (d) e (g), obtendo, assim, 3.200 como resposta.
116. Note, primeiramente, que, a fim de satisfazer as condições de distribuição
das cartas, seu número deve ser 14-2 + 3 + 4 + 5 = 15, correspondendo, de
fato, ao número de cartas fornecido pelo enunciado. Assim, para a caixa
1, podemos selecionar uma carta de (115) maneiras. Para ocupar a caixa
2, duas cartas podem ser selecionadas de (^) maneiras. Assim, suces­
sivamente, como as caixas são distintas, há (115) (^) (32) (®) (|) = 37.837.800
maneiras distintas de distribuirmos as cartas.
117. Este exercício tem solução assaz elegante e curiosa. Imaginemos que a
partição já tenha sido realizada, isto é, em vez de observarmos um con­
junto de n elementos, consideremos n conjuntos unitários cuja união re­
sulta no conjunto original. Seja A = {xi, x%,..., xn} o conjunto dado e
P^ = {{xi}, {X2},..., {xn}} a partição obtida deste conjunto. Como a
partição já foi realizada, analisaremos como isso ocorreu. No passo imedi­
atamente anterior à obtenção de P^, dois dos seus elementos ainda estavam
“fundidos”20 num único conjunto. Digamos que {xj} e {xj} estivessem na
forma {xf, x7}, que seria o último subconjunto ainda não particionado de A,
com z, j G {1,2,..., n}. As diversas possibilidades de obtermos x< e Xj são,
pois, em número de (2). Agora, denotemos Xj0 e xJO esses dois elementos
selecionados. Então, a essa altura, temos obtida a “partição” P^ igual a:

+70}’ í3'! }»•••! {^íq —1}> {*£ío+l}’ • • • > {■Ejo—1}’ {a'jo+l}> • • • >

Agora, ocorre que, na etapa anterior à da obtenção de P^\ dois dos seus
elementos (os quais são subconjuntos de A) deviam estar “fundidos”. Ora,
como P^ possui n — 1 elementos, o número de possibilidades para que
isso tivesse ocorrido é A seguir, obtivemos um outro conjunto P^
contendo n — 2 elementos. No passo anterior, dois deles haviam de estar
“fundidos”, o que pôde ocorrer de (n22) maneiras diferentes. Portanto,
prosseguindo com esse raciocínio até 2\ contendo apenas dois subcon­
juntos de A (resultado da primeira partição realizada), poderiamos obter
P^ de (2) = 1 maneira. Assim, pelo processo inverso ao do particiona-
mento, chegamos ao conjunto P^ l\ cujo único elemento é o conjunto
A. Portanto, uma vez tendo analisado todas as possibilidades disso ter
20Consideraremos, nesta resolução, que a “fusão” é o processo inverso da partição.
Parte II. Resoluções - Combinatória básica 119

sido feito, e, como todas as etapas do processo de particionamento são re­


versíveis, o Princípio Multiplicativo nos garante que o resultado procurado
é igual a:

n\ (n — 1)! (n — 2)! 3! 2!
” 2!(n- 2)! ’ 2!(n —3)! ’ 2!(n - 4)! 2! • 1! 2! 0!
n!(n — 1)1
— 2n-1 ’

correspondendo àquele que nos determinamos a mostrar.

118. (a) Vamos considerar n tipos de objetos separados em n caixas distin­


tas contendo, cada uma, pelo menos dois objetos idênticos. Suponha
que desejamos selecionar k 4-1 objetos destas caixas em duas etapas:
primeiro selecionar k distintos e depois selecionar mais um dentre os
restantes. Podemos, assim, selecionar inicialmente k objetos distintos
de (£) maneiras e, em seguida, selecionar mais um objeto de um dos n
tipos disponíveis. Logo, n(£) conta as seleções possíveis dos k 4- 1 ob­
jetos. Analogamente, poderiamos dividir tais seleções em dois grupos:
o daquelas em que o último objeto selecionado é uma repetição de um
dos k selecionados anterior mente (neste caso, as seleções constariam
da repetição de um único objeto) e o daquelas em que ele é distinto
de cada um dos k selecionados anteriormente (neste caso, a seleção
constaria de k 4- 1 objetos distintos). Com relação ao primeiro grupo,
seus elementos são em número de Zc(£). De fato, basta selecionarmos
um dos k tipos de objetos já selecionados. Para o segundo grupo,
podemos optar por qualquer um dos n — k objetos restantes, obtendo:

n!
(n — Zc)
fc!(n — &)!
n!
= (k + 1)
(Zc 4- l)!(n — k — 1)!
/ n \
= (fc+1) U +1/

possibilidades. Assim, as mesmas seleções dos k +1 objetos podem ser


realizadas de (k + l)(fc”1) + Zc(£), provando o que queríamos.
Agora, como demonstração algébrica, temos, partindo do lado direito
120 Problemas Resolvidos de Combinatória

da identidade, que:
7l!
(fc+1) k + l)+k\k) + + l)!(n - fc - íj!
n\
+k
k\(n — k)\
(n — k)n\ k • n!
+
kl(n — fc)! fc!(n — fc)!
(n — fc)n! 4- k • n!
fc!(n — fc)!
n\
n k\{n — k)\
- ■(:)

(b) Sem perda de generalidade, podemos supor m = k. Para melhor


entendimento do problema, vamos expandir, parcialmente, o somatório
dado:

§C)(*T n v
J \k + s- jj
(;)(*:•
K:M)( / n
k+s—
) \k + ns — 1
1}( + ...

J \k + s — kj'
Como na resolução do item anterior, vamos considerar n caixas distin­
tas contendo, cada uma, pelo menos dois objetos idênticos e nenhum
diferente destes e, ademais, que caixas distintas possuam objetos dis­
tintos. Agora, desejamos selecionar s + k objetos dessas caixas em
duas etapas: primeiro, selecionar s distintos entre si e depois mais k
distintos entre si, mas não necessariamente distintos dos s já escolhi­
dos. O número de maneiras de se fazerem tais seleções é igual a (”) (£).
Analogamente, podemos dividir a mesma contagem em diversos ca­
sos. Primeiro, podemos selecionar k + s objetos distintos dentre os n
tipos existentes de maneiras e, em seguida, podemos selecionar
s objetos dentre os k + s já selecionados de (fc^s) maneiras (estes
corresponderíam à primeira etapa da escolha e os k restantes à se­
gunda etapa). Assim, (q) (fc^s) (fc”s) conta o número de seleções em
Parte II. Resoluções - Combinatória básica 121

que são selecionados k + s objetos distintos. De modo semelhante,


(;)(‘+r‘)(,.
fc+”_i) conta o número de seleções de k + s objetos dentre
n de modo que haja repetição de um único tipo de objeto. Em geral,
©c‘+
: D(.jt+s-j) conta ° número de seleções em que há repetição de
exatamente j tipos de objetos. Procedendo desta forma, fica claro que
j deve ir somente até k. Assim, conta o número de escolhas
de k + s — j objetos dentre os n distintos, escolhe quais os s
objetos selecionados na primeira etapa dentre os k + s —j, e Q) escolhe
quais os j tipos de objetos que serão repetidos na segunda etapa. Por
essa razão é que se tomam, inicialmente, apenas k + s—j objetos: k—j
são distintos dos s já escolhidos e cada um dos j restantes é idêntico
a um dos s já escolhidos. Portanto, segue o resultado.
Agora, como demonstração algébrica, temos:

00 - oc s + (n — s)
k

■ 0 §(■)(:: 3
■ §(:)(:©©■ (2)

utilizando, na penúltima passagem, o resultado provado no Exercício


71, tomando n = n, m = k e p = s. Agora, note que:

ín\ ín — s\ n! (n — s)!
s!(n —s)! (k — j)!(n — k — s + j)l
n! (k + s — j)!
(k + s — j)l(n — k — s + f)l sl(k — j)\
n
k+s—

Assim, substituindo este valor na expressão (2), temos que:

(:)(:) -
§(■k + ns-jjV'\

- £©(' n i
k + s - jj ’
122 Problemas Resolvidos de Combinatória

como queríamos mostrar. Com relação ao fato de ter aparecido k em


lugar de m = min{k, s}, podemos justificá-lo observando que, para
p > n, (p = 0. Desse modo, se k > s, então poderiamos restringir o
somatório anterior, fazendo j variar apenas de 0 a s.
119. (a) Podemos provar a identidade solicitada expandindo os coeficientes bi-
nomiais que nela aparecem, constatando a identidade de modo trivial.
Todavia, existe uma solução mais simples, baseada na observação de
que n = . Desse modo, temos:

/n\ k(n 4- 1) 4-4- (n


k(n4-1) k) /n\
(n—— k)
n\k) ~ kk ++ i1 vJ
k(n 4-1)
k 4-1
k(n 4-1)
©<■
n\
n—k
©
k 4- 1
n—k n!
k 4-1
+
k\(n — fc)! k 4- 1 k\(n — k}\
(n 4-1)! n\
k
(fc 4-1)!(n — k)l
+ (k 4- l)!(n — k — 1)1
(n+ 1 n \
k l fc 4- 1 4- k 4-1)

(b) Para demonstrarmos esta identidade, observemos que:

0© ■ C)(‘"’©A
- oècr
- g ©(”©)(/<)■ (3)

utilizando, na penúltima passagem, o resultado do Exercício 71, para


n = n, m=kep=n— s. Agora, notemos que:
ín\ín — s\ _ n! (n — s)!
\s) \ j J sl(n-s)!
_ n! (s 4-j)!
(s 4- j)!(n - s - j)! j!s!
_ / n \ /s + A
V + 3/\ 3 )'
Parte II. Resoluções - Combinatória básica 123

Logo, substituindo esta expressão em (3), resulta que:

(:)(:)
S + j) \ j J\k- j)

= v (s+J; V 5 V n (4)

Agora, temos que:

s+jV s
. j J\k~ j) = (s + j)!
j!s!
s!
(k - j)!(s - k + j)!
(s + j)! kl
à:!(S + j-T)! jl(k-j)l

Portanto, substituindo esta última expressão em (4), e observando


que (í) (fc-i) (novamente pela identidade do Exercício
71, tomando n = s + j, m = k e p = s), obtemos, então, que:

(:)© - k

- g©g(*)(z (5)

Através do seguinte desenvolvimento, temos, ainda, que:

©(.'-<) = kl____________ jl________


jl(k-f)l (fc - i)!(j - k + i)l
A:! ________ í!________
il{k — i)l (k — f)l(i — k + j)l

- ©(©,)
124 Problemas Resolvidos de Combinatória

Finalmente, substituindo esta expressão em (5), obtemos:

0© - £©£©(,!,)(.«)
i y n \
- S0©2( k-jj \s+jj

- £ (•) © GVÍ'
como queríamos demonstrar. Na última das passagens acima, uti­
lizamos, novamente, o Exercício 71, para n = n + i, m = s + kep = n.
Com relação ao fato de, na demonstração anterior, ter aparecido, como
limite superior do somatório, k em lugar de m, podemos justificá-lo
da mesma forma que fizemos no item (b) do Exercício 118.

120. (a) Como pode haver prateleira vazia, o número de livros em cada uma
corresponde ao número de soluções inteiras não negativas da equação
X\ +%2 + + ^4 + X5 = 25, que corresponde a = (24)- Além
disso, para cada uma dessas divisões, podemos permutar os livros de
25! maneiras diferentes, uma vez que devemos considerar a sua ordem
em cada uma delas. Logo, 25! • (249) = número da ordem de 1029,
é a quantidade de divisões possíveis dos livros nas prateleiras.
(b) Considerando as 23 letras do nosso alfabeto escritas em seqüência,
como não pode haver palavras “vazias”, temos 22 espaços entre as
letras para dispor 4 divisões, que determinarão, obviamente, 5 palavras
distintas. Assim, (22) escolhe tais espaços. Uma vez fixados tais espa­
ços, podemos permutar as letras de 23! maneiras diferentes, e temos
também que desprezar a ordem das palavras formadas, dividindo o
resultado final por 5!. Assim, a resposta é ^r(242), número da ordem
de 1024.

121. (a) Para resolver este item, consideremos todas as m-combinações com
repetições de p diferentes moedas21 e n—p diferentes notas de dinheiro.
Além disso, suponhamos que cada moeda e que cada nota de dinheiro
21 Uma m-combinação com repetições de p diferentes objetos corresponde a uma seleção de m
objetos a partir de p tipos distintos, de tal maneira que cada tipo de objeto possa ser tomado
mais de uma vez. Pelo Exercício 25, ainda, segue que o total dessas m-combinações corresponde
ao número de soluções inteiras não negativas de xi + • • • + xp = m, ou seja,
Parte II. Resoluções - Combinatória básica 125

exista em quantidade ilimitada. Podemos separar tais combinações


em classes, de tal forma que a A:-ésima classe seja aquela que contenha
exatamente k moedas e m — k notas de dinheiro. O número de m-
combinações da classe k é, pois, (k1 ) • Variando k de
0 a 77i, temos que o total de m-combinações é, por um lado:

m+n—p— m+n—p—2
. n-p-1 _
+ •••
. n-p-1
m + p — 1\ ín-p- 1\
+ J \n ~ P - V ’
. P~ 1
isto é, o segundo membro da identidade em questão. Por outro lado,
sabemos que o total de m-combinações com repetições de p moedas
e de ?i - p notas de dinheiro é igual ao total de m-combinações com
repetições de n elementos distintos, a saber, Logo, o resul­
tado está demonstrado.
(b) No resultado do item anterior, basta substituir p por p+1, n por n + 2,
e m por m — n. A identidade, então, segue imediatamente.

122. Sendo ai o termo inicial das progressões geométricas em questão e q suas


respectivas razões, os trios de números devem ser da forma (ai,aiç,aiQ2),
com ai,q G N. Devido à restrição do problema e, como a^q2 é o maior dos
termos da cada trio, segue que aiç2 < 125, ouüi < Como ai eNe y
nem sempre satisfaz tal condição, ai pode assumir qualquer valor inteiro
compreendido entre 1 e j > assumindo, portanto, j valores distintos,
para cada q considerado. Segue, então, também do fato de ai ser natural,
que q deve variar de 1 a 11. Logo, o número de trios procurado é igual a:
li

E .91252.
9=1
= 125 + 31 + 13 + 7 + 5 + 3 + 2 + 1 + 1 + 1 + 1 = 190.

123. (a) Cada grupo de 6 compatriotas pode ser permutado de 6! maneiras


diferentes. Podemos, então, considerar a permutação dos n blocos
de compatriotas contendo, cada um, 6 pessoas. Assim, a resposta é
(6!)n -n! = 720n -n!.
(b) Agora, ambos os blocos devem ser compostos de 3 compatriotas. Ora,
o número de maneiras de se dividirem os compatriotas em 2 blocos de
mesma cardinalidade é | (®) (J) = 10, e cada um dos 2 blocos pode ser
permutado de 3! = 6 maneiras. Logo, 10 ■ 62 = 360 é o número de
126 Problemas Resolvidos de Combinatória

maneiras de se dividirem os compatriotas, considerando suas posições


na fila. Assim, como dispomos de 2n blocos ao todo, temos como
resposta 360n(2n)l possíveis filas.
124. (a) Para cada p compreendido entre 0 e n, podemos selecionar p objetos
idênticos de uma só maneira e n — p objetos distintos de (n”p) = (”)
maneiras. Assim, utilizando o resultado do Exercício 107, vem que
E£=o Ç) . = 2" é o número de maneiras de se realizar a escolha re-
querida.
(b) Para cada p compreendido entre 0 e n, podemos selecionar p objetos
do primeiro tipo e n—p objetos do segundo tipo. Como p pode assumir
n+l valores distintos, esta vem a ser, também, a resposta do problema.
(c) Para cada escolha realizada no item anterior, podemos permutar seus
n elementos de p!(”lp)i = (p) maneiras. Assim, ^2p=o (p) = 2n é °
número possível de ordenações. De fato, este simples resultado pode
ser obtido de outra maneira: considerando n posições a serem preenchi­
das por n objetos, há duas opções para cada posição, preencher com
um objeto do primeiro tipo ou com um do segundo. Assim, 2n é o
número de ordenações dos n objetos escolhidos.
125. Por um lado, o número 1 deve estar situado na primeira ou na última coluna,
pois se ele estiver numa coluna intermediária, não poderia ser formada uma
progressão aritmética na linha em que ele estiver. Por argumento seme­
lhante, podemos ver que 1 também deve ocupar a primeira ou a última
linha. Portanto, 1 deve ocupar um dos 4 cantos do retângulo quadriculado.
Suponhamos que 1 ocupe o quadrado da primeira linha e da primeira coluna.
Desconsiderando esta linha e esta coluna, o número 2 não pode ser colocado
em nenhuma posição. Logo, este número também deve ocupar a primeira
coluna ou a primeira linha. Ora, como 1 já ocupa a primeira posição, 2
deve ocupar a segunda posição da primeira linha ou a segunda posição da
primeira coluna. Suponhamos que 2 ocupe a segunda posição da primeira
linha. Então, esta já é determinada: 1,2, ...,n, nesta ordem. Feito isso,
n+l deve ser disposto logo abaixo do número 1, n + 2 deve ser disposto
abaixo do número 2, e, assim, por diante, determinando a posição de todos
os números no retângulo. Como pudemos notar, as diferentes configurações
dependem apenas do local em que situamos o número 1 e do local em que
situamos o número 2, após tê-lo feito para 1. Logo, 4 • 2 = 8 é o número de
configurações possíveis.
126. O número de maneiras de se escolherem n maçãs dentre as 2n + r exis­
tentes é (2nnbr) e o número de maneiras de se escolherem n pêras dentre as
Parte II. Resoluções - Combinatória básica 127

2n — r existentes é (’2n—r' Assim, podemos escolher 72 maçãs e 72 pêras de


* n ‘

(2T)C 2n~r) maneiras. Desenvolvendo tal expressão, temos:

'2n + r\ /2n — r\
72 J \ 72 J
(2n4-r)! (2n —r)!
71!(t2 4”7")! 721(77.-7')!
(2n 4- r)!(2n — r)!(2n)(2n — !)••• (2?2 — r 4- 1)72(72 — 1) • • • (72 — r 4- 1)
n!(72 4- r)!72!(?2 — r)l(2n)(2n — 1) • • • (2t2 — r 4- 1)72(72 — 1) • • • (n — r 4-1)
(2?2 4- r)(272 4- r — 1) • • • (2tt 4- l)(272)!(272)!n(?2 - 1) • • • (72 - r 4- 1)
71!(72 4- r)(n 4- r — 1) ■ • • (72 4- l)72!n!72!(272)(272 — !)••• (2n — r 4-1)
[(2t2)!]2 272 4-t" 2714-7" — 1 +1 n
2n 4-1 n—1
72 n — r+1
(6)
(n!)4 n+r n+r—1 n 4-1 2n 2n — 1
tz 2n — r + 1

Agora, observemos que, para cada k entre 1 e r — 1, vale a desigualdade:

2n + k 2n — k
<
n+k n—k '

De fato, esta desigualdade é válida se, e somente se:

(2n + tyÇn — k) < (2n — fc) (n 4- fc) <=>


<=> 2n2 — 2nk + nk — k2 < 2n2 4- 2,nk — nk — k.2
<=> — nk < nk,

o que é sempre verdade. Logo, a desigualdade é verdadeira para cada k


pertencente ao intervalo considerado. Assim, utilizando esta propriedade
em (6), temos:

2n 4- r
n
[(2r»)!]2 2n 4- r 2n — r 4- 1 272 -1 n n—1 n — r 4-1
<
(n!)4 n+r n — r 4- 1 72—1 2n 272 -1 2n — r 4-1
((2n)!]a 2n 4- r n
(n!)- n+r 2n

<
[(2n)f 2n n
(n!)4 72 272

=C)’
128 Problemas Resolvidos de Combinatória

Na última desigualdade, utilizamos que 22±r < ^i. De fato, isto vale se, e
somente se, (2n + r)n < (n + r)(2n) <=> 2n2 + nr < 2n2 + 2nr <=> 0 < nr, o
que é sempre verdade. Logo, segue que:

/2n + r\ Í2n — r\ < /2n\ 2


\ n / \ n J ~ \n/
Portanto, o valor máximo para o número de maneiras de se realizar a escolha
em questão ocorre ao se tomar r = 0.
127. Apesar de aparentemente complicado, este exercício possui solução muito
simples. Observemos, primeiramente, que temos seis possibilidades distin­
tas de resultado para cada um dos n dados. Uma vez que os dados são
idênticos e lançados simultaneamente, importa, para nossa solução, apenas
o número de vezes em que cada uma das faces aparece nos lançamentos.
Assim, a resposta ao problema é o número de soluções inteiras não negativas
da equação xi + x% 4------------ = n, que sabemos ser igual a
Desenvolvendo, pois, este número, temos:
) = m-
n+5 (n + 5)!
5 n!5!
(n + 5)(n + 4)(n + 3)(n + 2)(n + 1)
5-4-3-2- 1
n 4- 1 n + 2 n + 3 n + 4 n + 5
4 5
n
5
que é justamente o resultado por que procurávamos.
128. A prova será feita por indução sobre N. Para N = 0, basta que, para cada
k, tome-se Xi = i — 1 para 1 < i < k. Nesse caso, temos:

N
\kJ \k — 1/ (?)
(k- ! fc-2
\ k
+ k—1
+■"+(?)
= 0 + 0 +•••+ 0
= 0,

sendo tal representação única, dado que > 0, para 1 < i < k, e
0 < xi < x2 < ■ • . < xk.
Parte II. Resoluções - Combinatória básica 129

Agora, como hipótese de indução, suponhamos que, para cada l G N, todo


inteiro não negativo menor do que N possua representação única na forma
considerada. Queremos provar que N também possui tal representação. E
trivial o fato de existir um único x € N satisfazendo:
x 4-1
k

Portanto, subtraindo (£) de cada um dos membros das desigualdades acima,


vem que:

utilizando também a relação provada no Exercício 60. Agora, se (£) = 0,


então x < k, donde x + 1 < k. Ora, nesse caso, 0 < N < (^J1) < (£) = 1,
donde N = 0, que já sabemos possuir a representação única considerada.
Por outro lado, se Q) > 0, então N — (£) é um inteiro não negativo e menor
do que N. Por hipótese de indução, este inteiro deve possuir representação
única para cada l E N. Tomando, então, l = k—1, a hipótese de indução nos
assegura a existência de únicos xi,..., x^-i E N U {0} sujeitos à condição
0 < a?i < ... < e:

(x\ = (Xk~1
N = (Í} \k k—~\}
ykj + (k
N - \k) \k — 1 1/

Assim, tomando = x, temos a representação de N que procuravamos,


sendo sua unicidade decorrente das de a?i, ..., Xk-i e x^.

129. Aparentemente difícil, este exercício admite resolução muito simples. De


fato, basta que, para cada x E X, escolhamos o menor índice i, 1 < i < k,
tal que x E Si. Nesse caso, x E Sj para todo j entre i e k. Então,
incluindo-se também o caso em que não existe o menor índice i (ocorre se x
não pertencer à seqüência de subconjuntos), temos k + 1 possíveis escolhas
para cada x E X. Como |X| = n, segue que o número de seqüências é
(fc + l)n.
130 Problemas Resolvidos de Combinatória

Princípio da Inclusão e Exclusão

130. (a) Para responder tal questão, procederemos contando a quantia de nú­
meros em que o dígito 9 não aparece, subtraindo este resultado do total
de números que podem ser formados, obtendo, assim, o valor procu­
rado. Como só nos interessam números menores do que 1.000, pode­
mos nos restringir aos 999 números compreendidos entre 1 e 999. Não
contendo 9 em sua formação, temos 9*9-9 = 729 números diferentes
(observe que os números de um dígito e de dois dígitos também foram
contados como, por exemplo, 081 e 007). Nesse caso, todavia, conta­
mos também o número 000 = 0, donde se tem apenas 728 números a
serem retirados de 999, obtendo-se 999 — 728 = 271 como resposta ao
problema.
Para que o dígito 9 ocorra duas vezes, o número deve ter pelo menos
dois dígitos. Se tiver exatamente 2, só é possível o número 99. Se tiver
3, os 2 dígitos 9 podem ocupar as últimas duas casas (8 opções), ou
as extremidades (9 opções), ou as duas primeiras posições (9 opções).
Lembrando que também há o caso do número 999, temos como res­
posta: 1+84-9-1-9-1-1 = 28 possibilidades.
(b) Aplicaremos o mesmo procedimento do item anterior. Para números
de 3 dígitos, são 9-9-9 = 729 os números em que 0 não ocorre.
Considerando números de 2 dígitos, o número de vezes em que 0 não
ocorre é 9 • 9 = 81. Por último, são 9 os números com um dígito
distintos de 0. Logo, temos 729 + 81 + 9 = 819 os números menores do
que 1.000 que não apresentam 0 em sua composição. Assim, a resposta
ao nosso problema é o total de 999 — 819 = 180 números.
Com relação ao número de vezes em que o dígito 0 aparece duas vezes,
é fácil ver que isso só é possível se exatamente as últimas duas casas
forem ocupadas por tal dígito. Logo, são 9 as possibilidades nesse caso.
(c) Vejamos a quantidade de números que não apresentam nem 0 nem 9
em sua composição. E claro que, neste caso, temos 8 opções para cada
casa, donde 8-8-8 = 512 é a quantia de números de 3 dígitos que
não apresentam 0 nem 9 em sua formação. Quanto a números de 2
dígitos, são 8 • 8 = 64 aqueles que não apresentam 0 nem 9 em sua
formação. Finalmente, para números de um só dígito, temos apenas
8 opções. Portanto, é igual a 512 + 64 + 8 = 584 o número de vezes
em que nem 0 nem 9 ocorrem. Logo, 999 — 584 = 415 é a quantia de
números que apresentam 0 ou 9 em sua composição. Logo, sendo n o
valor procurado, temos, utilizando os resultados dos itens anteriores,
Parte II. Resoluções - Princípio da Inclusão e Exclusão 131

que22 415 = 180 4- 271 — n, donde n = 36, que é o número de vezes em


que 0 e 9 aparecem simultaneamente.
Analogamente, vem que 7-8-8 + 7- 8-I-7 = 511 é a quantia de
números que não apresentam nem 8 nem 9 em sua composição. Logo,
são 999 — 511 = 488 os números em que 8 ou 9 ocorrem. Portanto,
aplicando o mesmo raciocínio anterior, e utilizando a simetria entre os
números 8 e 9, vem que 488 = 271 + 271 — n, donde n = 54 é o número
de vezes em que os dígitos 8 e 9 ocorrem simultaneamente.

131. (a) Caria lançamento pode resultar em 6 valores distintos e, como são 6
os lançamentos, o número de possibilidades é igual a 66 = 46.656.
(b) Por razões didáticas, vamos dividir o problema em casos:
i. número de maneiras de resultarem 6 faces idênticas;
E fácil ver que, como só temos seis valores possíveis, são apenas 6
as possibilidades nesse caso.
ii. número de maneiras de resultarem exatamente duas faces distin­
tas;
Sejam, por exemplo, tomadas as faces 1 e 2 dos dados. Calculare­
mos o número de maneiras de que exatamente tais faces resultem
no lançamento. Como temos 6 dados e cada um deverá resultar
em 1 ou 2, são 26 as possibilidades. Nesse número, porém, estão
embutidos os dois casos em que só uma das faces ocorre (só o valor
1 ou só o valor 2). Então, para cada par de faces distintas, temos
26 — 2 opções em que exatamente duas faces ocorrem. Ocorre,
ainda, que são as escolhas possíveis das duas faces. Então, o
resultado é (®) ■ (26 - 2) = 15 • 62 = 930.
iii. número de maneiras de resultarem exatamente 3 faces distintas;
Aplicando uma resolução similar à da parte ii, tomemos as faces
1, 2 e 3 dos dados e vejamos qual o número de maneiras de as três
22
Aqui, utilizamos que, dados dois conjuntos A e B, é válida a igualdade:

|AuB| = |A| + |B|-|AnB|,

sendo |X| a cardinalidade, ou seja, o número de elementos, do conjunto X. Em nosso caso,


consideramos:

A = {números menores do que 1.000 que apresentam o dígito 0 em sua formação} e

B = {números menores do que 1.000 que apresentam o dígito 9 em sua formação}.


Ao leitor que não estiver familiarizado com a idéia aqui utilizada, contida no Princípio da
Inclusão e Exclusão, sugerimos a leitura do Capítulo 4 de [13].
132 Problemas Resolvidos de Combinatória

ocorrerem sem exceção. São 36 as possibilidades de que pelo menos


uma das 3 ocorra. Nesse número, porém, estão incluídos os casos
em que pelo menos uma das 3 faces não ocorre. Definamos os
conjuntos Ai = {lançamentos em que a face i não ocorre}, para
i = 1,2,3. Temos, pois, pelo Princípio da Inclusão e Exclusão,
que o número de possibilidades de pelo menos uma das faces não
ocorrer é igual a:
|.4i U A? U 4s| = |j4i| 4- I-A21 + |A.31 — |Ai D A2I — |Ai D A3|
— IA2 n A3I + |4.i n A2 n A3I
= 3 • 26 — 3 • l6 4- 0
= 189.
Logo, como o total de lançamentos é 36, para cada trio de faces
distintas, temos 36 — 189 = 540 lançamentos em que exatamente
3 faces distintas são obtidas. Assim, temos (®) • 540 = 10.800
maneiras possíveis, sendo que (®) escolhe quais são as 3 faces dis­
tintas.
iv. número de maneiras de resultarem exatamente 4 faces distintas;
Repetindo o raciocínio do item anterior, consideremos apenas as
faces 1, 2, 3 e 4 e Aj’s os conjuntos formados pelos lançamentos
nos quais a face i não ocorre. Então, procuramos:
|Ai U A2 U Á3 U A41 =
= 1^11 + |^2| + 1^31 + M41 — |Ai n A2I — |Ai A A3I
— |Ai n a4| - IA2 n a3| - |A2 d a4| — IA3 n a4|
4- |Ai n a2 n A3| + |Ai n Á2 n a4| + |Ai n A3 n a4|
4- |a2 n a3 n a4| - |Ai n a2 n a3 n a4|
= 4-36 -6-26 4-4-l6 -0 = 2.536.
Logo, são 46 — 2.536 = 1.560 as possibilidades para cada 4 faces
escolhidas. Portanto, são (®) • 1.560 = 23.400 as possibilidades de
que exatamente 4 faces distintas apareçam no lançamento.
número de maneiras de resultarem exatamente 5 faces distintas;
O raciocínio empregado é o mesmo que o dos itens anteriores. Sem
tantos detalhes, vem como resposta:

©WCP-G) • 46 6
• 26

= 6(15.625 - 20.480 4- 7.290 - 640 + 5) = 10.800.


Parte II. Resoluções - Princípio da Inclusão e Exclusão 133

Observe que a resposta deste caso coincide com a do caso iii..


vi. número de maneiras de resultarem exatamente 6 faces distintas.
Este caso pode ser resolvido de uma maneira mais simples. Como
são 6 as faces distintas e são também 6 os lançamentos possíveis,
é claro que temos à nossa disposição 6! = 720 possibilidades nesse
caso.
Observe agora que, somando as parcelas obtidas nos itens i. a vi.,
temos 6 + 930+10.800 + 23.400 + 10.800 + 720 = 46.656, valor idêntico
ao do item (a), para alívio da matemática!

132. Suponhamos que A, B, C e D sejam as 4 atendentes. Considere os conjuntos


X = {recebimentos nos quais X fica sem ocupação}, para X = A, B, C, D.
A fim de resolvermos o problema, vamos subtrair do total de recebimentos
possíveis, que é 48 (cada ligação pode ser atendida por qualquer uma dar
quatro moças), o total de recebimentos nos quais alguém fica desocupad
Este número é igual a |A U B U C U B|, que pode ser assim calculado:

IAUBuCUDI = |A| + |B| + |C| + |E>| - |A D B\ - |A D C| - |A O D\


- |B n C| - |B n z?| - |C n D\ + |A n B n C|
+ |A n B n r>| + |A n c n 2?| + |B n c n D|
- |AnBnCnP|
= 4 ■ 38 — 6 • 28 + 4 • l8 — 0
= 24.712.

Logo, o resultado é igual a 48 — 24.712 = 40.824 recebimentos possíveis.

133. Para resolvermos essa questão, vamos considerar o Princípio da Inclusão e


Exclusão. Sejam os conjuntos:

A = {an agram as em que há dois N’s juntos};


B = {anagramas em que há dois S’s juntos};
C = {anagramas em que há dois A’s juntos}.

Para resolvermos nosso problema, precisamos encontrar a cardinalidade de


A U B U C para, em seguida, retirar este valor do total de anagramas. Po-
rém, |A| = |B| = |C| = 2^i = 180, |A n B| = IA A C| = |B n C| = § = 60
134 Problemas Resolvidos de Combinatória

e |A n B O C| = 4! = 24. Portanto:

|A U B U C| = |A| + |B| + |C| - |AnB| - |A n C| - |B n Cj


+ lAnBnCI
= 3-180 - 3-60 + 24
= 384.

Portanto, o número de anagramas procurado é igual ao total de anagramas


menos |A U B U C|, isto é, 21.2'1.21 “ 384 = 630 — 384 = 246.

134. Observe que temos uma expressão com 10 letras constando de 5 pares de
letras iguais. Para maior praticidade, vamos dividir o problema em casos:

(a) Selecionam-se letras de 5 tipos distintos;


Nesse caso, devemos inevitavelmente selecionar 4 letras distintas e um
par de letras iguais. Considere, por exemplo, a seleção OOATLE.
Temos, pois, || = 360 anagramas distintos, sendo que em 5! = 120
deles as letras O ficam juntas. Logo, temos 360—120 = 240 anagramas
nos quais não aparecem letras O juntas. Como qualquer uma das letras
pode ser dupla na escolha, temos, nesse caso, 240 ■ 5 = 1.200 opções.
(b) Selecionam-se 2 pares distintos de letras iguais e mais duas letras dis­
tintas;
Por exemplo, temos a escolha AATTLE. Eis um pequeno problema a
ser resolvido pelo Princípio da Inclusão e Exclusão. Sejam os conjun­
tos:
A = {an agram as em que há duas letras A juntas};
T = {anagramas em que há duas letras T juntas}.
Logo, vem que:

|A U T\ = |A| + |T| - |A A T) = 2 • - 4! = 120 - 24 = 96.

Logo, o número de arranjos das 6 letras escolhidas que não contêm


letras iguais juntas é — 96 = 84. Portanto, o número possível de
escolhas é (2) • (2) • 84 = 2.520, sendo que ($) escolhe as letras que
aparecerão aos pares e (2) escolhe as outras duas letras distintas.
(c) Selecionam-se 3 pares distintos de letras iguais.
Agora, consideremos a escolha OOTTLL. Sejam os conjuntos:
Parte II. Resoluções - Princípio da Inclusão e Exclusão 135

O = {anagramas em que há duas letras O juntas};


T = {anagramas em que há duas letras T juntas};
L = {anagramas em que há duas letras L juntas}.
Temos, então, que:

|OUTUL| = |O| 4- |T| + |L| - |O n T\ - |O D L\ - |TA L\


+ |OnTDL|

= 3-
60.
5!
-34+3!
2! • 2!

Logo, o número procurado de arranjos com as letras escolhidas é igual


a 2,j;.2! — 60 = 90 — 60 = 30. Unindo a este resultado o número de
escolhas de 3 pares distintos de letras, temos Q) • 30 = 300 possibili­
dades.

Portanto, temos 1.200 + 2.520 4- 300 = 4.020 permutações possíveis.

135. Consideremos as três pessoas denotadas por A, B e C.

(a) Vamos inicialmente considerar o total de divisões que podem ser feitas,
incluindo casos em que alguma pessoa não recebe letras. Como dispo­
mos de 6 letras a, devemos dividir tais letras entre três pessoas o que,
como sabemos, pode ser relacionado com a equação xi 4- X2 4- X3 = 6.
Nela, cada variável representa as quantidades de letras a que vão ser
distribuídas para cada pessoa, e 6 é o total disponível de letras a
serem distribuídas. Então, o número de divisões possíveis da letra em
questão é igual ao número de soluções inteiras não negativas da dita
equação, que sabemos ser dado por (3+|-1) = (|) = 28. Além disso,
devemos dividir as 9 letras restantes (que são duas a duas distintas)
entre as três pessoas. Assim, chegamos a um total de divisões igual
a 28 • 39 = 551.124. Desse número, entretanto, devemos retirar os
casos em que alguém não recebe nenhuma letra. Considere os con­
juntos X — {divisões em que a pessoa X não recebe nenhuma letra},
para X = A,B,C. Procuramos o valor de |A U B U C|. Observe
agora que |A| = |B| = |C| = 7 • 29 = 3.584, sendo que 7 e 29 con­
tam, respectivamente, o número de maneiras de se distribuírem as
letras a e o número de maneiras de se distribuírem as outras letras
entre as duas pessoas candidatas a recebê-las. Além disso, observe
que |A A B| = |A Cl C| = \B D C| = 1, pois só há uma maneira de dar
todas as letras a alguém. Por último, é claro que |,4 A B Cl C| = 0.
136 Problemas Resolvidos de Combinatória

Portanto, |A U B U C| = 3 ■ 3.584 — 3 • 1 + 0 = 10.749. Finalmente,


descontando este valor do encontrado inicialmente, temos a resposta
551.124 - 10.749 = 540.375.
(b) Explicaremos a idéia utilizada na resolução para maior clareza. Vere­
mos primeiramente as distribuições possíveis da letra a. Adotaremos,
para tanto, a seguinte notação: |aaa|aa|a|, que indica que uma das
pessoas recebe 3 letras a, outra recebe duas e outra recebe apenas
uma. Temos ainda, nesse caso, 3! maneiras para decidir quem re­
ceberá 3 letras a, quem receberá duas e quem receberá uma. Em
seguida, faremos a distribuição das letras que faltam para cada pes­
soa. Na situação acima, por exemplo, precisaríamos dar mais duas
letras para aquela que recebeu 3 letras a, 3 para quem recebeu duas e
4 para quem recebeu somente uma. Vejamos, então:
i. Caso 1: |aaaaa|a|-|;
3! ■ © ■ (4) • © = 6 • 126 = 756.
ii. Caso 2: |aaaa|aa|-|;
3! • (1) ’ (3) ’ (5) = 6 • 9 • 56 = 3-024.
iii. Caso 3: |aaaa|a|a|;
3 ‘ (1) • G) ’ (4) = 3 • 9 • 70 = 1-890-
iv. Caso 4-' |aaa|uaa|-|;
3 • (2) • (2) ■ © = 3 • 36 • 21 = 2.268.
v. Caso 5: |aatz|aa|a|;
3! • (2) ’ (3) • (4) = 6'36'35 = 7-560-

vi. Caso 6: |aa|aa|ua|.


1' (3) ' © ' (3) = 84 ■ 20 = 1.680.
Somando os resultados obtidos, temos um total de 17.178 maneiras de
distribuir 5 letras para cada pessoa.

136. Considere os seguintes subconjuntos do conjunto de todas as permutações


de 4 letras do nosso alfabeto:

j4i= {primeira e segunda posições ocupadas por letras iguais};


Â2 = {segunda e terceira posções ocupadas por letras iguais};
A3 = {terceira e quarta posições ocupadas por letras iguais}.

O enunciado sugere, então, que encontremos o número de elementos do


conjunto A1UA2UA3. Para tanto, observemos que |Ai| = I-A2I = IA3I = 233,
|Ai n A2I = |Ai n A3I = |Á2 n A3I = 232 e |Ai n A2 n A3I = 23
Parte II. Resoluções - Princípio da Inclusão e Exclusão 137

Logo, temos que:

|AuAuA3| =
= |Ai| + |A| + |AI — |Xi n Al - |A n Al — IA nAl
+ IA n A2 n A3| = 3 • 233 - 3 • 232 + 23 = 34.937.

Portanto, uma outra maneira de se chegar ao mesmo resultado do exercício


proposto é retirar este valor do total de permutações de 4 letras, isto é,
fazer 234 - 34.937 = 244.904 = 23 • 223.

137. (a) Na fórmula do Binômio de Newton, tomemos a = 10 e & = — 1. Então:

n n
9n = (10 + (-l))n (-ir =Ê(-üQio’
i=0 v ' i=0 ' '

que é justamente a identidade a ser demonstrada.


(b) Agora, vamos considerar a contagem de n-seqüências, isto é, de seqüên­
cias de n dígitos. Podemos entender 9n como o número de n-seqüências
nas quais não se utilizam dígitos iguais a 0 em sua composição. Por
outro lado, se contarmos as n-seqüências que apresentam pelo menos
um dígito 0 em sua composição e, em seguida, subtrairmos tal valor do
total possível de n-seqüências, também estaremos contando as mesmas
seqüências contadas por 9n. Assim, para i E N tal que 1 < i < n,
definamos:

A = {n-seqüências que apresentam o dígito 0 na i-ésima posição}.

Assim, o total de seqüências de n dígitos que apresentam pelo menos


um 0 em sua composição pode ser calculado, pelo Princípio da Inclusão
e Exclusão, como:

u* - (?)»- + ... + (_l)n-2 n


n—1
101

+ (-1)"-1
n
= E(-l) (:)■”
138 Problemas Resolvidos de Combinatória

Assim, pelo raciocínio apresentado acima, temos:

gn 10n lOn-f

- w+ £(-!>■ Q) 10n-i

n
10n-i

que é o resultado esperado.

38. (a) Para encontrarmos a resposta deste item, basta que tenhamos clara a
definição de função, segundo a qual cada elemento do domínio deve
possuir uma única imagem no contradomínio. Assim, para cada um
dos n elementos do domínio A, existem k possíveis imagens no con­
tradomínio B, donde kn é o número de funções f : A —> B.

(b) Tomando um dos n elementos de A, este possui n possíveis imagens


(fc = n). Em seguida, tomando outro elemento de A, este possuirá n-1
imagens possíveis, pois a função deve ser bijetora. Prosseguindo com
esse raciocínio, temos n(n — 1) • • • 1 = n! funções bijetoras. Observe
que, se n = k = 0, então, como 0! = 1, o resultado também vale,
pois de fato só existe uma função bijetora que leva um conjunto vazio
noutro.

(c) Analogamente ao que fizemos no item (a), temos k(k — 1) • • • (fc —n+1)
funções injetoras.

(d) Este item demanda a aplicação do Princípio da Inclusão e Exclusão.


Como desejamos saber o número de funções sobrejetoras de A em
B, podemos contar primeiro aquelas que não são sobrejetoras e, em i
seguida, subtrai-las do total de funções. Antes, porém, dado um con­
junto B' C B, adotemos que f^ÇB1) = {x G A | f(x) G B'}. Assim,
sejam ê>i, 62, .os k elementos distintos de B. Assim, podemos
construir Ci = {f : A —* B | /-1({òi}) = 0}- Então, o número de
funções não sobrejetoras de A em B será igual ao número de elemen­ I
I
tos de Ci U C2 U ... U Cjt- Agora, utilizando o item (a), vem que
|<Sf| = {k — l)n, |Ci D Cj| = (k — 2)n, e, assim, sucessivamente, até
Parte II. Resoluções - Princípio da Inclusão e Exclusão 139

|Ci n ... n Ck| = (fc — k)n. Logo:


|C1 U C2 U ... U Cjtl =

Q)(fc-2)»+ •••+ (-l)‘-‘


k

= E(-d O-*)”
1=1

Logo, o total de funções sobrejetoras é igual a:


k k
kn E(-d
número que denotaremos T(n, k).

139. (a) Cada pessoa deve receber 7 peças. Para tanto, uma maneira simp._
de resolver o problema é colocar todas as peças em fila e, em seguida,
distribuir as 7 primeiras para a primeira pessoa, as próximas 7 para
a segunda pessoa e, assim, sucessivamente. E claro que, para pôr as
peças em ordem, temos 28! possibilidades. Entretanto, como a ordem
das mesmas em cada um dos 4 grupos de 7 peças é irrelevante (pois
só desejamos distribuí-las às pessoas), o total de distribuições a que
chegamos é
(b) O problema que temos agora equivale ao de dividir 28 objetos distintos
entre 4 caixas distintas de modo que nenhuma caixa fique vazia. Note,
porém, que o número de maneiras de se realizarem tais divisões é
justamente o de funções sobrejetoras de um conjunto de 28 elementos
em outro de 4 elementos. Como vimos no item (d) do Exercício 138,
este número é:

T(28,4) = (-1)° Q) (4-O)28+ (-!)’

+ (-1>2© (4-2)28 + (-l)3Q) (4 - 3)28

+ (-!)“ Q)(4-4)28
428 - 4 • 328 + 6 • 228 - 4,

valor da ordem de 1017.


140 Problemas Resolvidos de Combinatória

140. (a) Denotemos Ai = {disposições em que o i-ésimo casal fica junto}. As­
sim, | AuAuAuAuAl é igual ao número de permutações circulares
nas quais pelo menos um dos casais permanece junto. Bastar-nos-á,
então, subtrair esse valor do total possível de disposições dos 5 ca­
sais. Observe, porém, que |A| = 2-8!, sendo o fator 2 responsável
por considerar a ordem do i-ésimo casal e 8! responsável pela per­
mutação circular das 8 pessoas restantes juntamente com o i-ésimo
casal. Analogamente, temos | A D Aj\ = 22 • 7!, | A O Aj D A| = 23 • 6!,
|A n Aj n Ak n Al = 24 • 5! e | A n a2 n A3 n A n Al = 25 • 4!. Logo,
pelo Princípio da Inclusão e Exclusão, vem que:
|A u Au A u A u A| =
= 0(2-89- (|)(22-7!) + Q}(23-6!)- Q)(2<5!)
+ G) (25 • 4!) = 250.368.

Logo, o resultado do problema é igual a 9! — 250.368 = 112.512.


(b) Na mesma linha do item (a), temos que, para n casais:

IAU...U A| =
= f”}(2-(2n-2)!)- Q(22-(2n-3)!) + ---

(2"(n - 1)!)
n

= E(-1)-' Q(2i-(2n-i-l)l).
1=1

Logo, a resposta será:


(2„-l)!-^(-l)í-l Q^(2<-(2n-í-l)!) =
1=1
n
= (2n-l)! + ^2(-l)i (2n-»-!)!)
i=i

i-0 ' '

141. Consideremos A = {1,2,... ,m} e, para cada i entre ler, definamos o


conjunto Ai = {x e A | x é múltiplo de pi}. Desse modo, observe que o
Parte II. Resoluções - Princípio da Inclusão e Exclusão 141

fato de um número ser divisor de m equivale ao fato dele pertencer a algum


desses A’s. Assim, ç»(m) é igual à cardinalidade do complemento da união
rn
dos Ai's em relação ao conjunto A. Não é difícil ver, agora, que |Af| = Pi '
m
|Ai n e, assim, sucessivamente, até |Ai D ... n Ar| = P\P2'”Pr ’
Logo, segue que:
m
0(m) = m~y--- 1-
v—a m
y 2ZL + ... + (-ir
i Pi íèi- pw P1P2'‘' Pr

1 1
m i-y-+ ^<í —
y PiPj + + PlP2--'Pr
xÍíjPiP^
i- —
v pij \ P2J ' Pr
como queríamos mostrar.
142. (a) Havendo tantas latas quanto queiramos de cada tipo, basta que conte­
mos quantas latas de cada um dos tipos serão compradas, número que
sabemos coincidir com a quantidade de soluções inteiras não negativas
da equação xi 4- x? 4- X3 4- X4 = 15. Assim, há (
possíveis compras, utilizando o Exercício 25.
i5ir*) = ca=«is
(b) Neste caso, teremos que nos valer do Princípio da Inclusão e Exclusão.
Considerando a mesma equação do item (a), definamos, para cada i,
os conjuntos Ai = {compras de 15 latas nas quais Xi > 7}, sendo x, o
número de latas de refrigerante do tipo i que foram compradas. Assim,
do total calculado em (a), devemos retirar IA1UA2UA3UA4I para obter
o número desejado. Consideremos, por exemplo, o cálculo de |Ai|. Se
rzq > 7, então x±—7 > 0. Logo, podemos substituir, na equação do item
(a), xi — 7 por yi, e Xi = yi para i 1. Então, temos a nova equação
2/1 + 2/2 + 3/34-3/4 = 8. Assim, |Ai | corresponde ao número de soluções
inteiras não negativas desta equação, isto é, = (V) = 165- ^or
simetria, vem também que JAsl = IA31 = | A4I = 165. Analogamente, o
número de elementos de Ai D Aj é igual ao número de soluções inteiras
não negativas de yi 4- 2/2 4- 1/3 + V4 — 1> que é igual a 4. Agora,
observando que as intersecções de três ou mais dos conjuntos Ai são
vazias (para elas, o número de latas compradas é pelo menos 21, que é
maior do que 15), temos que |Ai U A2UA3UA4I = 4-165— (2) -4 = 636.
Logo, a resposta é 816 — 636 = 180.

143. Denotemos por P, I e F os conjuntos de crianças que estudam, respectiva­


mente, português, italiano e francês. Pelo Princípio da Inclusão e Exclusão,
142 Problemas Resolvidos de Combinatória

|PUZUF| = |P|4-|Z|4-|F|-|PAZ|-|PAF|-|ZAF|4-|PAZAF|. Logo,


pelo enunciado, 40 — 10 = 26 4- 204-14 — |P A Z| — |P A F| — |Z AF| + 0, donde
|P AZ| 4- |PAF| 4- |ZAF| = 30 = |PUZUF|. Para melhor compreendermos
o que ocorre, observemos o seguinte esquema:

Observe nele que, como |P A Z| 4- |P A F| 4- |Z A F| = 30 = |P U I U F|,


não existe nenhum aluno que estude somente uma das três línguas. Logo,
obtemos o seguinte sistema:

' 26 = |P| = |PAZ| + |PAF|


< 20 = |Z| = |PAZ| 4- |ZAF|
14 = |F| = jz A F| 4- |PAF|

Resolvendo-o, segue que |P A Z| = 16, |Z A F| = 4 e |P A F\ = 10. Logo,


16 crianças estudam português e italiano, 4 estudam italiano e francês e 10
estudam português e francês.
144. Para tanto, vamos definir os seguintes conjuntos:

A= {ordenações das 9 letras nas quais há dois a’s juntos};


B= {ordenações das 9 letras nas quais há dois b’s juntos};
C= {ordenações das 9 letras nas quais há dois c’s juntos};
D= {ordenações das 9 letras nas quais há dois d’s juntos}.

Agora, para solucionarmos o problema, basta que retiremos |AuBuCUjD|


do total possível de ordenações das letras dadas. Observemos, porém, que
as contagens envolvendo o conjunto B devem ser tratadas com cuidado
especial, uma vez que, diferentemente das outras letras, existem três ò’s
no multiconjunto dado. Assim, temos |A| = }C'| = |Z)| = — 1-680.
Com relação ao conjunto B, ao efetuarmos sua contagem como fizemos com
os outros conjuntos, contamos duas vezes as ordenações que constam de 3
Parte II. Resoluções - Princípio da Inclusão e Exclusão 143

6’s juntos. Assim, devemos fazer a mesma contagem que fizemos com os
conjuntos anteriores e, em seguida, subtrair uma vez o total de ordenações
das nove letras nas quais há 3 6’s juntos. Logo, |B| = — ^jp = 4.410.
Analogamente, temos \A l~) C*| = |A A £>| = |C Cl £>| = 5T3! = 420, e ainda
|A A B| = |B D C| = |B A D| = pjrp — ^p = 1.080. Além disso, temos
lAnCnPI = g = 120 e |AabaC| = |Aabal>| = |BaCa£>| = = 300
e, finalmente, |A A B D C A £>| = 5! — 4! = 96. Portanto:
|AUBUCUD| = (3-1.680 + 4.410)-(3-420 + 3-1.080)
+ (120 + 3 ■ 300) - 96
= 5.874.
9! - 5.874 = 1.686.
Portanto, a solução que procuramos é igual a (2!)á-3!

145. Podemos considerar os 10 camelos numa ordem inicial preestabelecida de­


signando-os por números de 1 a 10. Assim, consideremos que, inicialmente,
os camelos se encontrem na ordem 1-2-3-4-5-6-7-8-9-10. Agora, podemos
denotar Ai como sendo o conjunto das permutações em que o camelo 1 vem
antes do camelo 2, A como o conjunto das permutações em que 2 vem antes
de 3, e, assim, sucessivamente, até definirmos Ag, conjunto das permutações
em que 9 vem antes de 10. Para que, na nova fila, nenhum camelo tenha à
sua frente um camelo igual ao que tinha na disposição anterior, basta que,
do total de permutações, retiremos o número de elementos da união dos
A’s. Agora, temos que |A| = 9!, |A D Aj\ =8!, e, assim, sucessivamente,
até |A A ... A Al = 1- Logo:
|A U...U A| =

9 8 6 5 4 3 2 1
= 9!
1!
+7
2! '3! 4!
+ 5!
+
6! '7!
+ 777
8! ' 9!
Portanto, o número de permutações possíveis é igual a:
98765432 1
10! - 9!
1! 2!
+ 3! 4! + 5! 6! +'7! ~ 8! +' 9!}

= 9!
w _ 2,1 _ I , 1 _ £ , £ _ _3 1
1! + 2! 3! + 4! 5! + 6! 7! + 8!
9!
= 1.468.457.
144 Problemas Resolvidos de Combinatória

146. Consideremos A = {1,2,3,..., 100.000}, A± = {n € A | n é múltiplo de 2}


e A 2 = {n € A | n é múltiplo de 3}. Queremos calcular |Ai U A2I e, para
tanto, temos que |Ai| = 59 goo, IA2I = [J00300C!j = 33.333 e
|Ái D A2I = L102-3°°J = 16.666, sendo [ J : IR —» Z a chamada função maior
inteiro, que associa a cada número real o maior inteiro menor do que ou
igual a ele23. Assim, |Ai U A2I = 50.000 + 33.333 — 16.666 = 66.667 inteiros
é a resposta do problema.

147. Como os algarismos de 1.000.000 não têm soma igual a 15, podemos con­
siderar todos os inteiros entre 1 e 999.999 como sendo de seis dígitos. Por
exemplo, 43 pode ser visto como 000.043. Representando cada algarismo
por a?i, X2, ..., xq, o resultado do problema consiste no número de soluções
inteiras não negativas da equação aq + • • • + xq = 15, desprezando-se as
soluções em que Xi > 10, para 1 < i < 6. Ocorre que o número de soluções
da equação acima é igual a = (25°), pelo Exercício 25. Supondo
agora, em particular, x^ > 10, podemos fazer a substituição xi = yi + 10
e Xi = yi, 2 < i < 6, recaindo na equação yi + • • • + y& = 5, com yi > 0
para cada i. Assim, obtemos uma nova equação que possui — (5°)
soluções. Como tal raciocínio pode ser empregado para cada uma das seis
variáveis da equação original, a resposta do problema é (25°)~ 6(5°) = 13.992.
Observe que esta é uma aplicação do Princípio da Inclusão e Exclusão.

148. (a) Para resolver o problema, vamos primeiramente contar as permutações


que possuem dois A’s juntos, subtraindo-as, em seguida, do total de
permutações. Para tanto, podemos supor que duas letras A formam
um único bloco AA. Assim, temos sete letras e o bloco AA, o que
resulta 8 blocos, sendo 3 iguais a B, 3 iguais a C e 2 distintos (A e
AA). Logo, há 373? = 1-120 permutações possíveis desses oito blocos.
Todavia, nestas incluem-se duas vezes as que possuem o bloco AAA.
De fato, cada vez que AA vem seguido de A, poderiamos ter também
A seguido de AA, sem distinção. Assim, como = 140 é o número
de permutações que contêm o bloco AAA, temos 1.120 — 140 = 980
permutações contendo duas letras A juntas. Logo, como o total de
permutações das 9 letras é 31.31.3f = 1.680, temos 1.680 — 980 = 700
nas quais não figura o bloco AA.
(b) Considere os seguintes conjuntos:
X = {anagramas de AAABBBCCC nos quais figura o bloco AA};
Y = {anagramas de AAABBBCCC nos quais figura o bloco BB}.
23Para maiores detalhes, veja o Capítulo 4 de [13].
Parte II. Resoluções - Princípio da Inclusão e Exclusão 145

Para resolvermos este item, procederemos como no anterior, calcu­


lando, primeiramente, |X uy|. Já sabemos que |%| = |y| = 980, pelo
item anterior. Resta-nos, pois, encontrar |X ny|. Para tanto, vamos
considerar os blocos AA e BB. De modo análogo, temos que o número
de permutações que possuem esses dois blocos é igual a = 840.
Nesse número, porém, foram contadas duas vezes as permutações que
contêm o bloco AAA e o bloco BB e duas vezes as permutações que
contêm BBB e AA. As permutações que contêm AAA e BB são em
número de |í = 120, o mesmo valendo para as que contêm AA e BBB.
Logo, aparentemente, 840 — 2 • 120 = 600 deve se referir ao número
procurado. Entretanto, fazendo a contagem acima, retiramos duas
vezes os anagramas que contêm AAA e BBB, que são em número de
|í = 20. Portanto, 600 + 20 = 620 é, de fato, o número procurado.
Ássim, |Xur| = |X| + |y|-|Xny| = 980 + 980-620 = 1.340, donde
o número de anagramas que não contêm dois A’s nem dois B’s juntos
é 1.680 - 1.340 = 340.

(c) Além dos conjuntos já criados no item (b), consideremos:

Z = {anagramas de AAABBBCCC nos quais figura o bloco CC}.

Para calcularmos |XU VUZ|, os itens (a) e (b) nos fornecem, respecti­
vamente, |X| = |y| = \z\ = 980 e |x n y| = |x n z\ = |y n z\ = 620.
Logo, resta-nos calcular jX ny n Z\, o que não será tarefa fácil! Con­
siderando os blocos AA, BB e CC, temos 6! permutações envolvendo
eles e as letras remanescentes A, B e C. Nestas, contamos duas vezes
as permutações que contêm AAA, BB e CC, duas vezes as que contêm
AA, BBB e CC, e duas vezes as que contêm AA, BB e CCC. Por
exemplo, as que contêm AAA ocorrem 5! = 120 vezes. Logo, como
são três os casos, temos uma nova quantia de 720 — 3 • 120 = 360 per­
mutações. Acabamos, agora, por retirar duas vezes as permutações
que contêm AAA, BBB e CC, duas vezes as que contêm AAA, BB e
CCC, e duas vezes as que contêm AA, BBB e CCC. Cada um desses
grupos de permutações tem número de elementos igual a 4! = 24.
Portanto, nosso novo sub-resultado é o valor 360 + 3-24 = 432. Final­
mente, estamos agora contando duas vezes as permutações que contêm
AAA, BBB e CCC, que são em número de 3! = 6. Temos, enfim, que
|X A y A Z| = 432 - 6 = 426. Logo:
146 Problemas Resolvidos de Combinatória

|x u y u z\ = |x| + |y| + |z|-|xnr|-|xnz|-|ynz|


+ |xn ynz|
= 3 • 980 - 3 • 620 + 426
= 1.506,

donde o número procurado é 1.680 — 1.506 = 174.

149. Denotemos os alemães por Ai, A2, A3, os espanhóis por Ei, E2, E3, e os
franceses por Fi, F2, F3. Então, nosso problema se reduz a encontrar o
número de permutações das letras A1A2A3E1E2E3F1F2F3, de modo que
não haja letras de mesma espécie juntas. Ora, o item (c) do Exercício 148
nos garante que a resposta seria igual a 174 caso não houvesse índices nas
letras. Ocorre que, se considerarmos cada permutação das letras sem os
índices, podemos acrescentá-los de (3!)3 maneiras (3! escolhas para cada
tipo de letra). Logo, a resposta é 174 • (3!)3 = 37.584 modos distintos.

150. Considerando que são apenas duas pessoas, basta que dividamos os objetos
para uma delas, pois, dessa forma, os da outra também ficam determinados.
Sendo assim, considere que xy, X2 e x3 sejam as quantidades de cada um
dos três tipos de objetos distribuídos a uma das duas pessoas. Então,
nosso problema se resume a encontrar o número de soluções inteiras não
negativas da equação x^ + X2 + x3 = 3n, sujeita à restrição de que Xi < 2n,
para i = 1,2,3, pois só dispomos de 2n objetos de cada tipo. O número de
soluções da equação acima é ('. 3n+3-1
3-1 ) ~ Agora, retiraremos desse
montante as soluções que apresentam uma das variáveis maior do que 2n.
Particularmente, para zi > 2n, tomemos x{ = xi — 2n — 1 > 0, x'2 = %2 e
x'2 = X3. Assim, temos a nova equação x{ + x'2 + x'3 = n — 1, que possui
zn—1+3—1
l 3-1 ) = (”2*) soluções. Como os casos em que X2 > 2n e x3 > 2n
possuem soluções idênticas, o resultado é ('3n2+2) - S^J1) = 3n2 + 3n +1.
Observe, por fim, que esta foi uma aplicação do Princípio da Inclusão e
Exclusão.

151. (a) Basta que efetuemos a distribuição dos objetos para duas pessoas, pois,
assim, os objetos da terceira também ficam determinados. Sendo zi,
X2 e X3 as quantidades de cada um dos três tipos de objetos distribuídos
à primeira pessoa e y\, y2 e 7/3 as quantidades de cada um dos três tipos
de objetos distribuídos à segunda pessoa, nosso problema se resume a
Parte II. Resoluções - Princípio da Inclusão e Exclusão 147

encontrar o número de soluções do sistema de equações:

xi 4- xq + X3 = n
yi+y2 + yz = n,
sujeito à condição Xi+yi < n, para i = 1,2,3, uma vez que só dispomos
de n objetos de cada tipo. O número de soluções inteiras não negativas
de cada uma das equações do sistema acima é (”3^1 ) = (nJ2)- Logo, o
número de soluções do sistema, sem restrições, é ("J2) C*^2)- Todavia,
o número de soluções do problema que violam a condição rei 4- yi < n
corresponde ao número de soluções do sitema:

^2 + = r, 0 < r < n
?/2 4- ?/3 = s, 0 < s < n,

com r 4- s < n, ou seja, r < n — s — 1. Para cada par (r, s), as soluções
do sistema são em número de (r 4-l)(s 4-1). Como s varia de 0 a
n — 1 e r varia deOan — s — 1, temos o seguinte número de soluções
inconvenientes, nesse caso:
n—1n—s—1
(r+i)(s+i) =
s—0 r=0
n—1 n—s—1

= 3=0 12
r=0
(r+
= + 1) (n-5)(n-s+l)
So 2
n—s+1

-GXTHXMXV)- -G)©
í n 4* 2 4“ 1\
= \ 34-1 )

ín 4- 3\
"l 4 /’
aplicando, na penúltima passagem, o resultado encontrado no item (b)
do Exercício 121, para m = n + 2, n — 3 e p = 1. Obviamente, para
os casos X2 4- y-2 < n e x$ 4- yz < n, por simetria, devemos chegar
148 Problemas Resolvidos de Combinatória

ao mesmo resultado. Observe, ainda, que nunca pode ocorrer, por


exemplo, + y\ > n e X2 4- y2 > n ao mesmo tempo, pois, nesse
caso, X3 + 7/3 < 0, isto é, o número de objetos recebidos pela terceira
pessoa é negativo, o que é contraditório. Assim, retirando as soluções
inconvenientes do problema, temos como resposta:

possíveis distribuições.
(b) Basta aplicar a resposta encontrada no item anterior para n = 6. Logo,
temos 2) (6 2 2) — 3(6^3) = (®)2 — 3(9) = 406 maneiras de realizar
tal distribuição.
Parte II. Resoluções - Funções geradoras e partições 149

Funções geradoras e partições


152. (a) Podemos resolver este item de duas maneiras distintas: por argumen­
tos combinatórios e algebricamente.
Da primeira, suponhamos que se disponha de um número ilimitado de
objetos de cada um de n tipos distintos. Já utilizamos em diversos
problemas anteriores que o número de maneiras de se escolherem r
objetos dentre os disponíveis é igual ao número de soluções inteiras
não negativas da equação x± + ■ • ■ + xn = r, sendo Xi o número
de objetos selecionados do i-ésimo tipo (particularmente, reveja o
Exercício 25). Isto é justamente o que temos por combinação com
repetições. Sabemos, também, que esse número de soluções é igual a
("níy1) = (n+r-1)- Além disso, é fácil ver que esse número de soluções
corresponde ao coeficiente de xr na expansão de (l+rr+z2 4---- )n. De
fato, podemos imaginar que cada um dos n polinômios 1 + x + x2 4----
“controla” uma das variáveis Xi da equação apresentada, de tal modo
que, se no desenvolvimento do produto desses n polinômios obtemos
um termo xT, é como se estivéssemos encontrando uma das soluções da
equação acima. Por exemplo, tomando xr-1 no primeiro polinômio, x
no segundo e 1 nos restantes, há equivalência com a escolha de r — 1
objetos do primeiro tipo, 1 do segundo e nenhum dos restantes. Assim,
o coeficiente procurado é igual a (n+^-1).
Por outro lado, podemos demonstrar o mesmo resultado de maneira
algébrica, observando que:
oo
1 n = (l-x)
(1 + x + x2 + • • • )n =
1—x i=0

Agora, é fácil ver que o coeficiente de xr na expansão acima é igual a:

(_l)r = ( (-”)(-”-1)-"(-n-r + l)

( l)r( 1)rn(n+1)---(n + r~ !)

(n + r — 1)!
= (~l)2r
r!(n — 1)!
n 4- r — 1\
r J
No desenvolvimento anterior, utilizamos o conceito de Binômio de
150 Problemas Resolvidos de Combinatória

Newton em sua forma generalizada, a qual inclui valores negativos


como possíveis parâmetros dos números binomiais24.
(b) Este item, com sutis alterações, admite resolução idêntica à do ante­
rior. Entretanto, como já temos realizado aquele, vamos simplificar
este, observando que:
oo
xm :m)-n

i=0
(-l/z™

Assim, para descobrirmos o coeficiente de xrm na expressão acima,


basta que tomemos i = r na mesma, obtendo, como em (a), o valor
cr1)-
153. (a) Como as variáveis da equação estão sujeitas a restrições muito es­
pecíficas, é conveniente que utilizemos funções geradoras para resolver
o problema. Assim, a função geradora que controla cada uma das qua­
tro variáveis é dada por x4 4-x5 4-l-z9, uma vez que cada uma delas
deve estar compreendida entre 4 e 9. Portanto, o número de soluções
da equação dada corresponde ao coeficiente de z30 no desenvolvimento
de:

(x4 + X5 4- ... + x9)4 x16(l 4- x 4- x2 4-... + rr5)4

x16 1 —X )

= x16(l - 4x6 4- 6x 12 - 4a;18 + x24)(l - a;)-4


= (a;16 — 4a;22 4- 6a;28 — 4a;34 4-a:40)(l — a;)-4.

Em (1 — x)~4, portanto, necessitamos apenas dos coeficientes de z14,


x8 e x2 pois, no termo que o multiplica, devemos considerar apenas
a:16, x22 e z28. O coeficiente de 2:14 , como foi visto no Exercício 152, é
igual a (4+i4-1) = (iJ) = 680, o de x8 é igual a (':4+i-1) = (V)=165.
e o de x2 é igual a (4+2-1 ) = (2) = 10. Portanto, o coeficiente de
a:30 na expressão acima e, portanto, a resposta deste item, é igual a
1 ■ 680 - 4 • 165 + 6 • 10 = 80.
(b) Observemos que há uma equivalência entre a equação dada e a equação
aq + a?2 + £3 + a?4 = r, sendo 24 múltiplo de 2, a?2 e X4 múltiplos de 3 e
X3 múltiplo de 5, de tal forma que cada solução de uma das equações
24 Caso desconheça ou não esteja familiarizado com este conceito, sugerimos a leitura do
Capítulo 5 de [13].
Parte II. Resoluções - Funções geradoras e partições 151

se associa com somente uma solução da outra. Assim, a função que


controla xi é a função 1 4- x2 -I- x4 + • • • = Analogamente, e
X4 são controladas por 1 4- x3 4- x6 4- • • • = e X3 é controlada por
1 + x5 4- x10 4---- = Portanto, a função geradora cujo coeficiente
de xr nos remete ao número de soluções inteiras não negativas da
equação original é:

1___________
(1 — x:2)(1 — x3)2(l — x5)’

154. (a) Uma das maneiras de provarmos este resultado é imaginar que se
dispõe de n caixas distintas, cada uma contendo m objetos idênticos.
Supondo que só se pode selecionar caixas inteiras, podemos não sele­
cionar nenhuma caixa de uma única maneira. Podemos selecionar uma
caixa de n maneiras. Em geral, r caixas podem ser selecionadas de (”)
maneiras distintas, sendo que, ao selecionarmos r caixas, estamos, na
verdade, selecionando mr elementos. Desta maneira, por um lado, a
função geradora para tais escolhas é a função (1 4- xm)n e, por outro, é
a função 1 4- (™)xm 4- (^(z”1)2 -I------ 1- (xm)n. Como ambas as funções
controlam as mesmas escolhas, vale a identidade.

(b) Com relação a f(x), podemos imaginar que dispomos de três caixas
distintas, cada uma das quais contendo n objetos idênticos. O coefi­
ciente de x2n+1, bem o sabemos, pode ser interpretado como o número
de maneiras de se escolherem 2n 4-1 objetos das três caixas. Como, de
cada uma, podemos retirar no máximo n objetos, pelo menos um de
cada caixa deve ser selecionado. Portanto, uma vez efetuada a seleção,
tomemos um objeto proveniente de cada caixa (como cada caixa pos­
sui n elementos idênticos, tal seleção é única). Ora, isso é como se
as caixas iniciais tivessem, cada uma, apenas n — 1 objetos e, em vez
de selecionarmos 2n 4- 1 objetos, selecionássemos 2n — 2, seleção que
é contabilizada pelo coeficiente de x2n~2 em g(x). Reciprocamente,
se tivermos três caixas contendo, cada uma, n — 1 objetos idênticos e
desejarmos selecionar 2n — 2 objetos, podemos acrescentar um objeto
de cada tipo na seleção, acabando por escolher 2n 4-1 objetos, o que é
contabilizado pelo coeficiente de x2n+1 em /(x). Assim, os coeficientes
em questão são realmente idênticos.
152 Problemas Resolvidos de Combinatória

Agora, temos que:

= (1 4- x 4- x2 4- • • • 4- xn-1)3
/I -xn 3
\ 1 —X
= (1 — 3xn 4-3x2n - x3n)(l - x)-3.

Então, para se encontrar o coeficiente de x2n~2 em p(x), basta que


se encontrem os coeficientes de x2n~2 e de xn~2 em (1 — x)-3. Pelo
Exercício 152, o coeficiente de x2n~2 em (1 — x)~3 corresponde ao
valor de = (2£2,) = (T) ’ e ° coeficiente de xn~2 é igual a
/3+(n-2)-l
k n—2 ) = (...n2_.2) = (2)- Portanto, o coeficiente procurado é igual
a(22")-3®.
155. (a) Cada um dos seis tipos de balas tem distribuição controlada por um
polinômio do tipo:

x25 + x50 4-x75 + x100 ‘ +x■50


x25(l +x:25 J + x75)
x25(l-x:100)(l - X25)-1,

observando-se que temos de selecionar de um a quatro pacotes de cada


tipo e que cada pacote contém 25 balas. Portanto, como são seis os
tipos de balas, a resposta ao nosso problema é igual ao coeficiente de
x2'5 na expansão de:

[x25(l-x100 )(l-x,25)-l]6 =
_ t150 (1 — X:100)6(l — x::25)-6

= (x150 - 6x:250 + 15x' :25r6.


Assim, o valor procurado é igual a uma vez o coeficiente de x125 = x5’25
menos seis vezes o coeficiente de x25 = x1-25
1-25 na expansão de (1—x25)-6.
Pelo Exercício 152, este valor é (6+|-1) — 6(6+|-1) = (19) — 6(®) = 216.
)-«('
(b) O polinômio que controla a distribuição dos livros de Matemática para
um dos alunos é igual a:

x2 + x3 +---- 1- x11 = x2(l + x 4-------1- x9)


x2(l — x10)(l — x)-1.

Limitamos a 11 o número máximo de livros distribuídos, nesse caso,


porque o outro aluno, que ficará com os livros restantes, também deve
Parte II. Resoluções - Funções geradoras e partições 153

ficar com pelo menos dois livros de Matemática. Analogamente, os


de Português e os de História são controlados por x2(l — x7)(l — x)-1
e x2(l — x14)(l — x)-1, respectivamente. Assim, a resposta ao nosso
problema será o coeficiente de x20 na expansão de:

[x2(l — x 10)(l-x) ^[x^l - x7)(l - x) ^[x^l —x »)(1 - x)-1) =


= x6 (1 - x7)(l - x10)(l - xl4)(l - x)"3
= x6(l — a:7_a.10_a.14 + ...)(1_a.)-3

= (x6 — X13_a.16_
— ax.20 + ...)(1_a.)-3

Portanto, o número de distribuições de livros ao aluno considerado é


igual a (3+}J-1 '
i-m-r**n-f '3+0—1
0 ) = 120-36-15-1 = 68.
Efetuando tal distribuição, os livros do outro aluno ficam, também,
determinados. Logo, 68 é a resposta ao nosso problema.

156. Primeiramente, observe que (1 — x2)n = E7=o(—(7)x2t- Além disso,


(1 - x)-" = E”=o(-1)>(7)^ = E”=o C pelo Exercício 152, e
também (1 + x)n = Ek=o (k)2^- Assim, como forma alternativa à iden-
tidade fornecida no enunciado, temos que (1 + x)n = (1 — x2)n(l — x) —n
donde: n n
n + j- 1
xj.
j=G X n — 1

Obviamente, o coeficiente de xm deve ser o mesmo em ambos os membros


da igualdade acima. Todavia, pelo lado esquerdo, ele é igual a (^) e, pelo
lado direito, é igual a:

f-i V (n + m - 2^ — 1 n \ Zn — 1\
1 n-1 m/2/ \n — 1/
n + m — 21 — 1
1=0 \ / \ n—1

Portanto, a fórmula está demonstrada.


1 157. (a) A função geradora (1 + x)(l + x2)(l + x4) • • • (1 + x2*) • • • é tal que
o coeficiente de xn refere-se ao número de maneiras de se expressar n
como uma soma de potências distintas de 2, a saber, 1 = 2o, 2 = 21,
4 = 22, ..., 2k, ... Assim, para provarmos que, para cada n, existem
154 Problemas Resolvidos de Combinatória

potências de dois cuja soma resulta n e, ademais, que tal existência é


univocamente determinada, basta provarmos que o coeficiente de xn,
para cada n, é igual a 1, isto é, temos de provar que a função geradora
acima é igual a 1 4- x + x2 4- • • • = . Mas observe que:

(1 — x)(l + x)(l 4- x2)(l 4- x4) • • • (1 + x•2/c)... =


= (1 — x2)(l + x2)(l 4- x4) • • • (1 + x
= (l-x4)(l + x4)---(l + x2fc)-.- .

Assim, dado p G N, tome k G N tal que k > log2 p. Ocorre que, em


algum momento, o produto acima se reduz a (1 — x2<:)(l + a?2*" ) • • •,
sendo 2k > 21o^p = p, ou seja, o coeficiente de xp no desenvolvimento
de tal produto é igual a 0, para cada p. Logo, o produto acima se
reduz a 1, e está provado o que queríamos.
(b) Dada uma base b como no enunciado e um inteiro n, desejamos provar
que existem os inteiros a; G {0,1, 1} satisfazendo a relação
n = aob° + fliò1 4- (12b2 4- • ■ •. Isto é equivalente a averigüar a existên­
cia de soluções para a equação n = xq 4- x\ 4- X2 4- ■ ■ •, sujeita à
x0 G {0, l,...,ò-l},xi G {0, b,..., (ò-l)ò}, x2 G {0, b2,..., (b—l)b2},
e, assim, sucessivamente. Ora, a função geradora que controla tais
soluções é igual a:
(1 4- X 4------ F X6-1 )(14-xò4----4-x^-1^)-
•(1 + x62 + ... + ^-1)^)...
(1- a?2) (1 - xfc3)
(1 -x) (1—0?) (1 — X62)
1
1—X
= 1 4- x 4- x2 4---- .

Assim, o coeficiente de xn, na expansão acima, é igual a 1, donde, para


cada n, existe uma única solução para a equação dada. Portanto, o
resultado está demonstrado.
158. Estabeleceremos aqui uma relação biunívoca entre o conjunto das partições
de um inteiro em partes distintas e o das partições do mesmo em partes
ímpares.
Supondo que n esteja particionado em partes ímpares, temos sua expressão
dada por ai parcelas iguais a 1, as parcelas iguais a 3, e, assim, sucessiva-
Parte II. Resoluções - Funções geradoras e partições 155

mente, até a2m+i parcelas iguais a 2m 4-1. Isto é:

n = I 4- 4-1 + 3 + ... + 3 + ... + (2m 4- 1) 4- • • • 4- (2m 4- 1)


ai parcelas 03 parcelas aim+i parcelas

= Ui • 1 4- 03 ■ 3 4- • • • 4- a2m+i(2m 4- 1).
Agora, pelo Exercício 157, sabemos que cada um dos a^s pode ser escrito
de maneira única como soma de potências distintas de 2. Então, temos:

n - (2X11 + 2X12 + • • • + 2X1“i )1 + (2X31 4- 2X32 -I------- 1- 2X3°3 )3 4-----


-|- (2a:(2m+1)1 + 2x<2m+1>2 4----- 4- 2I(2m+1)“2m+1 )(2m 4- 1) (7'
= 2Xn 4- 2X12 4- • • • 4- 2xi“i 4- 3 • 2X31 + 3 ■ 2X32 4------- 1- 3 • 2X3a3 4- •
4- (2m 4- l)2x<2”>+rii + (2m 4- 1)2X(2’"+D2 4- • • •
4- (2m 4- l)2X(2m+1)a2m+i.

Ora, esta é uma partição de n composta de partes duas a duas distintas.


De fato, duas potências de dois na expansão acima só podem ser iguais se
multiplicadas por ímpares distintos.
Agora, dada uma partição de n em partes distintas, podemos expressar cada
uma de suas partes como o produto de um número ímpar por uma potência
de 2. Em seguida, colocando juntas todas as partes que contiverem fatores
ímpares idênticos, e somando-se as potências de 2 que surgirem, obtemos
justamente a expressão de n exibida em (7), ou seja, uma partição de n
composta somente de parcelas ímpares, concluindo o caminho inverso ao
que já tínhamos feito.

159. (a) Suponhamos uma dada partição de n cuja maior parte é igual a k.
Representando-se tal partição num Diagrama de Ferrers25, sua primei­
ra linha, claramente, deve possuir k pontos. Em seguida, tomemos sua
partição conjugada26. Ora, este novo diagrama possui k Unhas, donde
a partição deve possuir k partes. Assim, a cada partição de n cuja
maior parte é fc, associamos outra que possui k partes. Similarmente,
dada uma partição de n em k partes, tomando sua partição conjugada,
obtemos uma partição de n cuja maior parte é igual a k. Assim, está
demonstrado o que queríamos. Observe que pequenas alterações na
demonstração anterior poderão nos conduzir ao fato de que o número
25Caso não esteja familiarizado com este conceito, consulte a referência bibliográfica [13],
26 A partição conjugada de uma dada partição é aquela cujo diagrama de Ferrers é obtido
trocando-se cada linha do diagrama original por coluna. Observe que esta transformação dá
origem a uma nova partição de n, já que o número total de pontos é preservado.
156 Problemas Resolvidos de Combinatória

de partições de n que possuem partes de tamanho máximo k é igual


ao número de partições de n em, no máximo, k partes.
(b) Considere a figura que vem em seguida:

Considere-se, então, uma partição autoconjugada de n. Construindo-


se o seu Diagrama de Ferrers, pode ser visualizada uma outra partição
de n, em ímpares distintos, da seguinte maneira: a primeira parte
corresponde, na figura, ao número de pontos que se encontram no ‘L’
externo, a segunda parte corresponde ao segundo ‘L’, de fora para
dentro, e, assim, sucessivamente. Observe que, agindo dessa forma, as
partes obtidas são, de fato, ímpares distintas, uma vez que as partições
originais são auto conjugadas. Por exemplo, na partição da figura,
referente ao número 44, temos a partição autoconjugada:

44 = 10 4-9 + 64-5 + 4 + 3 + 2 + 2 + 2 + 1,

à qual associamos a partição:

44 = 19 + 15 + 7 + 3.

Reciprocamente, dada uma partição de n em ímpares distintos, pode­


mos construir seu Diagrama de Ferrers em ‘L”s, de tal maneira que,
realizando a leitura tradicional do Diagrama, temos uma partição au-
toconjugada de n.
(c) Dada uma partição de n na qual aparece j, desconsiderando esta parte,
obtemos uma partição genérica de n — j. Reciprocamente, dada uma
partição de n — j, acrescentando-se a ela a parte j, obtém-se uma
partição de n em que aparece j. Assim, existe uma relação biunívoca
entre o conjunto das partições de n — j e o das partições de n nas quais
j aparece como parte.
Parte II. Resoluções - Funções geradoras e partições 157

160. (a) Caso não houvesse recomendações de se fazer uso de Funções Gerado­
ras para resolver este problema, o mesmo tornar-se-ia trivial. De fato,
para expressarmos n como soma de duas partes, digamos a eb, basta
que façamos com que a percorra valores entre 1 e [^J.
Primeiramente, vamos calcular o número de partições de n em, no
máximo, duas partes. Como vimos no final da resolução do item (a)
do Exercício 159, este número é o mesmo que o de partições de n
cujas partes são limitadas por 2. É fácil ver que a função geradora
que controla o número de partições de n em partes iguais a 1 ou 2 é a
função:
1
(1 — x)(l — X2)
Para resolvermos o problema, então, devemos calcular o coeficiente de
xn na expansão acima. Observe, porém, que:

1 1 1
(1 — x)(l — X2) 2(1 -x)2
+ 2(1 -x2)'

Logo, devemos encontrar o coeficiente de xn em cada um dos termo:,


do lado direito da expressão acima. Pelo Exercício 152, o coeficiente
de xn em é igual a |( 2+n-l
n ) = 5 CF) = Hp- Além disso’
observe que:
1 = i(l+x2 + x4 + •••).
2(1 - x2)

Assim, neste membro, o coeficiente de xn é igual a |, se n for par, e 0,


caso contrário. Logo, se n for par, então o coeficiente de xn na soma
acima é igual a 2^- + | = f + 1 = + 1. Por outro lado, se n for
ímpar, então esse coeficiente é igual a 2±1 = 4-1. Como dissemos,
este é o número de maneiras de se particionar n de modo que nenhuma
parte seja maior do que 2, que é igual ao número de maneiras de se
particionar n em, no máximo, duas partes. Como só há uma partição
de n constando de uma única parte (a saber, n = n), de fato, o número
de partições de n em exatamente duas partes é igual a [^J.
(b) Sabemos que a função geradora que controla o número de partições de
n em partes distintas e não múltiplas de 3 é a função:

oo

n(i + x3i+I )(1 + x^


: +2)>
158 Problemas Resolvidos de Combinatória

e que a função geradora para o número de partições de n em partes


da forma 6i + 1 ou 6i + 5 é dada por:
°o 1
n (1 -X6Í+1)(1 _ S.6Í+5)-

Assim, se demonstrarmos a igualdade entre estas duas funções gerado­


ras, o problema está terminado. Antes, porém, observe que os números
da forma 3i +1 são exatamente os da forma 6i + 1 e os da forma 6i + 4
juntos. Analogamente, os da forma 3i + 2 são os da forma 6i + 2 e
6i + 5 juntos. Assim, passa a ter justificativa, também, a penúltima
das igualdades abaixo:
oo

IP1**:3i+1)(l +x:3i+2)
t=i
: =

_ TT (1 + #3l+1)(l + a:3l+2)(l — x3l+1)(l — x3í+2)


“ 11 (1 -z3’+1)(l -:r3i+2)

(1 - x6í+2)(l - x6i+4)
“ 11 (i _ X3Í+1)(1 _ 3-31+2)
OO oo
1
:6i+2)(l-x' (l-a:3*+i)(1-a;3i+2)
í=l i=l
oo

= ID -
Í=1
■6i+2)(l - x',6i+4) .

°o i
n (i - a^+lXi - X6i+4)(i - x6í+2)(1 _ 3»6i+5)

o° 1
=n (i-x6i+i)(i-x6i+5) ’

e o resultado está demonstrado.


161. (a) Dada uma partição de n em exatamente k partes, há, para ela, duas
possibilidades disjuntas: possuir 1 como parte ou possuir todas as
partes maiores do que 1. Caso 1 seja uma de suas partes, desconside-
rando-a, temos uma partição de n— 1 em k — 1 partes, o que é contado
por qk-i(n — !)• Por outro lado, se 1 não figurar entre suas partes,
retirando uma unidade de cada uma de suas k partes, obtemos uma
partição de n — k em k partes, o que é contado por — fc).
Parte II. Resoluções - Funções geradoras e partições 159

Reciprocamente, dada uma partição de n — 1 em k— 1 partes, podemos


acrescentar-lhe uma parte igual ala fim de obtermos uma partição
de n em k partes tal que 1 aparece. Assim, o conjunto das partições
cuja cardinalidade é igual a Çfc_i(n — 1) está contido no conjunto das
partições cuja cardinalidade é gfc(n). Ainda, dada uma partição de
n — k em k partes, podemos acrescentar 1 a cada uma de suas partes,
obtendo uma partição de n em k partes, todas maiores do que 1.
Portanto, o conjunto das partições contadas por qk{n — k') está contido
no das partições contadas por Qfc(n). Assim, a igualdade do enunciado
é, realmente, válida.
(b) Pelo item anterior, temos que:

Çk(n) = qk-i- 1) + Qk(n - k)


gfc_!(n-l) = çfc_2(n-2) 4-<7fc-i(n-A:)

g2(n —A;4-2) = qi(n — k 4- 1) 4- çs(^ — &)


qi(n-k+l) = qo(n - k)+qi(n - k).
0

Somando, então, todas as igualdades acima, e efetuando os cancela­


mentos possíveis, temos:
k
qk(n) = qk(n - fc) 4- Çfc-i(n - A:) 4------1- çi(n - k) = <n(n ~

como desejávamos.

162. (a) Dada uma partição de n em três partes, é claro que pelo menos
duas dessas partes devem ser menores do que pois, caso contrário,
teríamos duas partes maiores do que ou iguais a o que é uma
contradição. Assim, suponhamos que n = a + b + c, com a,b <
e acrescentemos a a e a ò, obtendo a' = a + % < % + % = n,
b' = b+ ^<^ + ^=n, e d = c < n, de modo que 2n = a' 4- b' 4- d é
uma partição de 2n em três partes menores do que n.
Por outro lado, dada uma partição de 2n em três partes menores do
que n, é claro que pelo menos duas delas devem ser maiores do que
5, pois, caso contrário, teríamos duas menores do que ou iguais a
implicando que a terceira fosse maior do que n, uma contradição.
Assim, seja 2n = a 4- b 4- c, com a, b > Tomando a' = a — > 1,
160 Problemas Resolvidos de Combinatória

b' = b — > 1 e c7 = c, temos que n = a' + b' 4- c7 é uma partição de n


em três partes. Assim, como estabelecemos uma bijeção entre ambos
os tipos de partições, provamos o resultado.
(b) Sabemos que o número de partições de 2n em n partes é igual ao
número de partições de 2n nas quais a maior parte é igual a n, pelo
Exercício 159. Assim, dada uma partição de n, acrescentando-a uma
parte igual a n, teremos uma partição de 2n em que a maior parte é
igual a n. Reciprocamente, dada uma partição de 2n tendo n como
maior parte, desconsiderando-a, teremos uma partição irrestrita de n.
Assim, o resultado está demonstrado.
(c) Dada uma partição de 2n 4- k em que a maior parte é igual a n 4- k,
retirando esta parte, obtemos uma partição genérica de n. Por outro
lado, dada uma partição de n, podemos acrescentar-lhe a parte n 4- k,
obtendo uma partição de 2n 4- k tal que a maior parte é igual a n 4- k.
Assim, para cada k, observamos que o número de partições de 2n 4- k
tais que a maior parte vale n 4- k é igual ao número de partições de n,
ficando demonstrado o resultado.
163. (a) Observe que devemos escolher n letras e ordená-las, já que estamos
interessados na formação de palavras. Portanto, a função geradora a
ser utilizada deve ser, de fato, exponencial. Desse modo, cada uma
das 5 vogais deve ser controlada pela função 1 4- x, e cada uma das 18
consoantes deve ser controlada por 1 4- x 4- 4- • • • = ex. Assim, a
função geradora cujo coeficiente an de nos dá o número de palavras
de n letras contendo no máximo uma vogal de cada tipo é a função:

(1 4- z)5e18a:

(b) Neste caso, importam os símbolos escolhidos para cada palavra e,


também, a ordem em que eles se encontram nelas. Assim, podemos
contar o número de distribuições de 14 objetos idênticos em 4 caixas
distintas de tal maneira que a primeira caixa contenha pelo menos
dois objetos e as outras caixas contenham, cada uma, pelo menos um
objeto. Em seguida, devemos multiplicar o resultado encontrado por
14!, considerando a ordem dos símbolos nas palavras. O número de
maneiras de se distribuírem 14 objetos idênticos dentre 4 caixas distin­
tas sob tais condições corresponde ao número de soluções da equação
xi 4- X2 4- X3 4- X4 — 14, com Xi > 2 e > 1, para i = 2,3,4, ou seja,
ao número de soluções de yx 4- y2 4- y$ 4- = 9, para yi > 0. Logo, o
total de maneiras de se realizar a distribuição solicitada no enunciado
Parte II. Resoluções - Funções geradoras e partições 161

é igual a (9^4 xx) • 14! = (32) • 14!, valor da ordem de 1013. Nessa última
etapa, utilizamos o resultado do Exercício 25.
(c) E trivial o fato de que o número de n-uplas possíveis de se formarem
com os elementos de {1,2,... ,k} é igual ao número de maneiras de
se distribuírem n objetos distintos em k caixas distintas. De fato, no
primeiro caso, temos k opções de preenchimento para cada posição.
Logo, kn é o número de n-uplas. No segundo, temos k escolhas de
caixa para colocar cada objeto. Assim, o número de distribuições é,
também, kn. E interessante observar que este é, ainda, o número
de funções existentes de um conjunto de n elementos em um de k
elementos.
Assim, considerar a contagem das distribuições dos 14 símbolos nas 4
palavras de modo que a primeira contenha pelo menos dois símbolos
e cada uma das outras contenha pelo menos um símbolo equivale a
considerar a contagem das 14-uplas formadas por elementos do con­
junto {1,2,3,4} nas quais o número 1 aparece pelo menos duas vezes,
e os números restantes aparecem, pelo menos, uma vez. Ora, a função
geradora para a contagem de tais 14-uplas é a função:
X2 3
(x2 x3 \ /
(ã+3T + "J V X+ 2! + "'
= (e1 - x - iXe1 - 1) 3
= (ex — x — l)(e3a: — 3e,2x + 3ex _
= e4x — xe3x — 4e3x + 3xei2x
2* + 6e2x — 3xex — 4ex + x + 1.
14
Estamos, portanto, interessados no cálculo do coeficiente de yyy na
expansão acima. De maneira simples, o valor procurado é igual a:
414 - 14 • 313 - 4 • 314 + 42 • 213 + 6 • 214 - 42 - 4 = 227.425.380.

164. (a) Como já vimos em exercícios precedentes, esta quantia equivale ao


número de soluções inteiras não negativas da equação xi -|------ l-zt = n,
já que efetuar a distribuição equivale a apenas determinar quantos são
os objetos postos em cada caixa (uma vez que eles são idênticos).
Assim, o número de distribuições é igual a ~ ( Se
nenhuma caixa puder ficar vazia, o número de distribuições torna-se
igual ao número de soluções inteiras positivas da equação acima, ou
seja, (fcZÍ)- Neste item, utilizamos os resultados do Exercício 25.
(b) O número de maneiras de se distribuírem n objetos distintos em k
caixas distintas desprezando a ordem dos mesmos em cada caixa é
162 Problemas Resolvidos de Combinatória

idêntico ao número de funções f : A —*■ B, com |A| = n e |B| = k. De


modo simples, vemos que este número é igual a kn (reveja o Exercício
138). Caso a ordem dos objetos em cada caixa deva ser levada em
consideração, basta que encontremos o número de distribuições de n
objetos idênticos em k caixas distintas, como fizemos no item anterior,
e, em seguida, multipliquemos o resultado encontrado por n!. Assim,
(Hfc*?1) ’ n- = ® ° número procurado. Finalmente, caso a or­
dem dos objetos em cada caixa seja irrelevante, mas não possa haver
caixas vazias, temos tantas distribuições quantas são as funções so-
brejetoras f : A —> B tais que |A| = n e |B| = k. Como vimos no
Exercício 138, este número é igual a T(n, k) = 52*=o(—1)*(í)(^ — 0n-
(c) Para distribuirmos n objetos distintos em k caixas idênticas sem deixar
nenhuma vazia, basta que façamos a distribuição considerando as
caixas distintas, como no item (b), e, em seguida, desprezemos sua
ordem. Assim, o número procurado é igual a = S(n, fc), quantia
que é denominada número de Stirling de segunda espécie, equivalente
ao número de maneiras de se particionar um conjunto de n elemen­
tos em exatamente k subconjuntos não vazios. Se forem permitidas
caixas vazias, basta que variemos k na expressão de S(n, k), isto é,
somemos as distribuições que consideram entre 0 e k — 1 caixas vazias.
Assim, S(n, i) é o número procurado, equivalente ao número de
partições de um conjunto em, no máximo, k partes.
(d) O número de maneiras de se distribuírem n objetos idênticos em k
caixas idênticas é igual ao número de partições de n em, no máximo,
k partes, valor que denotaremos por Pk(ri)- Explicitamente, Pk(n)
corresponde ao coeficiente de xn na expansão de — x1)-1.
Analogamente, se nenhuma caixa puder ficar vazia, então o número
de distribuições corresponde ao de partições de n em exatamente k
partes, número denotado por Çfc(n), e igual ao coeficiente de xn na
expansão de xk nLi(i — x1} 1, isto é, igual ao coeficiente de xn k em

165. Observe, primeiramente, que o seguinte fato é equivalente à identidade


fornecida pelo enunciado da questão:
___________ 1___________
1 —x
(1 + x)(l 4- x2)(l + z4) ■ • •
= (1 — x 4- x2 — x3 4- • ■ • )(1 — x2 + x4 — x6 4-----)
• (1 — x4 4- x8 — x12 + • (8)
Parte II. Resoluções - Funções geradoras e partições 163

Tal fato nos informa que, para r > 1, o coeficiente de xr na expansão de


(8) é igual a 0. Observemos, também, que a função:

(1 + i + x2 + ••■)(! + i2 + s4 + -")(l + i4 + r8 + ••■)•■•

gera o total de partições de um número natural em potências de 2. Con­


siderando, pois, a expansão de (8), cada termo é igual ao produto entre
fatores obtidos de cada uma das expressões entre parênteses, sendo que,
cada vez que tomamos um fator de coeficiente positivo, aparece, na partição
considerada, um número par de partes que são potências de 2. Da mesma
forma, sempre que tomamos um fator de coeficiente negativo, aparece um
número ímpar de partes que são potências de 2. Assim, dada a observação
anterior de que, para r maior do que 1, o coeficiente de xr na expansão de
(8) deve ser nulo, temos que, para cada r, o número de fatores positivos
deve ser igual ao número de negativos e, portanto, o número de partições
de r em potências de 2 contendo um número ímpar de partes deve ser igual
ao número de partições contendo um número par de partes.

166. Considere-se o Diagrama de Ferrers de uma partição de r + k em k partes.


Em seguida, acrescentem-se k — 1 pontos a sua primeira linha, k — 2 à se­
gunda, e, assim, sucessivamente, até que se acrescente 1 ponto à penúltima
linha e nenhum à última. Assim, obtemos uma partição do número
r + fc + (fc — 1) 4- (fc - 2) + • • • + 1 + 0 = r + k(k+Ü = r + x) em k
partes distintas, uma vez que acrescentamos um número diferente de pon­
tos em cada linha.
Da mesma forma, considerando o gráfico de uma partição de r + (fc2l)
em k partes distintas, podemos retirar k — 1 pontos da primeira linha,
k — 2 da segunda, e, assim, sucessivamente, até retirarmos 1 ponto da
penúltima linha e nenhum da última. Assim procedendo, obteremos uma
partição de r + k em k partes. Observe a importância do fato das partes de
r + (^2x) serem distintas, nesse caso, permitindo-nos ter obtido, realmente,
um Diagrama de Ferrers de uma partição associada a r + k.

167. (a) A rigor, estamos interessados no número de partições de n em três


partes sujeitas a algumas restrições. Dado que, em partições, não
se deve considerar a ordem das partes, temos n = a + b + c, com
1 < a < 6 < c, e a, ò, c e N. Além disso, como condição necessária à
existência de triângulos, devemos ter a + b > c, ou, ainda, a > c — b.
Como artifício à resolução do problema, consideremos a relação dada
por n = 3a + 2(ò — a) + (c — ò). Fazendo as substituições y = b — a
e z = c — b, as restrições do problema tornam-se y > 0, z > 0 e
164 Problemas Resolvidos de Combinatória

a > z (donde a = z + x, para x > 1). Desta forma, resulta que


n = 3(z + x) + 2y + z = 3x + 2y + 4z. A função geradora que controla
o número de soluções da última equação e, portanto, a resposta ao
nosso problema, é a função:
(x3 + x6 + • • • )(1 + x2 + x4 -I----- )(1 + x4 + x8 d----- ),
ou, de modo mais sintético:
X3

(1 — x2)(l — x3)(l — X4)


A título de curiosidade, existe uma expressão fechada simples para an,
isto é, para o coeficiente de xn na expansão acima, a saber, o número
de triângulos incongr uentes de lados inteiros e perímetro n. A dedução
dessa expressão encontra-se no Apêndice B de [13], resultando:
n+2
a>n - {5} - 15J ~T~ ’
sendo notação para o inteiro mais próximo de • P°r exemplo,
o número de triângulos incongruentes de lados inteiros e perímetro 20
é igual a:
f 202 1 | 20 I | 20 + 2
= [ 4
= 33 - 5 • 5 = 8.

Segue uma ilustração desses 8 triângulos:

9 9

(b) A diferença deste item em relação ao anterior é que, neste caso, basta
que, em vez de a + b > c, tenhamos a + b > c. Assim, da mesma forma,
teremos y = b - a > 0, z = c- b > 0 e a > z. Nesta etapa, contudo,
a resolução diverge da do item anterior, pois teremos que dividir o
problema em dois casos:
Parte II. Resoluções - Funções geradoras e partições 165

i. z = 0, equivalente ao fato de as duas partes maiores serem iguais


(c = b);
Neste caso, teremos a = x > 1 e, assim, n = 3x 4- 2y, equaçao que
tem o número de soluções controlado pela função geradora

x3
(x3 + x6 4----- )(1 4- x2 + x4 4------ ) =
(1 — x:‘2)(1 — x3)

ii. z > 1, equivalente ao fato de as duas partes maiores serem distin­


tas (c > b).
Neste caso, teremos a = z+x, para x > 0 e, assim, n = 3x4-2y4-4z,
equação que tem o número de soluções controlado pela seguinte
função geradora:

(1 + x3 + x& 4----- )(1 + x2 + x4 4------ )(x4 4- x8 4- • • •) =


x4
(1 — x2)(l — x3)(l — x4)

Agora, dado que as duas funções controlam objetos pertencentes a


conjuntos disjuntos, a saber, aqueles em que c = b e aqueles em que
c > ò, a função geradora cujo coeficiente de xn é bn corresponde à
soma das duas acima encontradas:
x3 x4
I ■
(1 — x‘2)(l-x 3) 4-
+ (1 -x 2)(l -x 3)(l - x4)
x4 4- x3 — x7
(1 - x2)(l — x3)(l - x4) ’

168. (a) Suponhamos um objeto particular x dentre n 4-1 objetos iniciais. Com
relação à distribuição de tal objeto dentre as k caixas, ou ele ficará
sozinho numa delas, ou ele ficará acompanhado de ao menos um objeto.
No primeiro caso, só há uma maneira de depositá-lo sozinho numa das
caixas, uma vez que elas são idênticas, e há, por definição, S(n, k — 1)
maneiras de distribuirmos os n objetos restantes nas outras k — 1
caixas idênticas. No segundo caso, podemos primeiramente distribuir
os n outros objetos entre as k caixas de S(n, k) maneiras. Uma vez
ocupadas por objetos distintos, as caixas passam a ser distinguíveis,
havendo k possibilidades para o acréscimo de x a uma delas. Logo,
S(n 4- 1, Â:) = S(n, k — 1) 4- kS(n, k). Observe ainda que, como são
triviais os fatos de que, para cada n, 5(n,n) = S(n, 1) = 1, a relação
anteriormente provada torna o cálculo de 5(n, fc) possível para cada
166 Problemas Resolvidos de Combinatória

par (n, fc), através de seguidas aplicações da relação encontrada. Além


disso, aplicando seguidamente a relação de recorrência acima, nota-se
que é coerente definir, para cada n, S(n, 0) = 0.
(b) Sabemos, do Exercício 164, que:
- k /, \
S(n, k)

Logo, a função geradora procurada é igual a:


00 n 00 , k

I2s(n,ky-
n=0
oo
[(fc - i)r]n
n=0
n\

kí i=0 x 7
fc Z1 \
1
~ Zc’
’ í=o
(e1 - l)fc
k\
utilizando, na última passagem, a Fórmula do Binômio de Newton.
(c) Agora, por informação do enunciado da questão, temos que:
n n
Bn = S^n' = S(n’
fc=l fc=0
uma vez que 5(n,0) — 0. Logo, utilizando, ainda, o resultado do item
(b), temos:
k
-1)— = étex — 1
Z3 Bn^\’ = n=0fc=0
n=0
ZL è ZZ
= fc=0 fc!
Expandindo a função geradora acima num software matemático, obte­
mos:
X° X1 X2 X3 X4 X5 X6
= 1Õ!+1ir+22!+53!+154!+525!+2036r + -
Assim, por exemplo, Bq = 203, isto é, existem 203 partições de um
conjunto de 6 elementos.
Parte II. Resoluções - Funções geradoras e partições 167

l
169. (a) Considerando que se dispõe de um número ilimitado de cada um dos
dígitos em caixas separadas, o problema equivale a retirar dígitos das
4 caixas de tal maneira que a ordem da retirada é levada em conta.
Além disso, deve ser retirada uma quantidade par de 0’s e ímpar de l’s.
Portanto, é necessária a utilização de funções geradoras exponenciais
controlando a presença de cada um dos tipos de dígitos na seqüência.
A função que controla a presença do dígito 0 é igual a:

„ X2 X4 1 ,
1+2!+4!+- = 2(e +C >'

Por outro lado, o dígito 1 é controlado por:

x3 x5 l.x
*+ 3! + 5T + "“2(e

Finalmente, os dígitos 2 e 3 são controlados por:


i2
1 + x + — + ■■• = e1.

Portanto, o número de r-seqüências nas condiçoes do enunciado é igual


ao coeficiente de na expansão de:

- e-*)e2a: = J<^ -1),


donde a resposta ao nosso problema é | • 4r = 4r 1.
(b) De fato, a resposta final do item (a) sugere a existência de uma solução
mais simples (ou que, pelo menos, exija menos contas) para aquele
item. Sabemos que o total de r-seqüências quaternárias sem restrições
é igual a 4r. Nas seqüências que devem ser contadas, se identificarmos
0’s e l’s por a’s e 2’s e 3’s por b’s, como o total de 0’s e l’s deve
ser ímpar, estamos interessados apenas no número de seqüências que
contêm um número ímpar de a’s e um número qualquer de ò’s.
Vamos provar que tal número é igual à metade do total, isto é, Ç. Para
entendermos este fato, recorreremos à maneira como são construídas
as seqüências de a's e b's com um número ímpar de a's. Para tanto,
adotaremos raciocínio indutivo, em função do tamanho das seqüências.
Para as seqüências de somente um dígito, vale o que desejamos provar.
De fato, das duas possíveis (aeò), somente uma possui número ímpar
de a’s, a saber, a seqüência a. Agora, supondo que o mesmo fato
168 Problemas Resolvidos de Combinatória

valha para r — 1-seqüências, provaremos que ele também vale para r-


seqüências. Realmente, a partir das r— 1-seqüências, pode-se construir
todas as r-seqüências acrescentando-se um dígito a ou um dígito b ao
final de cada uma das r — 1-seqüências originais. Destas, por hipótese,
metade possuía um número ímpar de a’s e a outra metade possuía
um número par de a’s. Ao acrescentarmos um a a cada uma das
componentes da primeira metade, estas passam a ter número par de
a’s, e, fazendo o mesmo com a outra metade, estas passam a ter número
ímpar de a’s, ocorrendo, dessa forma, um equilíbrio no número total de
seqüências com número par e ímpar de a’s. Com relação ao acréscimo
dos ó’s, por sua vez, também ocorre equilíbrio, uma vez que o número
de a’s, nesse caso, não se altera. Portanto, está provado que o número
de seqüências com número ímpar de a’s é igual à metade do total de
seqüências de a’s e &’s.
Ademais, para que o total de a’s seja ímpar, devemos ter um número
pai- de 0’s e um número ímpar de l’s ou um número ímpar de 0’s e um
número par de l’s. Ambos os casos são simétricos, uma vez que 0’s e
l’s têm direitos iguais! Como estamos interessados em contar apenas
as r-seqüências do primeiro, o total das seqüências deve ser dividido
por dois novamente. Logo, o número de r-seqüências quaternárias com
um número par de 0’s e um número ímpar de l’s é igual a = 4r“1,
coincidindo com o valor encontrado no item anterior.

170. (a) Se nenhum dígito deve ocorrer exatamente duas vezes, a função gera­
dora que controla a presença de cada um deles é igual a:

z3 x4 .X
x2
1 + a:+3!+¥ + "=ff 2?
Logo, o número que procuramos é o coeficiente de yy na expansão de:

9 3 T2
X rr4 T
ex
T = é,3x - 33—
2C + 3— e
4 8

Assim, se r ' 6, então o coeficiente


r! de na expansão acima é igual
a:
3r(r — l)(r — 2)(r — 3)
3r - 3r(r - l)2r~3 +,

Para r = 6, basta que se subtraia 90 do resultado obtido para o caso


r / 6, uma vez que 90 •
Parte II. Resoluções - Funções geradoras e partições 169

(b) Os dígitos 0 e 1 são controlados pela função geradora exponencial


^(ex + e~x), e o dígito 2 é controlado por ex — Assim, resolver o
problema é encontrar o coeficiente de na expansão de:

1
#4)- X'
6
x3
+ 2 + e~2x ex
= 4 'T
.3 -r3
1.1 X' ,2x
37 e~2x
— -e
=
- 4 3x + 26x +
+ -e 24ee21 — —
4C x — —
4- -e~ 24 6e-2* - —
12'
Efetuando agora os cáculos, para r 3, temos como resposta o coefi­
ciente:
3r 1 f-l)r , ... _2nr
r —3 or-3
- r(r - l)(r - 2)— + (-l)r-2r(r - l)(r - 2)—,
4 2 4 v M 7 24 ' '
ou, pondo em evidência os termos adequados:
(—l)r + 3r + 2 r(r — l)(r — 2)2r~3 . ,
4 24 k k h
Para o caso r = 3, basta subtrair | do resultado precedente, já que
1 X3 __ X3
2 ' 3! — 12-

171. (a) Observe que, para que tenhamos soma par, o número de faces ímpares
que aparecem deve ser par. A função geradora que controla o número
de faces pares é a função que, na expressão fornecida pelo
enunciado deste item, está fora dos colchetes. De fato, sabemos que:
1
= (1 + X + x2 + .. .)(1 + X + x2 + .. .)(1 + X + x2 + . . .).
(1 — x)3
Observe, então, que os três termos entre parênteses, à esquerda, con­
trolam o número de faces iguais a 2, 4 e 6, respectivamente.
Agora, vamos mostrar que a expressão que está dentro dos colchetes
é a função que controla o número de faces ímpares, de tal maneira
que elas apareçam, em conjunto, um número par de vezes. De fato,
observemos que:
1 ___ 1
(1 — x)3 +
+ (1 + x)3
= (1 + x + x2 + .. .)(1 + x + x2 + .. .)(1 + x + x2 + . . .) (9)
+ (1 - X + x2 - .. .)(1 — x + x2 — . . .)(1 - X + x2 - ...). (10)
170 Problemas Resolvidos de Combinatória

Assim, obter um termo de expoente ímpar na expansão de (9) significa


que o lançamento dos dados resultou num número ímpar de faces
ímpares. Ora, mas este termo é sempre cancelado pelo seu correspon­
dente na expansão de (10). Por exemplo, x2 • x4 • x3 equivale a um
lançamento que resultou em duas faces iguais a 1, quatro iguais a 3
e três iguais a 5. Note que há um número ímpar de faces ímpares,
e este termo é cancelado, na soma, por x2 • x4 • (—x3). Por outro
lado, se o número de faces ímpares for par, então o termo é contado
duas vezes na soma. Por exemplo, x • x2 ■ x3 é contado, também, por
(—x) -x2 • (—x3). Isso explica, portanto, a aparição da fração à frente
da expressão original, e conclui a demonstração.
(b) Neste caso, o número de faces ímpares deve ser ímpar. Pelo mesmo
raciocínio apresentado na resolução do item (a), com relação aos ter­
mos que aparecem na expansão da expressão que está entre colchetes,
precisamos contar duas vezes os que possuem expoente ímpar e cance­
lar os que possuem expoente par. Assim, uma ligeira modificação na
expressão exibida no item (a) resulta na resposta deste:

1 1 1 1
2 (1 — x)3 (1 — x) 3 (1 + x)3_

(c) Primeiramente uma explicação acerca da seqüência enunciada. Supo­


nha uma festa que comece com dois convidados e, a cada momento,
chegue um novo convidado. E praxe que todos os presentes se cumpri­
mentem uma vez. Então, com dois convidados, há apenas 1 cumpri­
mento. Quando chega o terceiro convidado, este deve cumprimentar os
dois já presentes, resultando num total de 3 cumprimentos; chegando
o quarto, há mais três cumprimentos, resultando em 6 cumprimentos.
Assim, suponha que Cn represente o número de cumprimentos após a
chegada do n + 1-ésimo convidado. No momento em que chega, ele
deve cumprimentar os n já presentes, adicionando n aos Cn-i cumpri­
mentos anteriores. Assim, Cn = Cn-i +n, com ci = 1, coincidindo com
a relação de recorrência que define a seqüência dada no enunciado.
Portanto, tn pode ser visto como o número total de cumprimentos en­
tre n + 1 convidados de uma festa. Ora, é trivial que este número é
igual a (nJX), para n = 1,2,3,... Portanto, tn = (n£1) é a resolução
da relação de recorrência dada.
Agora, como facilitação ao que desejamos provar, vamos subtrair a
Parte II. Resoluções - Funções geradoras e partições 171

expressão encontrada em (b) da expressão encontrada em (a), obtendo:


oo

12
n=0
n bn)x —

OO oo
,n
= ^2 an^ 12 bnx,n‘
71=0 71=0

_ 1 1 _______
1
2 (1 — x)3 (1 — x)3
+
1 1 _______
1
2 (1 — x)3 (1 — x)3
_ 1 1 1
_______ 1
2 (1 — x)3 [(1 — x)3
+ (1 — x)3
+
=1. 1 -2- 1
2 (1 — x)3 (1 + x)3
1
(1 — x2)3
OO

=E
r=0
(-l)rx2r.

Através da simples observação desta última fórmula, decorre que os


coeficientes de xn, para n ímpar, são iguais a zero, donde an — bn = 0.
isto é, an = bn. Por outro lado, se n é par da forma 2r, seu coeficiente
é dado por:

(_l)r = ( 1}r —3(—3 — 1)(—3 — 2) • • • (—3 — r + 1)


r!
(r + 2)!
= (-l)2r
2!r!
/r + 2
~ \ 2 .
= ^r+l

= ^f + l-

Assim, se n for par, an — bn = t|+i, donde a n = bn + t«+l, provando


o que queríamos.
172 Problemas Resolvidos de Combinatória

Princípio da Casa dos Pombos


172. (a) Sejam (xjt, Zk), 1 < k < 9, os 9 pontos considerados, de tal modo
que Xk,yk,zk G Z. Sabemos que o ponto médio do segmento que une
(xí,j/í,2í) e (xj;, yj, Zj;) pode ser calculado por:

Xj + xj yj + yj Zj + Zj
2’2’2

Assim, para garantir que Xt 2X? G Z, xí e Xj devem possuir a mesma


paridade, isto é, ambos devem ser pares, ou ambos ímpares. Pelo
Princípio da Casa dos Pombos27, dentre os 9 valores Xi, ..., xg, xg,
pelo menos 5 devem ter mesma paridade. Tomando-se, então, os 5 t/i’s
associados a tais x/s, devemos ter pelo menos 3 com mesma paridade.
Finalmente, tomando-se os 3 z;’s associados a tais 2/í’s, pelo menos 2
devem ter mesma paridade. Denotando por zr e zs estes 2 valores de
mesma paridade, pela construção realizada, yr e ys também devem ter
mesma paridade, assim como xr e xs. Dessa maneira:

Xr + Xs yr 4- ys zr + zs
2 2 ’ 2

é ponto médio inteiro do segmento que une (xr,r/r,zr) e (xs,ys,zs)-


(b) Pela maneira como resolvemos o item (a), fica simples construir um
exemplo de 8 pontos inteiros que não satisfaçam a propriedade daquele
item. Tomemos, portanto, (0,0,0), (0,0,1), (0,1,0), (0,1,1), (1,0,0),
(1,0,1), (1,1,0) e (1,1,1).
(c) Em R, bastam 3 pontos (ou números) inteiros a fim de que exis­
ta a propriedade em questão, pois pelo menos 2 desses pontos de­
vem possuir a mesma paridade. Agora, suponhamos que, em seja
necessário um mínimo de pontos. Observe que este valor, uma
vez que mínimo, deve ser ímpar. Então, pelo mesmo raciocínio apli­
cado na resolução de (a), existem pelo menos ak2L valores de mesma
paridade nas primeiras coordenadas de cada um dos ak pontos con­
siderados. Nesse caso, como já eliminamos a primeira coordenada de
cada ponto, é o menor número de pontos de Rfc-1 que obedecem
à relação desejada. Ora, este número é, por definição, ak-i- Portanto,
27 “Ao distribuirmos k + 1 pombos em k gaiolas, pelo menos uma das gaiolas deve conter
pelo menos dois pombos.” Para generalizações e mais resultados envolvendo este importante
princípio, leia o Capítulo 7 de [13].
Parte II. Resoluções - Princípio da Casa dos Pombos 173

= afc_i, donde ajt = 2ak~i ~ 1- Como ai = 3, estamos diante


de uma Relação de Recorrência bem definida. Para resolvê-la, observe
que ak — 1 = 2(afc_i — 1). Denotando ajt — 1 = bfc, temos a nova relação
bfc = 2bfc—1, com bi = 2, a qual define uma Progressão Geométrica de
primeiro termo 2 e razão 2. Logo, b^ = 2fc, donde = 2fc + 1. Assim,
como a generalização procurada corresponde a On, o número mínimo
de pontos de Rn é 2n + 1.

173. Obviamente, qualquer pessoa deve possuir entre 0 e 300.000 fios de ca­
belo. Consideremos, portanto, esses 300.001 tipos de pessoas, e agrupemos
os habitantes de São Paulo em termos do seu número de fios de cabelo.
Observando agora que 10.788.682 = 300.001 • 35 + 288.647, deve haver 36
pessoas em São Paulo com o mesmo número de fios de cabelo. De fato,
se houver no máximo 35 pessoas em cada um dos 300.001 grupos exis­
tentes, então os habitantes de São Paulo ficariam reduzidos a, no máximo,
300.001 ■ 35 = 10.500.035, o que é uma contradição. Esta é uma aplicação
do que denominamos Princípio da Casa dos Pombos. Portanto, n = 36 é a
resposta do problema.

174. (a) Suponhamos que existam 5 caixas numeradas, de tal modo que, na i-
ésima caixa, coloquemos o(s) objeto(s), dentre a, e b,, que satisfaz(em)
a propriedade P, para i = 1,2,3,4,5. Como são 5 as caixas e dois os
tipos de objetos (aj’s e b/s), pelo Princípio da Casa dos Pombos, como
5 = 2 • 2 -I-1, pelo menos três caixas devem conter, em conjunto, três
aj’s ou três bj’s.
(b) Como no item anterior, suponhamos 10 caixas numeradas, de tal modo
que, na í-ésima caixa, coloquemos o(s) objeto(s), dentre a,, bi e Ci, que
satisfaz(em) P, sendo i = 1,2,..., 10. Como 10 = 3 • 3 4-1, deve haver
pelo menos 4 objetos de um mesmo tipo (a,, b, ou c,) satisfazendo P.
Logo, n = 4.
(c) Conjeturemos que n = 4. Suponhamos, então, que não haja quatro
ai’s com a propriedade P. Portanto, deve haver dois aj’s, digamos
ai e 02, que não satisfazem P, donde bi, b2, Cl e C2 satisfazem P.
Restam-nos, pois, três bi's e três c/s. Novamente pelo exposto no
enunciado, destes, deve haver pelo menos mais dois b^s ou pelo menos
mais dois c^s satisfazendo P, donde segue o resultado. Agora, se
n = 5, não há garantia da validade da afirmação. De fato, poderiamos
ter P satisfeita apenas por ai, 02, 0.3, bi, 63, 64, bs, C2, C4, C5, não
contemplando a afirmação dada, donde concluímos que n = 4. Note,
no exemplo fornecido, que aparecem quatro b/s satisfazendo P. De
174 Problemas Resolvidos de Combinatória

fato, isto realmente devia ocorrer pelo que provamos acima.


175. Em caso extremo, poderia ocorrer de serem selecionados apenas 34 alunos
de cada unidade, num total de 34 • 20 = 680 alunos. Assim, bastaria que
fosse selecionado mais 1 aluno, para obtermos os 35 alunos de um mesmo
curso. Logo, 681 é o número de alunos que garante a realização da pesquisa
independentemente da escolha realizada.
176. (a) Acompanhe a explicação aqui exibida na figura que vem em seguida:
B

•D

•'igura 6: Os segmentos cheios representam os vermelhos, e os tracejados, os


azuis.

Observe que qualquer um dos 6 pontos deve estar ligado aos 5 pontos
restantes, formando 5 segmentos, de sorte que, como são duas as cores
disponíveis, deve haver pelo menos três desses segmentos coloridos
com uma mesma cor. Seja, como na figura acima, A o nosso ponto
de origem, determinando três segmentos vermelhos: AB, AC, AD. Se
pelo menos um dentre BC, CD e BD for vermelho, então pelo menos
um dentre os triângulos ABC, ACD e ABD é vermelho. Por outro
lado, se BC, CD e BD forem todos azuis, então BC D é um triângulo
azul.
(b) Sugerimos, neste caso, que o leitor acompanhe a explicação subseqiien-
te fazendo uma figura. Cada um dos 17 pontos deve ser ligado aos
16 restantes, formando 16 segmentos. Destes, como 16 = 3-5 + 1,
pelo menos 6 devem ser coloridos com a mesma cor. Por exemplo,
suponhamos que A seja o nosso ponto de origem, e que AB, AC, AD,
AE, AF e AG sejam segmentos verdes. Se um dos segmentos que
unem entre si os pontos B, C, D, E, F e G for verde, então já se
forma um triângulo verde. Por exemplo, se BD for verde, então o
triângulo ABD é verde. Por outro lado, se nenhum desses segmentos
Parte II. Resoluções - Princípio da Casa dos Pombos 175

for verde, então devemos notar que B, por exemplo, deve se ligar
a C, D, E, F e G, formando 5 segmentos. Como argumentamos
no item anterior, pelo menos três dentre estes devem ser, digamos,
vermelhos. Suponhamos, então, sem perda de generalidade, que BC,
BD e BE sejam vermelhos. Ora, pelo mesmo raciocínio aplicado no
item anterior, haverá um triângulo de mesma cor (vermelha ou azul)
envolvendo apenas os pontos B, C, D e E.

177. De fato, consideremos o sólido original dividido em 27 paralelepípedos de


dimensões 1, 2 e 3, como ilustra a figura a seguir:

3
EB3

Cada uma dessas partes, como mostra a figura, é um paralelepípedo de


dimensões 1, 2 e 3, e diagonal \/l2 4- 22 4- 32 = x/14. Assim, como são 28
os pontos que se encontram no paralelepípedo, deve haver pelo menos uma
das partes com pelo menos dois pontos, garantindo o resultado desejado.

178. Consideremos a seqüência aj, a2> •••, an2+i- Seja í; o número de ter­
mos da mais longa subseqüência crescente que tem início em ai, para
i = 1,2,..., n2 + 1. Se algum ít- for pelo menos n 4- 1, o problema está
terminado. Caso contrário, isto é, se, para cada i, tivermos 1 < ti < n,
teremos n2 4- 1 t^s a assumirem n valores distintos, a saber, 1, 2, ..., n.
Se cada um desses valores aparecer no máximo n vezes, teremos somente
n2 valores de ti, o que é uma contradição. Logo, existem pelo menos n 4- 1
ti’s iguais. Agora, vamos mostrar que os üí's associados a esses n 4- 1 t,’s
compõem uma subseqüência decrescente. Supondo, para tais termos, que
ti = tj para i < j, devemos mostrar que ai > aj. Como os termos da
seqüência original são distintos, se a, < aj, então a» seguido pelos tj termos
da subseqüência crescente que começa em aj compõem uma subseqüência
crescente de tj + 1 termos começando em a,, donde ti > tj 4- 1, o que é
contraditório. Logo, ai > aj, e o resultado está provado.
176 Problemas Resolvidos de Combinatória

179. (a) Primeiramente, observe que dois números consecutivos quaisquer são
primos entre si. Para que um subconjunto do conjunto dado não pos­
sua elementos primos entre si, ele não poderá conter números con­
secutivos. Ocorre que {1,3,5,...,2n — 1} e {2,4,6,... ,2n} são os
maiores subconjuntos que obedecem tal condição. Como ambos pos­
suem apenas n elementos, e o subconjunto que desejamos deve conter
n + 1 elementos, concluímos que qualquer subconjunto do conjunto
dado possui um par de números consecutivos. Portanto, o resultado
está demonstrado.
Agora, para provar que todo n+ 1-subconjunto do conjunto dado pos­
sui pelo menos um par de números tais que um divide o outro, basta
considerar que todo elemento compreendido entre 1 e 2n pode ser
escrito na forma 2^/71, sendo m ímpar e variando entre 1 e 2n, ou
seja, assumindo, no máximo, n valores distintos. Como os subconjun­
tos considerados devem possuir n + 1 elementos, concluímos que pelo
menos dois dos seus elementos devem ser produto de uma potência de
2 pelo mesmo m, a saber, 2rm e 2sm. Assim, como r s, fica provado
o resultado.
(b) Considere os 8 números abaixo:

43
4343
434343
43434343
4343434343
434343434343
43434343434343
4343434343434343
Como temos mais do que 7 números, pelo menos 2 deles devem possuir
restos iguais ao serem divididos por 7. Tomando a diferença entre esses
2 números, obtemos um múltiplo de 7 da forma 43... 43 • 102A:. Como
102fc não é múltiplo de 7, concluímos que 43... 43 deve ser múltiplo de
7. Assim, encontramos um múltiplo de 7 da forma 43 ... 43. Agora,
ampliando, de maneira conveniente, a seqüência original de números
da forma 43... 43, obtemos tantos múltiplos de 7 quanto desejarmos,
o que prova o resultado solicitado.
Parte II. Resoluções - Princípio da Casa dos Pombos 177

180. Consideremos os números 0, o — [a], 2a— [2a], ..na — [na], e a seguinte


divisão do intervalo [0,1] em n partes:
F -t- -i-------- 1
0 2 n~l 1
n n n

Observe, então, que cada um dos números considerados pertence a este


intervalo. Logo, pelo Princípio da Casa dos Pombos, existe um intervalo
de comprimento ± contendo sa — [soj e ta — [íaj, para 0 < s < t < n.
Tomando, pois, q = t — s e p = [taj — [sa], temos p,q G N, 1 < q < n, e:

|çq -p| = |(t - s)a - ([ia] - LsqJ)I = l(ÍQ _ lÍQJ) ~ (sa - LsqJ)I
donde segue que:
P 1
a---- < —.
nq
Isso prova que todo intervalo da reta, por menor que seja, sempre contén
números racionais, isto é, que o subconjunto Q dos números racionais é
denso no conjunto dos números reais R, como se costuma afirmar em Análise
Matemática.
178 Problemas Resolvidos de Combinatória

Probabilidade

181. (a) Denotemos a e v, respectivamente, cada bola azul e cada bola vermelha
retirada da caixa. Para que o palhaço permaneça seco, em cada ponto
da seqüência formada de a’s e v’s, o número de a's deve ser maior
do que ou igual ao número de v’s. Ou seja, em cada momento, o
número de passos para trás dados até então deve ser maior do que ou
igual ao número de passos para frente. Ora, estamos exatamente nas
condições do Exercício 83, assumindo os a’s em lugar dos c’s e os v's
em lugar dos d’s. Analogamente, o fato do palhaço estar inicialmente
na ponta do trampolim da piscina equivale ao fato do caixa daquele
exercício inicialmente não possuir troco. Logo, o número de maneiras
do palhaço ficar seco é igual a:
n — n + 1 (n +ri 1 (2ri\
n n + 1 \ n )'

tomando m = k = n naquele exercício. Como o total de maneiras de se


retirarem as bolas das caixas é (2^) (de fato, basta decidirmos em quais
momentos retiraremos as n bolas azuis), temos que a probabilidade de
que o palhaço permaneça seco é igual a:

(Í)
(b) Vamos reduzir este problema ao tratado no Exercício 83 de uma manei­
ra muito simples. Uma vez organizadas as duas filas de n pessoas de
acordo com suas alturas, podemos dar 5 reais a cada uma das pessoas
da primeira fila e 10 reais a cada uma das pessoas da segunda. Feito
isto, arranjamos as 2n pessoas numa única fila por altura, obtendo
uma nova fila de 2n pessoas na qual n possuem 5 reais e n possuem 10
reais. Afirmamos, então, que, uma vez tendo feito a organização por
altura, obtemos uma fila “boa” no sentido empregado no Exercício 83.
De fato, considere a pessoa que pertencia à fc-ésima posição da segunda
fila. Então, há pelo menos k pessoas maiores do que ela e que possuem
5 reais (as k primeiras da primeira fila) e, no máximo, k — 1 maiores
do que ela e que possuem 10 reais (as k — 1 primeiras da segunda fila).
Isso implica que não há problemas com troco para nenhum membro
da fila, isto é, que esta fila é “boa”. Além disso, tomando qualquer
pessoa que pertencia à primeira fila, também não há problema com
troco, pois cada uma dessas pessoas já possui 5 reais.
Parte II. Resoluções - Probabilidade 179

Reciprocamente, tomando uma fila “boa” de n pessoas com 10 reais


e n pessoas com 5 reais, podemos supor, sem perda de generalidade,
que as 2n pessoas estão ordenadas por altura. Assim, podemos tomar
aqueles que possuem 5 reais, do maior para o menor, e irmos colocando
um atrás do outro. Assim, estes formarão a primeira fila de n pessoas
dispostas em termos de altura e os n que restam formarão a segunda
fila de n pessoas. Ademais, uma vez que a fila original era “boa”,
considerando-se a fc-ésima pessoa da segunda fila, a qual possui 10
reais, existem pelo menos k pessoas com 5 reais à sua frente, donde
a fc-ésima pessoa da primeira fila é maior do que ela, obedecendo à
organização das filas requerida pelo enunciado. Assim, identificamos
cada uma das ordenações das duas filas com uma única ordenação, que
conta as filas “boas” do Exercício 83 para o caso em que m = k = n,
donde (2^) é a resposta do problema.

182. Note que as faces devem ser números naturais, pois a face de um dado
nunca é nula. Assim, |A| corresponde ao número de soluções positivas da
equação a+ò+c= 9 e |£?| corresponde ao número de soluções positivas
dea + ò + c = 10, contanto que a,b,c < 6. Ora, as únicas soluções de
a + b + c = 9 que devem ser desprezadas são as três formadas pela trinca
(7,1,1). Portanto, |A| = (3Z]) — 3 = (®) — 3 = 25, utilizando o Exercício
25. Analogamente, as soluções de a + 6 + c = 10 que devem ser desprezadas
são as 3! = 6 da trinca (7,1,2) e as 3 da trinca (8,1,1). Portanto, vem
que |B| = (— 6 — 3 = (9) — 9 = 27. É claro, ainda, que o conjunto
de todos os lançamentos dos dados tem cardinalidade 63 = 216, donde

183. (a) Como o número total de r-seqüências formadas por elementos de A é


igual a nr, basta que calculemos o número de r-seqüências sujeitas à
condição de não possuírem termos repetidos. Para sua primeira casa,
existem n opções de preenchimento. Para a segunda, apenas n — 1
opções, visto que não se pode utilizar o mesmo valor que se utilizou na
primeira. Logo, n(n— l)(n—2) • • • [n — (r — 1)] é o número de seqüências
em tais condições. Portanto, a probabilidade desejada é igual a:

n(n — l)(n — 2) • • • (n — r + 1) n!
nr nr(n — r)!'

(b) Como um ano possui 365 dias, se r = 366, então, pelo Princípio da
Casa dos Pombos, pelo menos duas pessoas farão aniversário no mesmo
dia, donde a probabilidade de que duas pessoas façam aniversário no
180 Problemas Resolvidos de Combinatória

mesmo dia é igual a 1. Além disso, se r = 1, então a probabilidade


de haver duas pessoas aniversariando no mesmo dia é zero. Portanto,
é claro que 2 < r < 365. Assim, calculemos primeiro a probabili­
dade de que, num conjunto de r pessoas, não haja duas que aniver-
sariem no mesmo dia. Ora, isto é equivalente ao que foi realizado em
(a) para n = 365. De fato, temos 365r possibilidades de aniversários
para as r pessoas, e 365 • 364 • • • (366 — r) possibilidades de que todas
essas r pessoas façam aniversário em dias distintos. Assim, observando
que a probabilidade de que algum dia do ano seja aniversário de, pelo
menos, duas pessoas é complementar à probabilidade de que cada dia
do ano seja aniversário de, no máximo, uma pessoa, e impondo a
condição do problema, temos que descobrir qual o menor valor de r
que verifica:
365 • 364 • • • (366 - r) 1
1-
365r “ 2
■ • • (366 - r)
> i <=> x = 365 • 364365 r “ 2"

Sugerimos agora que se recorra a um computador ou a uma calcu­


ladora adequada a fim de se encontrar o menor valor de r que torne a
afirmação verdadeira. Observe o que ocorre para os seguintes valores
de r:
r x («) 1 — x (~)
10 0,88 0,12
20 0,59 0,41
22 0,52 0,48
23 0,49 0,51
30 0,29 0,71
40 0,11 0,89
50 0,03 0,97
Na tabela anterior, x é igual à probabilidade de que não haja duas pes­
soas fazendo aniversário no mesmo dia, donde 1 — x é a probabilidade
de que pelo menos duas façam aniversário no mesmo dia. Além disso,
é evidente, embora se possa demonstrar formalmente que, conforme r
aumenta, x diminui e 1 — x aumenta. De fato, quanto maior o número
de pessoas (r), realmente é de se esperar que aumente a chance de haver
duas nascidas no mesmo dia do ano. Assim, por esta observação e pela
da tabela acima, concluímos que r = 23 é o valor procurado. Como in­
terpretação do resultado encontrado, temos que, de um grupo aleatório
de 23 pessoas, é mais provável que haja duas fazendo aniversário no
mesmo dia do que o contrário. E interessante notar, pela tabela acima,
Parte II. Resoluções - Probabilidade 181

o quão rápido cresce a probabilidade em questão. Diante de um grupo


de 50 pessoas, por exemplo, praticamente garantimos a existência de
duas pessoas que nasceram no mesmo dia do ano! E certo, porém, que,
para um grupo real de pessoas, o resultado acima provado não se veri­
fica de maneira tão fiel, pois os dados reais mostram que o número de
pessoas nascidas em determinados dias é ligeiramente diferente do que
em outros, contrariando nossa convenção inicial de como a proba­
bilidade de que uma dada pessoa faça aniversário num dia qualquer
do ano.

184. Contando os casos favoráveis, temos 6 possibilidades para o grupo a ser


escolhido para Brasil e Argentina. Por conseguinte, restam 5 possibilidades
para o grupo dos países Alemanha e Itália. Escolhendo os outros 2 países
de cada um desses grupos e escolhendo os 4 componentes dos outros qua­
tro grupos, temos 6 • 5(22°) (18) (42) (8) (4) maneiras de se fazer a divisão
requerida. Como o total de maneiras de se dividirem os países nos grupos,
sem nenhuma restrição, é igual a (24) (4°) (\2) (4) (4), a probabilidade
solicitada é:

X)!?) 30
1.771'

Por outro lado, se as 4 equipes mencionadas pertencerem ao mesmo grupo,


então há 6 escolhas para tal grupo. Em seguida, basta distribuir as 20
seleções restantes nos demais cinco grupos. Então, nesse caso, a probabili­
dade é:
<W(M) 6 1
(?)(?)(■/) (7) (?)(1) (244)“
1.771

Como vemos, a probabilidade de que Brasil, Argentina, Alemanha e Itália


caiam num mesmo grupo é da probabilidade de que as equipes caiam
em grupos separados, a saber, Brasil e Argentina num grupo, Alemanha e
Itália noutro.

185. (a) A probabilidade de se ganhar um prêmio comprando 4 bilhetes de uma


só vez é igual a jõõTõõ = ?5 ooó = 0,00004. Por outro lado, a proba­
bilidade de ganhá-lo comprando um bilhete em cada uma das quatro
semanas é igual ao complemento da probabilidade de não ganhá-lo
em nenhuma delas. Ora, a probabilidade de não ganhar em determi­
nada semana, comprando um só bilhete, é igual a Portanto, a
182 Problemas Resolvidos de Combinatória

probabilidade de não ganhar em nenhuma das quatro semanas é igual


a ( ioo^oo)4’ d°nde ganhar tem probabilidade:
4
99.999
1- « 0,000039.
100.000
Assim, embora a diferença seja muito pequena, a probabilidade de
ganhar arriscando tudo numa única semana é maior do que a de ganhar
arriscando “aos poucos”. Portanto, o pai deve ouvir o conselho do filho!
(b) Repetindo o raciocínio do item anterior, a probabilidade de se ganhar
comprando n bilhetes num único período é igual a Por outro lado,
a probabilidade de se ganhar comprando um bilhete em cada um de n
períodos é igual a:
n n
TV — 1
1 -
N
=1- 14
Agora, vamos provar que, de fato:
n
n
N
1- 14
o que equivale a:
n
1-—
N 14
Observe, então, que:
n n—1

1-4 14
n—1 n—1
1
14 N 14
n—1

N
n—2
2
>
14 N
> • ■ •

n—n

04 n
N

1-4
Parte II. Resoluções - Probabilidade 183

sendo provado nosso resultado, qual seja, o fato de comprar n bi­


lhetes de uma só vez é mais vantajoso do que comprá-los em n períodos
diferentes.
186. Considere os conjuntos:
A = {nGN|l<n< 500 e n é divisível por 2};
B = {neN|l<n< 500 e n é divisível por 3};
C' = {nGN|l<n< 500 e n é divisível por 5}.
Calculando o número de elementos de A U B U C, saberemos quantos são
os naturais entre 1 e 500 divisíveis por 2, ou por 3, ou por 5. Ocorre que:
|ÁUBUC| =
= |A| +1B| + |C| - |A n B| - |A n c\ - |B n C| + |A n B n C|
500 500 500 500 500 500 500
“ |_ 2 + 3 + 5 6 10 15
+ ~3CT
= 250 + 166 + 100 -83-50-33 + 16
= 366.
Portanto, é a probabilidade de que um número aleatório entre 1
e 500 seja divisível por 2, ou por 3, ou por 5. Logo, a probabilidade de que
um número entre 1 e 500 não seja divisível por nenhum desses três valores
XI 183 _ 67
c 250 — 250-
187. Para que as duas bolas retiradas sejam da mesma cor, podemos retirar duas
bolas vermelhas ou duas bolas azuis, com reposição. Duas bolas vermelhas
podem ser retiradas de a ■ a = a2 maneiras, enquanto duas bolas azuis
podem ser retiradas de b • b = b2 maneiras. Portanto, como o total de
maneiras de se retirarem duas bolas da caixa é igual a 2n • 2n = 4n2, temos
a2+b2

Agora, façamos b = 2n — a na expressão de p, a fim de obtermos:


a2 + (2n — a)2 2a2 — 4na + 4n2 _ 1 2 1
P = P(a) = 4n2 4n2 2n2<1
Note, então, que p = p(a) pode ser vista como função quadrática de a.
Como o coeficiente que multiplica a2 em p(a) é positivo (2^7), p(a) possui
valor mínimo, que pode ser calculado por28:

a = --^- = n,
22^

28Dada uma função f : R —» R, definida por /(x) = aax2 + a\x -I- ao, para 02 > 0, seu valor
mínimo Xmin ® dado por Xmin = 202"
184 Problemas Resolvidos de Combinatória

donde b = 2n — a = n. Portanto, a = b = n minimiza p. Além disso, este


valor mínimo de p pode ser calculado substituindo-se a = b = n em sua
expressão original, isto é, p = = 5.

188. (a) Como os dados possuem cores distintas, podemos denotá-los por di,
d2, CÍ3, CZ4, d$ e de. Temos 6 possibilidades para o valor obtido em
di, 5 para d2, 4 para da, 3 para d^, 2 para ds e 1 para dg (não pode
haver faces repetidas). Portanto, como o total de lançamentos é 66, a
probabilidade de que todas as faces sejam distintas é
(b) O valor que aparecerá em todas as faces é um dos 6 possíveis. Logo,
® a probabilidade, nesse caso.
(c) Existem (3) = 20 maneiras distintas de ocorrer o mesmo valor em
três dados distintos, e o valor que neles aparece pode ser qualquer
um dos 6 possíveis. Em seguida, os valores que aparecerão nas faces
restantes podem ser escolhidos de 5 • 4 • 3 = 60 maneiras. Portanto, a
probabilidade requerida é 6'26°660 =
(d) Temos 6(3) = 120 possíveis resultados para os três dados que conterão
um mesmo valor, 5(2) = 15 possibilidades para os dois dados que
conterão outro valor, e 4 possibilidades para o dado restante. Assim,
a probabilidade é igual a 120665'4 =

189. (a) Como todos os seis dados são idênticos, para que todas as faces sejam
distintas, só há uma possibilidade. Agora, façamos a contagem de
todas as ocorrências possíveis. Ora, para caracterizarmos o que ocorre
num lançamento, basta que determinemos quantas faces de cada valor
aparecem. O número de possíveis lançamentos, desse modo, é igual
ao número de soluções da equação xi + • • • + xq = 6, para Xi > 0,
que é igual a = (V) ~ 462, pelo Exercício 25. Portanto, a
probabilidade de que todas as faces sejam distintas é
(b) Se todas as faces devem conter o mesmo valor, a probabilidade é igual
a _6_ _ J_
d 462 — 77’
(c) Temos 6 possibilidades para o valor que aparece nas três faces iguais.
Quanto às três faces restantes, devemos contar as ocorrências com 3
valores distintos dentre os 5 restantes, o que resulta (3) = 10 maneiras.
Portanto, é a probabilidade, nesse caso.
(d) Agora, são 6 opções para as três faces repetidas, 5 para as outras duas
repetidas, e 4 para a outra face. Logo, é a probabilidade
solicitada.
Parte II. Resoluções - Probabilidade 185

190. Devemos selecionar exatamente k bolas brancas e, conseqüentemente, r — k


bolas pretas. As bolas brancas podem ser selecionadas de (T^1) maneiras
distintas, e as bolas pretas de ( ”2fc) maneiras distintas. Como (”) é o total
de seleções possíveis, a probabilidade de que sejam selecionadas exatamente
k bolas brancas é igual a:

(?)(^)
(?) '
Em Teoria das Probabilidades, dizemos que a expressão acima, como função
de k, segue o modelo hipergeométrico de distribuição.

191. (a) Trabalhar com seleções de cartões sem reposição equivale a efetuar
contagens de números de 7 algarismos distintos dentre 1, 2, 3, 4, 5,
6 e 7. O total de números ímpares, neste caso, é 4 • 6!, sendo que 4
conta as possibilidades para o preenchimento da casa das unidades do
número (1, 3, 5 ou 7) e 6! conta as opções de preenchimento de suas
6 casas restantes. Ora, como 7! é o total de números, a probabilidadt
de que o número obtido seja ímpar é

(b) Agora, uma vez que há reposição dos cartões na caixa, pode haver
repetição de algarismos nos números considerados. Nesse caso, são
4 • 76 números ímpares, de um total de 77 números. Assim, temos
que a probabilidade cujo cálculo nos foi solicitado é = i. Note
que ela coincide com a obtida no item (a). Em geral, deixaremos ao
leitor a tarefa de provar que, se a caixa inicial contivesse n cartões
numerados por 1, 2, 3, ..., 72, então, ao se retirarem 72 cartões com
ou sem reposição, as probabilidades de se obter número ímpar seriam
idênticas (inclusive para 72 par).

192. (a) O total de maneiras de se distribuírem n bolas idênticas em n caixas


distintas corresponde ao número de soluções inteiras não negativas da
equação zi 4- |-a;n = n, que sabemos ser igual a 1-- —1)!
~1\ -2rt~-(2n
/ n!(n—1)! ’
pelo Exercício 25. Agora, para contarmos o número de distribuições
que contêm exatamente uma caixa vazia, basta que determinemos a
caixa desocupada e, em seguida, distribuamos as n bolas idênticas nas
n—1 caixas restantes, de modo que nenhuma fique vazia. Ora, o
total destas distribuições é o número de soluções inteiras positivas da
equação 4- • • • + xn-i = n. Logo, o número de casos favoráveis é
igual a 7i((n27)-i) = n(?-2)1 ~ n(n — !)• Logo, a probabilidade de
186 Problemas Resolvidos de Combinatória

termos exatamente uma caixa vazia é igual a:


n(n — 1) (n — l)(n!)2
(2n—1)! (2n—1)! '
n!(n—1)!

Com o auxílio de um software matemático, pode-se chegar à conclusão


de que n = 2 maximiza esta probabilidade (neste caso, ela vale 2/3),
a qual tende a 0 à medida que n aumenta. Rigorosamente, podemos
escrever:
n[(n+l)!]2 (n+1)
(n + 1)2 (n—l)(n!)2
[2(n+ 1) - 1]! “ 2(2n + l)(n- 1)
l)(n-l) ’ (2n - 1)! ’
para concluir que a seqüência de probabilidades é estritamente decres­
cente para n > 2. De fato, 2(2n+i)(n-i) < Para n — 2- Portanto,
n = 2 maximiza as probabilidades. Além disso:
(n + l)2 _ 1
lim
n—*oo 2(2n+ l)(n - 1) ~ 4*
donde as probabilidades tendem a zero à medida que n cresce.
(b) O total de maneiras de se distribuírem n bolas distintas em n caixas
distintas é igual a nn. De fato, basta que escolhamos em qual das n
caixas cada uma das n bolas será colocada. Para contar o número de
casos em que exatamente uma caixa fica vazia, basta selecionarmos
a caixa vazia e, em seguida, distribuirmos as n bolas distintas nas
n — 1 caixas distintas sem deixar nenhuma vazia. Ora, este número
é igual a nT(n, n — 1), sendo T(n,k) como definido no problema 138.
Todavia, como, em nosso caso, k = n — 1, não precisaremos recorrer à
formula de T(n, fc) para realizar a contagem (tal observação também é
válida para o item anterior, ainda que, naquele caso, não a tenhamos
aproveitado). De fato, basta notarmos que, para que as n — 1 caixas
fiquem ocupadas, deveremos ter, impreterivelmente, n - 2 caixas com
uma única bola e uma caixa com duas bolas. A caixa que conterá
duas bolas pode ser selecionada de n — 1 maneiras. Ademais, as duas
bolas que a ocuparão podem ser selecionadas de (2) maneiras. Em
seguida, basta selecionarmos as caixas das n — 2 bolas restantes de
(n — 2)(n — 3) • • • 2 • 1 = (n — 2)! maneiras. Portanto, a probabilidade,
nesse caso, vale:

nT(n,n — 1) n(n — 1)(a)(n ~ 2)- (n — l)(n — 1)!


nn nn 2nn~2
Parte II, Resoluções - Probabilidade 187

Novamente, um software matemático pode nos ajudar, mostrando que


a probabilidade acima é máxima para n = 3 (neste caso, ela vale 2/3) e,
além disso, que ela tende a 0 conforme n aumenta, sendo sempre maior
do que a probabilidade calculada em (a), para n > 3. Ou seja, para
n > 3, a probabilidade de se distribuírem n bolas distintas em n caixas
distintas deixando uma caixa vazia é maior do que a probabilidade de
se distribuírem n bolas idênticas em n caixas distintas deixando uma
vazia, embora ambas tendam a 0 à medida que n aumenta.

193. (a) Como as bolas de mesma cor são indistinguíveis, só há uma maneira
de se extraírem da urna 3 bolas de cada cor. O número total de
maneiras de se extraírem 9 bolas da urna corresponde ao número de
soluções inteiras não negativas da equação Xi + x% + x$ = 9, para
0 < Xi < 6. A função geradora que controla cada uma das variáveis Xi
é 1 + x + x2 d------ F x6 = Logo, o número de soluções da equação
acima é igual ao coeficiente de x9 na expansão de:

1-x7 3
= (1 - x7)3(l - x)~3 = (1 - 3x7 + 3x14 - x21)(l - x) 3.
1 —X

Este coeficiente é igual a — 3(■‘3S‘) = (V) - 3© = 37,


utilizando o Exercício 25. Logo, a probabilidade de se extraírem 3
bolas de cada cor sem reposição é igual a
(b) Neste caso, também só há uma maneira de se retirarem 3 bolas de
cada cor. O total de extrações possíveis, porém, uma vez que há
reposição, é igual ao número de soluções inteiras não negativas da
equação xi 4- x% + X3 = 9, que sabemos ser (^^í1) — (V) = 55.
Assim, havendo reposição, a probabilidade de se extraírem 3 bolas de
cada cor é
(c) Agora, supondo que as bolas sejam todas distintas, sem resposição,
podemos escolher 3 brancas, 3 pretas e 3 vermelhas de (|) = 8.000
maneiras. O total de maneiras de se retirarem 9 bolas, nesse caso, é
igual a (g8) = 48.620, donde a probabilidade de se retirarem 3 bolas
de caaa cor
ae cada 8-000
cor ée 48 - -á°Q_
620 — 2.431 •
Por outro lado, se as escolhas forem realizadas com reposição, então,
para cada uma das 3 cores, o total de maneiras de se retirarem 3 bo­
las é igual ao número de soluções inteiras não negativas da equação
Xi + x2 + • • • + X6 = 3, que é igual a (^y1) = (5) = 56. Por-
tanto, o total de maneiras de se retirarem 3 bolas de cada cor é
188 Problemas Resolvidos de Combinatória

563 = 175.616. Analogamente, o número de maneiras de se reti­


rarem as 9 bolas da urna corresponde ao número de soluções in­
teiras não negativas da equação aq -I- x? + • • • 4- xis = 9, que é igual
a (■9+18-1
r " ‘
. 18-1 ) = (17) = 3.124.550, donde a probabilidade desejada é
175.616
T"’’
_ 87.808
3.124?550 “ 1.562.275 ’

194. (a) Para que um apostador faça uma quadra, 4 dos 6 números sorteados
devem ter sido por ele escolhidos de maneiras. Quanto às outras
duas dezenas escolhidas, devem estar entre as 42 não sorteadas de (422)
maneiras. Assim, o número de casos favoráveis ao apostador é (8) (422).
Como o total de apostas possíveis é (468), a probabilidade de que 0
apostador acerte exatamente quatro dezenas é:

(!)(?) 4.305
(?) “ 4.090.504

(b) Agora, o apostador deve acertar somente 5 dos 6 números sorteados,


sendo o sexto número por ele escolhido um dos 42 não sorteados. Logo,
a probabilidade, nesse caso, é igual a:

(I) ■ 42 21
(?) 1.022.626

(c) Escolhendo 6 dezenas, só há uma maneira do apostador acertar todas,


donde a probabilidade de que ele ganhe a sena é:
1
(?) 12.271.512’

195. (a) Das 8 dezenas escolhidas pelo apostador, deve haver 3 sorteadas e 5
não sorteadas. As 3 sorteadas podem ser escolhidas de (3) maneiras e
as 5 não sorteadas, de (755) maneiras. Uma vez que o total possível de
apostas distintas é (8g), a probabilidade desejada vale:

©(?) 1.278
(?) 214.643’

Outra maneira de resolver o problema baseia-se na contagem dos


sorteios favoráveis ao apostador. Na resolução anterior, supomos que
o sorteio foi realizado e, então, contamos as escolhas do apostador
Parte II. Resoluções - Probabilidade 189

contendo apenas três dos números sorteados. Analogamente, pode­


mos supor que a escolha do apostador já tenha sido realizada e, então,
contarmos os sorteios que contenham apenas 3 dos números escolhi­
dos. Para tanto, as 3 dezenas acertadas pelo apostador devem estar
entre as 8 por ele escolhidas, e as duas não acertadas, entre as 72 que
ele não escolheu. Logo, como o total de sorteios é (85°), segue que a
probabilidade é igual a:

(5(7) 1.278
(?) ~ 214.643’
naturalmente coincidindo com o resultado anterior.
(b) De modo análogo ao item anterior, resolveremos o problema, a princí­
pio, baseados nas escolhas do apostador, supondo o sorteio já reali­
zado. Sabemos que, das 8 dezenas por ele escolhidas, 4 devem ter sido
sorteadas e 4 não devem ter sido sorteadas. Logo, (4) (45) é o número
de escolhas que o favorecem, donde a probabilidade desejada é igual
a:
(5(7) 315
(“) 1.502.501'
Como outra forma de resolução, podemos efetuar a contagem dos
sorteios que favorecem o apostador, tendo ele feito sua aposta. A fim
de que ele faça a quadra, ele deve ter escolhido quatro dos valores
sorteados de (8) maneiras e, com relação ao quinto valor do sorteio,
deve estar entre os 72 que ele não escolheu. Logo, a probabilidade
desejada é:
(5(7) 315
(?) 1.502.501

(c) Neste caso, aplicando novamente os dois tipos de resolução dos itens
(a) e (b), temos que a probabilidade é igual a:

©(?) (5(7) 7
(?) (?) 3.005.002 ‘

196. (a) Representando os dez carros por q, C2,..., cio, e cada lugar vago por v,
para que não haja vagas vazias consecutivas, basta que consideremos o
total de maneiras de se encaixarem 4 v’s entre as 10 letras de C]C? ■ • • Cio
de modo que cada v fique entre dois Ci’s ou num dos extremos da
permutação. Como são 11 os espaços em tais condições, temos (l4)
190 Problemas Resolvidos de Combinatória

maneiras de encaixar os v’s. Além disso, os Ci’s podem ser permutados


de 10! maneiras. Como o total de maneiras de se alocar os carros
nas 14 vagas é (permutações envolvendo os 10 Cj’s e os 4 u’s), a
probabilidade de não haver vagas vazias consecutivas é:

30
14!
= ---.
4!
91

(b) Considerando as permutações envolvendo os Cj’s e u’s já definidos em


(a), tomando três v’s juntos, temos 12! permutações envolvendo 12
blocos distintos: ci, C2, . .., cio, v e vvv. Todavia, 12! conta duas vezes
as 11! permutações que apresentam o bloco vvvv em sua composição
(uma vez nos casos em que vvv segue-se a v e uma vez nos casos
em que v segue-se a vvv). Portanto, o número de permutações que
possuem exatamente três v's juntos é 12! — 2 ■ 11! = 10 • 11!. Logo, a
probabilidade desejada é:
10 • 11! 10
14! 91
4!

197. (a) Tomando a livros de Português e b de Matemática, distribuindo-os às


pessoas que por eles têm preferência, restam-nos 2n — a — b pessoas
sem livro, n — a livros de Português e n — b livros de Matemática.
Decidindo quais das pessoas receberão os n — a livros de Português, é
evidente que as (2n — a — b) — (n — a) = n — b restantes deverão receber
livros...de
os n — b. ......... —Matemática.
.._____ , . „’ X(„2nn7—aarò>)/ conta o número de
Assim,
distribuições que respeitam as preferências pessoais. Além disso, o
total de maneiras de se distribuírem todos os livros é (2™). De fato,
basta que escolhamos, por exemplo, quais as n pessoas que receberão
os livros de Português. Logo, a probabilidade solicitada é:
/2n—a—b\
X n—a /

e) ■
(b) Dentre os p pés retirados do armário, deve haver k pares de sapatos.
Portanto, 2k sapatos devem estar aos pares e p — 2k não devem possuir
seu par. Os k pares de sapatos podem ser escolhidos de (£) maneiras.
Em seguida, (escolhe quais os pares aos quais pertencem os p—2k
sapatos que não tiveram seu par escolhido. Cada um desses p — 2k
sapatos, finalmente, pode ser pé esquerdo ou direito do seu par. Assim,
Parte II. Resoluções - Probabilidade 191

(fc) ’ 2P 2k é o número de escolhas favoráveis. Como o total de


escolhas é, obviamente, (2”), a probabilidade solicitada vale:

(n\fn-k\ np—2k
\kJ \p—2k) ' Z

(?)
198. Acompanhe a figura abaixo para entender a resolução deste problema:

v.
V^l

c*

Vn+1

Considere o vértice Vi do polígono, como mostra a figura. Considere a


reta r que passa por Ví e pelo centro C da circunferência. E um resultado
simples de geometria o fato de que r é perpendicular a um dos lados do
polígono, a saber, o lado Vn+iVn+2i contando-se a partir de Vi no sentido
horário, como ilustra a figura. A reta r divide a circunferência em duas
semicircunferências contendo, cada uma, n vértices do polígono. Para que
um triângulo tendo Ví como vértice contenha o ponto C em seu interior, é
condição necessária que os dois vértices restantes pertençam à semicircun-
ferências distintas. Com a notação da figura, V2 só pode ser combinado com
VÇi+2 a fim de que o triângulo formado contenha o centro da circunferência
em seu interior. O vértice V3 pode ser combinado com Vn+2 ou com Vn+3 e,
assim, sucessivamente, até VÇj+i, que pode ser combinado com qualquer um
dos n vértices dentre VÇi+2, ^n+3> •■•> ^2n+i- Assim, o total de triângulos
contendo C em seu interior e Ví como vértice é 1 4- 2 4- • • ■ 4- n — .
Repetindo o raciocínio anterior para cada um dos n vértices do polígono, e
dividindo o resultado final por 3, uma vez que cada triângulo é contado três
vezes, obtemos um total de |(2n4-l)n-^2+1-’ = n(n~rlX2n~rl) casos favoráveis.
Como o total possível de triângulos a serem formados é temos que
192 Problemas Resolvidos de Combinatória

a probabilidade do triângulo formado conter o centro da circunferência em


seu interior é:
n(n+l)(2n+l)
6 n+1
(V) 2(2n - 1)’

199. (a) Observe que, como há disputa de terceiro lugar, o jogador que chegar
à semifinal (n — 1-ésima etapa) jogará, necessariamente, mais um jogo
(a saber, a final ou a disputa do terceiro lugar). Assim, não é possível
que um jogador dispute exatamente n — 1 partidas durante o torneio,
donde p(n — 1) = 0. Para 1 < k < n — 2, temos que A deve participar
de exatamente k etapas do torneio. Ora, para que isso ocorra, ele
deve ganhar todas as k — 1 partidas anteriores e perder a k-ésima
partida. A probabilidade de ocorrência de cada um desses k eventos
é igual a | donde, nesse caso, p(fc) = (|)*. Finalmente, para k — n,
basta calcular a probabilidade de que A vença as n — 2 primeiras
partidas. Dessa forma, A chegará até a n — 1-ésima etapa, e jogará,
por conseguinte, n partidas, incluindo a desta eta^a e a da final ou a
da disputa do terceiro lugar. Assim, p(n) = (|)n~ ■
(b) Dado o interesse deste item, duas engenhosas resoluções serão apre­
sentadas.
Em primeiro lugar, suponhamos que p(fc) denote a probabilidade de
que A e B se enfrentem num torneio reunindo 2k jogadores (dentre
os quais A e B se encontram), de modo que p(n) refere-se à resposta
do problema. A fim de que A e B se enfrentem na primeira etapa do
torneio, eles devem ser uma das 2n-1 duplas desta etapa. Supondo
que isso ocorra, os 2n — 2 jogadores restantes podem ser agrupados de:

(2n-2
;)(2(V:
n-i -1)!

maneiras diferentes. Por outro lado, o total de divisões dos 2n jo­


gadores em duplas aleatórias é:

!)(V)-(Ê)©
(2n—1 )'■

Portanto, a probabilidade de A e B enfrentarem-se na primeira etapa


Parte II. Resoluções - Probabilidade 193

é a razão entre os dois valores encontrados anteriormente, ou seja:

(2"-1-1)! 1 (271-1)!
o (2—1)!
(2H-1 - 1)1
(V)
2n-i

2n-1 - 2! • (2n - 2)!


(2")!
2n
2n(2n - 1)
1
2n - r
Assim, a probabilidade de A e B se enfrentarem na primeira etapa é
Caso eles não se enfrentem nesta fase, então eles devem vencer
seus adversários a fim de que se enfrentem em etapas futuras. Ora,
a probabilidade de que A e B não se enfrentem na primeira etapa é
1— = fÃzf > e a probabilidade de que ambos vençam suas partidas
é (j)2 = Desse modo, passamos a ter apenas 2n-1 jogadores no
torneio, incluindo A e B. Por definição, a probabilidade de que A e
B se enfrentem deste ponto em diante é igual a p(n — 1). Portanto,
temos a relação de recorrência:

1 2n — 2 1 . 1 2n -2
p(n) =
2n - 1 + FTirp(n-1) = 2n — 1 + 22(2n - 1) p(n- 1).
Iterando seguidas vezes tal relação, temos:

1 2n - 2 / 1 2n-1 - 2
p(n) =
2n — 1 + 22(2n - 1) V2n-1 - 1 + 22(2n-l _ !)
p(n - 2)

1 1 271-1 - 2
2n — 1
+ 2(2n - 1) +
+ 23(2n - 1) p(n - 2)
1 1
2n — 1
+
2(2n - 1)
2n-i -2 1 on—2 _ 9 \
+ 23(2n - 1) + 22(2n—2
1 1 1 2n~2 - 2
+ 2(2n - 1)
2n — 1 + + 22(2n - 1) + 24(2n - 1)
p(n - 3),
194 Problemas Resolvidos de Combinatória

donde podemos conjeturar e facilmente provar por indução que:


1 1 1 1
p(n) =
2n - 1
+ 2(2n - 1)
+ 22(2n - 1) 4 h 2*-1(2n - 1)
2n-fc+l _ 2
+ 2^4-1 (2n_if(n-fc), (11)

para cada k e N tal que 1 < k < n — 1. Tomando k = n — 1 em


(11), temos que p(n — fc) = p(l) = 1 (torneio reunindo apenas os dois
jogadores A e B) e, portanto:

1 1 1 22 — 2
p(n) =
2n - 1
+’ 2(2n - 1) +•••+ 2n-2(2n _ 1) + 2n(2n - l)^1)
1 1 1 1
+’ 2(2n - 1)
+ ... + + 2^-i (2n - 1)
2n — 1 2n“2(2n - 1)
1 1 1 \
1 + - + -.- + 2n-i J
2n — 1
1
2n — 1 1 “ 2 /

' 2n —1 \
1 2n |
2n - 1 1 I
2 /
1
2n~i’

que é o valor procurado.


A outra resolução deste problema consiste em somar as probabilidades
de A e B se enfrentarem em cada uma das n etapas do torneio. Já
sabemos que a probabilidade de A e B se enfrentarem na primeira
etapa vale 2n1_1. A probabilidade de que A e B se enfrentem na se­
gunda etapa é igual à probabilidade de que eles não se enfrentem na
primeira etapa multiplicada pela probabilidade de ambos passarem à
segunda etapa multiplicada pela probabilidade de enfrentarem-se es­
tando entre os 2n~1 jogadores que passaram à segunda etapa. Com
relação a esta última probabilidade, basta que refaçamos os cálculos
da resolução anterior para chegarmos ao valor de 2n_1i_y. Logo, a
probabilidade de ambos se enfrentarem na segunda etapa é:

1 1 1 2n — 2 1 1
1-
2n-1 4 2n-1 - 1 2n - 1 ' 2(2n - 2) 2(2n - 1)'
Parte II. Resoluções - Probabilidade 195

Novamente, podemos conjeturar e provar por indução que a probabili­


dade de que A e B se enfrentem na fc-ésima etapa é igual a 2fe-i(2"-í)~
Então, como o torneio possui n etapas, a probabilidade requerida é
igual a:
1 1 1 1
2n — 1
+ 2(2n - 1) + ’" +
+ ••• + 2n-1(2n - 1) ' 2n—1 ’

efetuando os mesmos cálculos da resolução anterior.


200. Dada a informação de que não há surpresas nos resultados das partidas
realizadas no torneio, o melhor jogador dentre os 2" será, certamente, o
campeão. Logo, para que o segundo melhor jogador seja vice-campeão,
é necessário e suficiente que ele chegue à partida final contra o melhor
jogador. Além disso, o melhor jogador é o único empecilho à chegada do
segundo melhor à final. Portanto, o segundo melhor jogador (B) não deve
enfrentar o melhor jogador (A) em nenhuma das n — 1 etapas iniciais do
torneio. Como fizemos no Exercício 199, a probabilidade de que A e B não
se enfrentem na primeira etapa é igual a:
1 2n - 2
1 - 2n - 1 “ 2n - 1 ’

Como não há surpresas nos jogos, é claro que A e B vencem suas partidas,
passando à segunda fase do torneio. Nesta etapa, a probabilidade de que
ambos não se enfrentem é:
1 2n-1 - 2 2(2n~1 - 2)
1-
2n-1 - 1 ~ 2n-1 - 1 2n — 2 '
Prosseguindo com este raciocínio, a probabilidade de que A e B não se
enfrentem na fc-ésima etapa é:
1 2n-k+1 - 2 2(2n-fc+1 - 2)
1-
2n-(fc-i) _ | — 2n~k+1 — 1 — 2n~k+2 — 2 ’
para k e N tal que 1 < k < n - 1. Assim, a probabilidade de que os dois
melhores jogadores não se enfrentem até a n — 1-ésima etapa é igual ao
produto das probabilidades de não se enfrentarem em cada uma das n — 1
etapas iniciais. Portanto, o valor que procuramos é:
2n - 2 2(2n-1 - 2) 2(2l’n“2 - 2) 2(22 - 2) 2n~2(22 - 2) 2n-i

: n-2
2n — 1 ’ 2 2«-i _ 2 23 -2 2n - 1 2n - 1
E interessante observar que esta probabilidade é máxima (igual a 1) para
n = 1 e aproxima-se de | à medida que n cresce.
196 Problemas Resolvidos de Combinatória

201. A probabilidade de que uma dada pessoa não aniversarie hoje é igual a |||.
Logo, dentre n pessoas, a probabilidade de nenhuma aniversariar hoje é
(|H)n. Logo, 1 — (j^)n refere-se à probabilidade de que pelo menos uma
dentre n pessoas aniversarie hoje. Pelo enunciado, desejamos encontrar o
menor valor de n que torna verdadeira a seguinte inequação:
n
364 n 1 364 1 1
1- >2~ - <=> n > log 364 - <=> n > 252,65...
365 365 2 365 2

Portanto, o menor valor de n que torna verdadeira a inequação dada e, por


conseguinte, a afirmação do enunciado, é n = 253.
202. Seja Ai o evento que ocorre quando o relé i está fechado. Para que haja
corrente elétrica entre A e B, devem ocorrer, ao mesmo tempo, todos os
eventos de pelo menos um dos itens abaixo:
• Ai, A% e A5;
• Ai, A2, A4 e Aq;
• Ai, A3 e Ag;
• Ai, A3, A4 e A5.
Logo, utilizando o Princípio da Inclusão e Exclusão, a probabilidade de
haver corrente entre A e B é igual a:
P((Ai n a2 n a5) u (Ai n A2 n a4 n A6) u (Ai n a3 n a6)u
(Aí n a3 n A4 n a5)) =
= P(Ai n a2 n a5) + P(Ai n a2 n a4 n a6) + P(Ai n A3 n A6)
+ P(Ai n A3 n A4 n A5) - P(Ai n A2 n A4 n A5 n A6)
- P(Ar n A2 n a3 n A5 n a6) - P(AX n a2 n a3 n a4 n A5)
- P(Ai n a2 n a3 n a4 n a6) - p(Aí n a2 n a3 n a4 n a5 n a6)
- P(Ai n A3 n A4 n A5 n A6) + P(Ai n a2 n a3 n a4 n a5 n A6)
+ P(Ai n A2 n A3 n A4 n A5 n A6)
4- P(Ai n a2 n a3 n A4 n a5 n a6)
+ P(Ai n A2 n A3 n a4 n a5 n A6)
— P(Ai n a2 d A3 n a4 n A5 n Ag)
= p3 + p4 + p3 + p4 - p5 - p5 - p5 - p5 - p,66 - p5 + p6 + p6 + p6
+ p6 - p6
= 2p3 + 2p4 - 5p5 + 2p6
= p3(2p3 - 5p2 + 2p + 2).
Parte II. Resoluções - Probabilidade 197

Utilizando um software matemático, é possível estimar que p > 0,663805


faz com que a probabilidade de haver corrente entre A e B seja maior do
que i.

203. (a) Para que a porta seja aberta na A:-ésima tentativa, todas as k — 1
tentativas anteriores devem ter sido frustradas, e somente a fc-ésima
deve ser bem-sucedida. A cada tentativa, há uma chave a menos no
molho. Assim, a probabilidade de errar a chave na primeira tenta­
tiva é 2=1, uma vez que só uma das chaves é correta. Na segunda, a
probabilidade é igual a 2^2. Prosseguimos com este raciocínio até a
k — 1-ésima tentativa, cuja probabilidade de fracasso é H=|±l.
n—k+Z
Final-
mente, na fc-ésima tentativa, a probabilidade de acerto é ^4+t- L°g°’
a probabilidade desejada vale:
n—1 n—2 n—3 n—k+1 1 1.
n n—1 n—2 n—k+2 n—k+1 n
Uma outra resolução consiste em associar cada conjunto de k tenta­
tivas de abertura da porta com uma fc-seqüência, de tal maneira que
sua i-ésima posição seja ocupada pela chave que se prestou à i-ésima
tentativa de abertura da porta. Ora, o total de seqüências de chaves,
nesse caso, é igual a n(n— l)(n —2) • • • (n —fc + 1), enquanto que o total
de seqüências que contêm a chave correta na fc-ésima posição é igual
a (n — l)(n — 2) • • • (n — k + 1) (número de casos favoráveis). Assim,
por outro caminho, reencontramos:
(n — l)(n — 2) • • • (n - k + 1) 1
n(n — l)(n — 2) • • • (n — k + 1) n

que é a probabilidade desejada.


(b) Como, nesse caso, há reposição das chaves no molho, até a k — 1-
ésima tentativa devemos escolher uma chave incorreta dentre as n
disponíveis. A probabilidade de que isso ocorra é igual a para
cada tentativa. Em seguida, temos £ como probabilidade da chave
correta ser escolhida na fc-ésima tentativa. A probabilidade, então, é
igual a:
n — 1 k-1 1. (n — l)fc 1
n n nk‘
Agora, como fizemos no item (a), podemos contar as k-seqüências
favoráveis e dividir seu número pelo total de k-seqüências de chaves.
Considerando a chave correta na À:-ésima posição, temos (n —
198 Problemas Resolvidos de Combinatória

seqüências favoráveis num total de nk seqüências possíveis. Assim, da


mesma forma, 1— é a resposta procurada.
204. (a) Como só permaneceram m + 1 carros no estacionamento, devem ter
ido embora (n + 1) — (m +1) = n — m carros. Supondo que dois destes
tenham sido o.s que eram adjacentes ao carro do consumidor, o número
de casos favoráveis é igual a /\n—n-2 ■
m—2.,). De fato, basta selecionarmos
quais os outros n — m — 2 carros que deixaram o estacionamento dentre
os n — 2 que se devem considerar. Como o total de maneiras dos
carros deixarem o estacionamento é (n27n) > temos que a probabilidade
solicitada é igual a:
/ n-2 \
\n—m—2/ (n — 2)! m!(n — m)l (n — m)(n — m — 1)
( ” )
\n—m/ m\(n — m — 2)! n! n(n — 1)

Observe que, para os casos m = n — 1 e m = n, a probabilidade deve


ser nula e, para o caso m — 0, ela deve ser igual a 1, coincidindo com
nosso senso comum.
(b) Se Maria ocupar a primeira posição, então João deve ocupar a m + 2-
ésima posição. Se ela ocupar a segunda posição, então ele deve ocupar
a de número m 4- 3. Prosseguimos este raciocínio até o caso em que
Maria ocupar a n — m — 1-ésima posição e João ocupar a n-ésima
posição. Assim, temos 2(n — m — 1) possíveis posições para João e
Maria, pois as posições de ambos podem ser alternadas em cada um
dos casos anteriormente contados. Quanto às n — 2 posições restantes,
podemos preenchê-las com as demais crianças de (n — 2)1 maneiras.
Assim, como o total de filas é nl, a resposta desejada é:
2(n — m — l)(n — 2)! 2(n — m — 1)
n! n(n — 1)

205. (a) Como a ordem de retirada das bolas importa, o total de seleções das n
bolas é igual a 9n. Para que o produtos dos números selecionados seja
divisível por 6, devemos retirar pelo menos um múltiplo de 2 e pelo
menos um múltiplo de 3. Definamos, pois, os seguintes conjuntos:

A = {seleções que não contêm múltiplos de 2}


= {seleções que não contêm 2, nem 4, nem 6, nem 8}
B = {seleções que não contêm múltiplos de 3}
= {seleções que não contêm 3, nem 6, nem 9}.
Parte II. Resoluções - Probabilidade 199

Retirando do total de seleções o número de elementos de A U B,


estaremos contando o número de seleções que contêm pelo menos um
múltiplo de 2 e pelo menos um múltiplo de 3. Trivialmente, temos que
|A| = 5n, |B| = 6n e |A A B| = 3n. Portanto:
[A U B| = |A| + |B| - |A D B| = 5n 4- 6n - 3n.
Assim, das considerações anteriores, resulta que:
9n — 5n — 6n 4- 3n
9"
é a probabilidade desejada.
(b) O total de seleções que podem ser realizadas corresponde ao número
de soluções inteiras não negativas da equação:
Xi 4- x2 4------- 1- xg = n, (12)
sendo Xj o número de bolas i selecionadas. Esse total, bem o sabemos,
é igual a (n^Í71) = (ng8), pelo Exercício 25. Para que o produto
dos números selecionados seja múltiplo de 6, devemos selecionar, no­
vamente, pelo menos um múltiplo de 2 e pelo menos um múltiplo
de 3. Então, definamos os conjuntos A e B como em (a), e con­
temos o número de elementos de A U B. Como as seleções, nesse
caso, desconsideram a ordem das bolas selecionadas, a cardinalidade
de A U B será diferente da calculada no item anterior. Nesse caso,
para contar o número de elementos de A, basta impormos, na equação
exibida em (12), x2 = X4 = x6 = z8 = 0, calculando o número de
soluções inteiras não negativas de x\ 4- X3 4- X5 4- X7 4- xg = n. Logo,
|A| = = 0*44)- Analogamente, impondo X3 = xq = xg = 0,
temos que |B| = ("g-?1) = 0*5 5)’ e X2 = Xs = 14 — = x$ = xg = 0
nos leva a |A A B| = = (nJ2). Deste modo, o número de
seleções que não contêm nenhum múltiplo de 2 ou nenhum múltiplo
de 3 é:
n 4- 4 n 4- 5 n 4- 2
|A U BI = |A| 4- |B| - |A n B| = 4-
4 5 2
Logo, (ng8) — |ÁUB| refere-se ao número de seleções que contêm pelo
menos um múltiplo de 2 e pelo menos um múltiplo de 3. Assim, a
probabilidade desejada vale:

(T) - (nD - ("D + (T)


("D
200 Problemas Resolvidos de Combinatória

Com o auxílio de um software matemático adequado, é possível des­


cobrir diversas propriedades muito interessantes das probabilidades
encontradas nos itens (a) e (b). Em primeiro lugar, elas são iguais
apenas para n = 1, caso em que valem Além disso, à medida
que n cresce, ambas se aproximam de 1, o que é bastante razoável.
Por último, é interessante notar que a probabilidade calculada em
(a), isto é, considerando-se a ordem de retirada das bolas, é sempre
ligeiramente maior do que a calculada em (b) (à exceção de n = 1,
como já observamos), sendo a diferença máxima entre elas o valor
aproximado de 0,098, que ocorre para n = 6.
206. (a) Como o apostador deve escolher exatamente 15 números, somente uma
escolha sua pode resultar no prêmio máximo. Como são (j|) as apostas
possíveis, a probabilidade dele acertar todos os números é igual a:
1 _ 1
(25) ~ 3.268.760'
(b) Esta probabilidade é igual ao complemento da probabilidade de se
ganhar algum prêmio. O apostador ganha um prêmio ao acertar de
11 a 15 dos números sorteados em sua aposta. Desta forma, temos:
i. probabilidade de se acertarem 15 pontos;
Em (a), foi encontrado o valor 326g 760 para esta probabilidade.
ii. probabilidade de se acertarem 14 pontos;
Supondo que já tenha ocorrido o sorteio, as apostas realizadas
devem conter 14 dos 15 números sorteados. O outro número que
ela contém deve estar entre os 10 não sorteados. Logo, (|®) (11°) é o
número de apostas favoráveis, num total de possíveis. Logo,
a probabilidade requerida vale:
150
© 3.268.760'
iii. probabilidade de se acertarem 13 pontos;
Aplicando raciocínio análogo ao de ii., temos o valor:
M . 4.725
(25) 3.268.760'
iv. probabilidade de se acertarem 12 pontos;

©O 54.600
© 3.268.760'
Parte II. Resoluções - Probabilidade 201

v. probabilidade de se acertarem 11 pontos.

(ac?) 286.650
© 3.268.760
Como os eventos calculados nos itens acima são independentes, a pro­
babilidade de que o apostador receba algum prêmio é igual à soma das
probabilidades encontradas, ou seja, 332681760 = biãsãO- Logo, a proba­
bilidade de que o apostador não seja premiado é:
15.733 132.847
1-
148.580 “ 148.580’
valor aproximadamente oito vezes maior do que a probabilidade de
ganhar.

207. Quando o matemático constata que uma das caixas está vazia, para que
a outra caixa contenha k palitos, 2n — k devem ter sido por ele consumi­
dos, e a 2n — k 4- 1-ésima seleção deve ser referente a uma caixa vazia.
Associando cada conjunto de escolhas de palitos com uma 2n — k 4- 1-
seqüência, devemos selecionar quais as n posições dentre as 2n — k iniciais
ocupadas pelos palitos da caixa vazia de (2n~fc) maneiras. Deste modo,
as posições dos palitos da caixa que contém k palitos também ficam de­
terminadas. A probabilidade de que cada uma das posições da seqüência
seja respeitada é igual a | pois, a cada cigarro, a caixa da qual será re­
tirado o palito para acendê-lo é escolhida aleatoriamente. Logo, como há
2n — k 4-1 posições e há duas possibilidades para a caixa a ser esvaziada, a
probabilidade requerida vale:

2
2n-k\ /i\2"-k+1 = (';V) (2n —- k)\
(2n
. " 1W ■2~n~k n\(n — fc)! • 22n-fc

208. (a) A probabilidade de resultar número par é igual à soma das probabili­
dades de resultar 2, 4 ou 6, isto é:

p(2) + p(4) 4- p(6) = 4- | 4- i = 5


12 o b 12’

(b) A probabilidade de resultar número não maior do que 3 é igual à soma


das probabilidades de que resulte 1, 2 ou 3, ou seja:

p(l) 4- p(2) + p(3) = i 4- 4- = 7


12’
202 Problemas Resolvidos de Combinatória

(c) A probabilidade de não resultar 4 é igual a:

1_p(4) = l-l = |.

209. (a) Denotemos por p a probabilidade de C\ ganhar o primeiro turno da


eleição (p(Ci) — p). Pelo enunciado da questão, temos £>(62) = P,
p(C3) = 2, pÇC^ = e p(C5) = Logo, como os eventos em
questão são independentes, vem que:

p(Ci) + p^) + p(C3) + p(Ci) + pÇC^ = 1 =>


p 2p 2p
=>p + p + ^ó +v
O
+v
O
=1

3
^P=ü-
Dessa maneira, p(Ci) = PÍÇz} = H’ PÍ^3) = ÍT e P(^) = p(Cs) = Tr
(b) Neste item, denotaremos p(C{) a probabailidade do candidato Ci ga­
nhar o segundo turno, tendo já ganho o primeiro. Caso apenas C\ e
C4 passem para o segundo turno da eleição, como suas chances de ga­
nhar serão proporcionalmente mantidas, teremos que p(C() = %p(Cj),
donde:
Q 2
p(CÍ) + p(Cj) = 1 =s- -p(CÍ) + p(Cl) = 1 => p(&4) = -

e, por conseguinte, p(C{) =


. Referências Bibliográficas

[1] BACHX, A. C., POPPE, L. M. B., TAVARES, R. N. O. Prelúdio à Análise


Combinatória. São Paulo: Companhia Editora Nacional, 1975. 238 pp.

[2] BALAKRJSHNAN, V. K. Introductory Discrete Mathematics. Toronto:


Dover Publications, 1991. 236 pp.

[3] CAMERON, P. J. Combinatorics: topics, techniques, algorithms. Cam-


bridge: Cambridge University Press, 1994. 355 pp.

[4] CHARALAMBIDES, C. A. Enumerative Combinatorics. Boca Raton:


Chapman & Hall/CRC, 2002. 609 pp.

[5] FELLER, W. An Introduction to Probability Theory and its applications.


New York: John Wiley & Sons, 1950. 509 pp.

[6] GOODMAN, A. W., RATTI, J. S. Finite Mathematics with applications.


New York: Macmillan Publishing, 1975. 541 pp.

[7] KOSTRIKIN, A. L, MANIN, Y. I. Linear Álgebra and Geometry. Moscou:


Moscow University Press, 1981. 312pp.

[8] MARCUS, M. A Survey of Finite Mathematics. New York: Dover Publica-


tions, 1969. 486 pp.

[9] MORGADO, A. C. O., CARVALHO, J. B. P., CARVALHO, P. C. P., FER-


NANDEZ, P. Análise Combinatória e Probabilidade. Rio de Janeiro: Insti­
tuto de Matemática Pura e Aplicada, 1991. 171 pp.

[10] NIVEN, I. Mathematics of Choice: how to count without counting. Washing­


ton: Mathematical Association of America, 1965. 202 pp.

[11] RIORDAN, J. Combinatorial identities. New Jersey: John Wiley &: Sons,
1968. 256 pp.
204 Problemas Resolvidos de Combinatória

[12] ROBERTS, F. S. Applied Combinatorics. New Jersey: Prentice Hall, 1984.


606 pp.

[13] SANTOS, J. P. O., MELLO, M. P., MURARI, I. T. C. Introdução à Análise


Combinatória. Campinas: Editora da Unicamp, 1995. 295 pp.

[14] SCHEINERMAN, E. R. Matemática Discreta: uma introdução. São Paulo:


Pioneira Thomson Learning, 2003. 532 pp.

[15] STANLEY, R. P. Enumerative Combinatorics. Monterey: Wadsworth &


Brooks/Cole Advanced Books & Software, 1986. 306 pp. v. 1

[16] TUCKER, A. Applied Combinatorics. New York: John Wiley & Sons, 1980.
447 pp.

[17] VILENKIN, N. Y. Combinatorics. New York: Academic Press, 1971. 296


pp.

[18] WHITWORTH, W. A. Choice and chance. New York and London: Hafner
Publishing Company, 1965. 342 pp.
Anotações
•m sol 9,

rtíorp -llCo

em mente o livro

uai o primeiro
0(0' gTogfe
0

lemas aqui

ISBN 978 85-399 0168-5

788539 901685

Você também pode gostar